Board study Flashcards

1
Q

A 2-week-old boy is brought to the clinic for persistent vomiting. The neonate is exclusively fed a standard cow milk–based formula. He takes 4 oz every 2 hours and vomits after every feeding, despite burping. The emesis is not bloody, bilious, or projectile. He has 4 yellow, seedy, soft stools per day without blood. He is otherwise healthy. The neonate was born at term via an uncomplicated delivery and had an unremarkable newborn nursery course. His weight today is greater than his birth weight. His vital signs and physical examination findings are unremarkable.

Of the following, the MOST likely cause of this neonate’s symptoms is

A.	gastroesophageal reflux disease
B.	milk protein allergy
C.	overfeeding
D.	pyloric stenosis
A

C. overfeeding

How well did you know this?
1
Not at all
2
3
4
5
Perfectly
2
Q

A 12-year-old, 40-kg boy in the pediatric intensive care unit with acute respiratory distress syndrome is currently intubated and mechanically ventilated, and has persistent hypoxemia. His last arterial blood gas analysis showed a pH of 7.22, pCO2 of 65 mm Hg, and a pO2 of 58 mm Hg. After adjusting the ventilator, his arterial blood gas analysis shows a pH of 7.25, pCO2 of 63 mm Hg, and pO2 of 80 mm Hg.

Of the following, the MOST likely change made to the mechanical ventilator settings was a(n)

A.	decrease in positive end-expiratory pressure from 8 to 7 cm H2O
B.	decrease in tidal volume from 300 to 240 mL
C.	increase in inspiratory time from 0.7 to 0.9 seconds D.	increase in respiratory rate from 22 to 24 breaths/minute
A

C. increase in inspiratory time from 0.7 to 0.9 seconds

The child in the vignette has acute respiratory distress syndrome (ARDS) and persistent hypoxemia. The change made to the mechanical ventilator settings resulted in an improvement in oxygenation (pO2) with negligible change in ventilation (pCO2). Oxygenation is a function of mean airway pressure. The main drivers of mean airway pressure are peak inspiratory pressure (PIP), positive end-expiratory pressure (PEEP), and inspiratory time fraction (i-time) (Item C2A). Thus, the most likely ventilator change that resulted in a higher pO2 was an increase in the inspiratory time fraction. Decreasing the PEEP would have resulted in the opposite effect and likely a lower pO2. A decrease in the tidal volume would not have increased the pO2 and would have caused an increase in pCO2. An increase in the respiratory rate would have little effect on oxygenation and would have decreased the pCO2.

How well did you know this?
1
Not at all
2
3
4
5
Perfectly
3
Q

An 18-month-old boy is brought to the emergency department by his parents after they found him in the garage coughing and sputtering next to an open bottle of lamp oil. Prior to arrival his parents removed the clothing that was soiled with lamp oil. The boy is awake, alert, and interactive. His vital signs include a heart rate of 125 beats/min, respiratory rate of 35 breaths/min, and blood pressure of 85/60 mm Hg. He has audible wheezing but no stridor and is in mild distress. The boy’s oral mucosa is pink and moist with no lesions. His skin is warm and well perfused, and he does not have a rash. The remainder of the boy’s physical examination findings are normal. A chest radiograph is obtained and shown in Item Q81.

Of the following, the BEST intervention for this boy is the administration of

A.	activated charcoal
B.	corticosteroids
C.	ipecac syrup D.	oxygen
A

The boy in the vignette has ingested and aspirated lamp oil, which contains hydrocarbons. Given that the boy has respiratory distress and bilateral infiltrative disease on his chest radiograph, oxygen should be administered immediately, and he should be admitted to the hospital for further observation and management.

PREP Pearls
Aspiration of hydrocarbon-containing substances is related to the hydrocarbon’s low viscosity and low surface tension.
Asymptomatic children seen after hydrocarbon exposure should be observed for at least 6 hours with serial examinations; if they remain asymptomatic they can be safely discharged to home.
Children with any respiratory symptoms after hydrocarbon exposure should receive oxygen and a trial of β<span>2</span>-agonists if there are signs of bronchospasm. Chest radiography should be performed.
Neither prophylactic antibiotics nor systemic corticosteroids should be administered routinely in pediatric hydrocarbon aspiration.

How well did you know this?
1
Not at all
2
3
4
5
Perfectly
4
Q

A term neonate had Apgar scores of 8 and 9 at 1 and 5 minutes, respectively, after birth. After delivery, he was active and crying, with room air oxygen saturations ranging from 93% to 97%. Approximately 10 minutes after birth, when he was quiet and calm, the neonate became cyanotic, his oxygenation saturation level decreased to 65% to 70%, and he developed significant respiratory distress with subcostal retractions and nasal flaring. He was treated with bag/mask positive-pressure ventilation using inspired fractionated oxygen of 100% without significant improvement in clinical status. He was, therefore, intubated, and positive-pressure ventilation was continued. The neonate’s clinical status improved almost immediately after intubation. His oxygen saturation increased to 100%; oxygen supplementation was quickly weaned and discontinued. He was placed on continuous positive airway pressure of 5 cm H2O. He is now pink, well perfused, and breathing comfortably in room air, with oxygen saturations of 95% to 98%.

Of the following, the BEST next step in this neonate’s management is to

A.	obtain an arterial blood gas B.	obtain echocardiography
C.	pass a nasogastric tube through both nares
D.	place an umbilical venous catheter
A

c
The neonate in the vignette most likely has bilateral choanal atresia. After delivery, he had no respiratory distress and had normal oxygen saturation levels. He quickly developed oxygen desaturation and respiratory distress when he became quiet and calm. The best next step in his management would be to pass a nasogastric tube through botThe neonate’s clinical status markedly improved once he was provided a stable airway through an endotracheal tube, which is suggestive of nasal obstruction rather than a cardiac lesion; therefore, echocardiography is not immediately indicated. An arterial blood gas and an umbilical venous catheter may be needed as part of ongoing care but would not be prioritized over the attempt to pass an nasogastric tube.

h nares. In the case of bilateral choanal atresia, the physician would be unable to pass the tube through either naris.
Bilateral choanal atresia in a neonate is a medical emergency because neonates are obligate nose breathers.
Passage of a nasogastric tube through both nares should be attempted in a neonate with suspected choanal atresia.
A neonate with choanal atresia or stenosis should undergo evaluation for associated anomalies, such as those seen in CHARGE syndrome

How well did you know this?
1
Not at all
2
3
4
5
Perfectly
5
Q

A 3-year-old boy is evaluated for a 1-month history of intermittent fevers up to 40°C, night sweats, and 2.2-kg weight loss. He has no known sick contacts and has not traveled recently. The boy is afebrile. He is irritable but consolable and appears well nourished and well developed. There are multiple subcentimeter cervical lymph nodes palpable, more on the left side than the right, and a 4 × 3–cm, firm, immobile, nontender, left-sided, supraclavicular lymph node. His spleen is palpable 1 cm below the left costal margin. The remainder of his physical examination findings are unremarkable.

Of the following, the BEST next step in this boy’s management is referral to a(n)

A.	emergency department
B.	gastroenterologist
C.	infectious disease specialist
D.	otolaryngologist
A

The boy in the vignette has cervical and supraclavicular lymphadenopathy. Cervical lymphadenopathy is very common and has many possible etiologies. Supraclavicular lymphadenopathy, however, is highly concerning for malignancy (eg, lymphoma) and requires further investigation with a lymph node biopsy. Of the response choices, an otolaryngologist is best suited to perform a biopsy. Interventional radiologists and pediatric surgeons also perform lymph node biopsies, but these specialties are not among the response choices.

Supraclavicular lymphadenopathy is usually pathologic and requires further investigation with a lymph node biopsy.
Persistent or worsening lymphadenopathy requires further investigation.

How well did you know this?
1
Not at all
2
3
4
5
Perfectly
6
Q

A 13-month-old girl is seen for a health supervision visit. She was diagnosed with perinatally acquired HIV infection at 2 weeks of age and is currently receiving combination antiretroviral therapy. She has no history of opportunistic infections. She received her routine childhood immunizations at ages 2, 4, and 6 months including Haemophilus influenzae type b (Hib) conjugate vaccine and pneumococcal conjugate vaccine (PCV13). Her physical examination findings are normal. Laboratory data are notable for a CD4+ T lymphocyte count of 750 cells/µL (reference range for age 1-5 years, ≥1,000 cells/µL), CD4+ percentage of 25% (reference range for age 1-5 years, ≥30%), and an HIV viral load of 4,900 copies/mL.

Of the following, in addition to diphtheria-tetanus-pertussis (DTaP), Hib, PCV13, and hepatitis A, the MOST appropriate vaccine(s) to administer today is

A.	measles-mumps-rubella
B.	measles-mumps-rubella and varicella
C.	measles-mumps-rubella-varicella
D.	varicella
A

The girl in the vignette is living with HIV infection and has evidence of low-level immunosuppression based on the absence of opportunistic infections, a CD4+ T lymphocyte count greater than 500/µL, and CD4+ percentage greater than 22%. In this setting, both measles-mumps-rubella (MMR) and varicella vaccines are indicated. In addition, all inactivated vaccines, including diphtheria-tetanus-pertussis (DTaP), Haemophilus influenzae type b (Hib), pneumococcal conjugate vaccine 13 (PCV13), and hepatitis A vaccines, should be administered at this visit. Given the lack of safety data, children with HIV infection should not receive the quadrivalent measles-mumps-rubella-varicella (MMRV) vaccine.

Administration of live vaccines, such as measles-mumps-rubella (MMR) and varicella, is recommended for children and adolescents living with HIV infection who have low-level or no immunosuppression.
Children living with HIV with high-level immunosuppression (defined as a CD4+ T lymphocyte percentage of less than 15% in children aged 1 through 13 years, or a CD4+ T lymphocyte count of less than 200 cells/µL in adolescents aged 14 years and older) must not receive measles-mumps-rubella (MMR) or varicella vaccines.
Children with immunosuppression, including HIV infection, should receive killed vaccines.

How well did you know this?
1
Not at all
2
3
4
5
Perfectly
7
Q

A 2-year-old girl with a noncontributory medical history is brought to the emergency department for intermittent abdominal pain over the past 12 hours. The pain, though progressively worsening, waxes and wanes. She had 3 episodes of nonbloody, nonbilious vomiting today. Over the past 6 hours, the girl has had a significantly decreased activity level and intermittent irritability.

Her vital signs include a temperature of 37.9°C, heart rate of 130 beats/min, respiratory rate of 24 breaths/min, and blood pressure of 108/76 mm Hg. On physical examination, the girl appears tired and cries intermittently while drawing her legs in toward her chest. She has right lower quadrant tenderness without rebound or guarding. The remainder of her physical examination findings are unremarkable.

Abdominal ultrasonography findings are shown in Item Q5.

Of the following, the BEST next step in this girl’s management is a(n)

A.	air enema procedure
B.	computed tomography of the abdomen
C.	emergency laparotomy
D.	upper gastrointestinal imaging with small bowel series
A

The girl in the vignette has signs and symptoms concerning for intussusception. Abdominal ultrasonography demonstrates the characteristic “target sign” (Item C5), also known as a “bull’s eye” or “coiled spring,” which represents layering of intestine within the intestine. Because the girl is hemodynamically stable, performing an air enema is the best next management step. An air enema can be both diagnostic and therapeutic. If the air enema fails to reduce the intussusception, an emergency laparotomy may be necessary. Surgical intervention is indicated as the initial treatment if free air is visualized on imaging studies or there is evidence of peritonitis on physical examination.

PREP Pearls
The classic triad of abdominal pain, currant-jelly stools, and a palpable abdominal mass is present in less than 15% of children with intussusception.
An air enema can be both diagnostic and therapeutic for intussusception.
Extreme lethargy or altered mental status may be the only presenting symptom of intussusception.

How well did you know this?
1
Not at all
2
3
4
5
Perfectly
8
Q

A 2-year-old girl was seen for a routine health supervision visit 2 weeks ago. At that time, her weight was inadvertently entered into the medical record in pounds but labeled as kilograms. The girl’s mother called the advice line last night because her daughter had a fever and was provided an acetaminophen dose based on the incorrect weight. The pediatrician noticed the error when reviewing the chart this morning and called the girl’s mother for follow-up. The girl received 1 dose of acetaminophen since the overnight phone call, is feeling better, and is not exhibiting negative consequences of the dosing error. The dose administered was well below a hepatotoxic level.

Of the following, this event is BEST categorized as a/an

A.	adverse event B.	medical error
C.	non-preventable adverse event
D.	sentinel event
A

b

How well did you know this?
1
Not at all
2
3
4
5
Perfectly
9
Q

A 3-month-old male infant born at term is brought to the emergency department via ambulance for a seizure. Initial evaluation shows an ionized calcium level of 3.3 mg/dL (0.8 mmol/L) (reference range, 4.5-5.3 mg/dL [1.1-1.3 mmol/L]). He is treated with intravenous calcium, which aborts the seizure. The infant’s parents report that he has been jittery and irritable for the past few days. He exclusively breastfed for the first 2 months after birth. Due to fussiness, his parents recently started making formula using a recipe found on the internet containing hemp seed hearts, coconut water, dates, and sea moss. The infant was recently diagnosed with laryngomalacia after an evaluation for noisy breathing. His physical examination findings are normal for age. His parents are of normal stature.

Laboratory evaluation drawn prior to treatment reveals the following:

Laboratory Test

Result

Total calcium

4.5 mg/dL (1.1 mmol/L) (reference range, 9-11 mg/dL [2.2-2.8 mmol/L])

Phosphorous

1.2 mg/dL (reference range, 2.7-4.5 mg/dL)

Magnesium

2.1 mg/dL (0.9 mmol/L) (1.6-2.6 mg/dL [0.7-1.1 mmol/L])

Parathyroid hormone

534 pg/mL (reference range, 10-65 pg/mL)

Alkaline phosphatase

1,021 U/L (reference range, 146-477 U/L)

25-hydroxyvitamin D

Pending

1,25-dihydroxyvitamin D

Pending

Of the following, the BEST next step in this infant’s management is oral administration of

A.	calcium, cholecalciferol, and calcitriol
B.	calcium and phosphorus
C.	magnesium
D.	phosphorus and calcitriol
A

The infant in the vignette has hypocalcemia due to severe vitamin D deficiency. His noisy breathing and seizure are manifestations of hypocalcemia. After treating his symptomatic hypocalcemia with intravenous calcium, the infant should be treated with oral calcium and cholecalciferol (dietary vitamin D3). Calcitriol (1,25-dihydroxyvitamin D, the active form of vitamin D) should be added given the severity of his hypocalcemia. Calcitriol acts immediately to absorb calcium from the intestine while vitamin D stores are replenished by cholecalciferol. The infant should also be switched to a cow milk–based formula, given its higher vitamin D content, once he can safely drink from a bottle.

Breast milk does not provide adequate vitamin D; infants who are exclusively or predominantly breastfed must receive supplementation with this vitamin. Although commercial formulas are fortified with vitamin D, homemade formulas, including the one described in the vignette, do not contain additional vitamin D. The American Academy of Pediatrics recommends that breastfed infants receive at least 400 IU of supplemental vitamin D daily.

Treatment with phosphorus and calcitriol is indicated for hypophosphatemia due to defects in the fibroblast growth factor 23 (FGF23) pathway, usually due to mutations in the PHEX gene. Fibroblast growth factor 23 is the main hormone responsible for phosphorus wasting in the kidney

In mild vitamin D deficiency, relative hypocalcemia will result in a rise in parathyroid hormone (PTH) to maintain serum calcium in the normal range. Calcium is reabsorbed from the kidney, absorbed from the gut (via conversion of remaining 25-hydroxyvitamin D stores to 1,25-dihydroxyvitamin D), and released from the bones. As vitamin D deficiency becomes more severe and PTH rises higher, calcium is depleted and levels begin to fall. Increased PTH levels cause phosphate wasting in the kidney, resulting in low serum phosphorus levels. Parathyroid hormone has an indirect effect on osteoclasts, resulting in increased bone resorption. Alkaline phosphatase levels rise as bone turnover is increased. When 25-hydroxyvitamin D stores are extremely low, there is inadequate vitamin D available for conversion to 1,25-dihydroxyvitamin D, which results in low levels of this hormone. The infant in the vignette is expected to have low 25-hydroxyvitamin D and 1,25-dihydroxyvitamin D levels
PREP Pearls
The American Academy of Pediatrics recommends supplementation of 400 IU daily of vitamin D for exclusively breastfed infants.
Manifestations of severe vitamin D deficiency may include rickets (widening of the wrists, rachitic rosary, craniotabes) and hypocalcemia with secondary hyperparathyroidism.
Homemade infant formulas may contain or lack ingredients that may affect mineral homeostasis.

How well did you know this?
1
Not at all
2
3
4
5
Perfectly
10
Q

A 12-day-old male neonate in the neonatal intensive care unit develops apnea, bradycardia, and oxygen desaturation requiring an increase in continuous positive airway pressure ventilation (CPAP) from 6 to 8 cm H2O, and fraction of inspired oxygen (FiO2) from 0.23 to 0.40. His mean upper limb cuff blood pressure has ranged from 18 to 25 mm Hg for the past hour. The neonate was born at 26 weeks’ gestation with a birthweight of 940 g. Delivery was by cesarean section due to maternal preeclampsia with rupture of membranes at delivery. The neonate has been tolerating nasogastric feedings of 20 calorie/oz breast milk at 80 mL/kg/day and receiving parenteral nutrition through a central intravenous line.

Laboratory data are shown:

Laboratory Test

Result

Arterial blood gas base deficit

10

White blood cell count

22,000/µL (22.0 × 109/L)

Neutrophils

40%

Bands

25%

Lymphocytes

20%

Monocytes

11%

Eosinophils

4%

Hemoglobin

10.8 g/dL (108 g/L)

Hematocrit

30.5%

Platelet count

87 × 103/µL (87 × 109/L)

Blood cultures are pending.

Of the following, the BEST treatment for this neonate is intravenous

A.	ampicillin and cefotaxime
B.	ampicillin and gentamicin
C.	meropenem and oxacillin
D.	vancomycin and gentamicin
A

The preterm neonate in the vignette has signs and symptoms of late-onset sepsis with clinical deterioration and an elevated white blood cell count with a left shift. The most likely causative organisms are coagulase-negative staphylococci and gram-negative bacteria. Of the response choices, the combination of vancomycin (effective against coagulase-negative staphylococci) and gentamicin (effective against gram-negative bacteria such as Escherichia coli and Klebsiella pneumoniae) is the best treatment pending blood culture results.

Ampicillin, gentamicin, cefotaxime, and meropenem are all effective against gram-negative bacteria, but they are not effective against coagulase-negative staphylococci. Oxacillin is also not effective against coagulase-negative staphylococci. Ampicillin and gentamicin is the empiric antibiotic combination of choice in early-onset sepsis and is effective against the common bacterial causes of early onset sepsis (eg, group B Streptococcus, E coli, and Listeria monocytogenes).

PREP Pearls
Risk factors for late-onset sepsis in preterm neonates include immature defense barriers, an immature immune system, invasive interventions, and comorbidities of prematurity.
The choice of empiric antibiotics for late-onset sepsis in a preterm neonate should be based on the most likely causative organisms, which include coagulase-negative staphylococci and gram-negative bacteria.

How well did you know this?
1
Not at all
2
3
4
5
Perfectly
11
Q

The mother of a 10-year-old girl with cystic fibrosis (CF), jointly managed by her pediatrician and a multidisciplinary CF center at a nearby children’s hospital, calls for advice about a positive airway culture. The girl was diagnosed with CF after a positive newborn screening result. She has been very healthy and has required no aggressive intervention for pulmonary infection. Her routine airway cultures, obtained in the CF center by deep throat/gag swab, have always grown normal flora or methicillin-sensitive Staphylococcus aureus. Her most recent culture was positive for Pseudomonas aeruginosa. The girl has no symptoms of illness. She is eating well and has had no weight loss. She is attending school regularly.

Of the following, the MOST appropriate management strategy in response to this girl’s laboratory finding is

A.	hospitalization for intravenous antibiotic administration
B.	inhaled antibiotic treatment for eradication of the identified organism
C.	no intervention while she is asymptomatic
D.	repeat airway culture before implementing treatment
A

The most appropriate management strategy for the girl in the vignette, who is experiencing her first acquisition of Pseudomonas aeruginosa infection, is an inhaled anti-pseudomonal antibiotic. Multiple studies demonstrate that proactive treatment to eradicate Pseudomonas from the airway in children with cystic fibrosis (CF) helps preserve lung function. A single 28-day treatment course of inhaled tobramycin given twice daily, with follow-up culture to document resolution and re-treatment if the culture is still positive, is the standard of care in the United States. The addition of oral antibiotics for acute eradication has not been shown to be more effective than an inhaled antibiotic alone. However, concurrent oral azithromycin has been demonstrated to delay symptomatic pulmonary exacerbation of cystic fibrosis in children with first acquisition of Pseudomonas. There are no data to support hospitalization for treatment with intravenous antibiotics as more effective than treatment with an inhaled antibiotic alone for eradication of Pseudomonas in a child with asymptomatic first acquisition.

PREP Pearls
Chronic colonization with Pseudomonas aeruginosa and/or methicillin-resistant Staphylococcus aureus is associated with a decline in lung function in children and adolescents with cystic fibrosis.
The most appropriate treatment of first acquisition of Pseudomonas in an individual with cystic fibrosis is 28 days of inhaled antipseudomonal antibiotic, usually tobramycin.
Cystic fibrosis-related diabetes is a later-onset complication of progressive pancreatic fibrosis.

How well did you know this?
1
Not at all
2
3
4
5
Perfectly
12
Q

A 5-year-old girl is seen for a health supervision visit. She attends kindergarten and is doing very well. Her medical history is significant for recurrent joint subluxations of the hips, shoulders, knees, and elbows bilaterally, easy bruising, and poor wound healing often requiring placement of sutures to achieve closure. The family history is significant for similar findings in her father and grandfather. The girl’s father had a recurrent incisional hernia at an appendectomy site. There is no family history of arterial aneurysm or rupture. On physical examination, there are bruises on her lower extremities, widened atrophic scars with evidence of previous sutures (Item Q10), and skin and joint hypermobility. The remainder of her physical examination findings are normal.

Of the following, the girl’s MOST likely diagnosis is

A.	Ehlers-Danlos syndrome
B.	homocystinuria
C.	Loeys-Dietz syndrome
D.	Marfan syndrome
A

The girl in the vignette has classical Ehlers-Danlos syndrome (cEDS). Ehlers-Danlos syndrome is a group of inherited connective tissue disorders. Cardinal features of EDS include joint hypermobility, skin hyperextensibility, and tissue fragility. The 2017 international classification describes 13 subtypes of EDS. The most common forms are classical, vascular, and hypermobile. There is wide phenotypic variability and genetic heterogeneity among the different EDS subtypes. The underlying genetic etiology is known for all subtypes of EDS except hypermobile EDS. Classical Ehlers-Danlos syndrome is inherited in an autosomal dominant manner. The clinical diagnosis of cEDS is made based on the presence of major and minor criteria.

Homocystinuria is characterized by the following features:

Central nervous system: developmental delay, intellectual disability
Eyes: ectopia lentis, severe myopia
Vascular system: thromboembolic episodes
Skeletal: tall stature, long limbs, scoliosis, pectus excavatum
Clinical features of Loeys-Dietz syndrome are described below:

Craniofacial: hypertelorism, bifid uvula/cleft palate, strabismus, craniosynostosis
Cutaneous: velvety, translucent skin, easy bruising, dystrophic scars
Skeletal: pectus excavatum/carinatum, scoliosis, joint hypermobility, arachnodactyly, cervical spine malformation, clubfeet
Vascular: cerebral, thoracic, and abdominal arterial aneurysms and/or dissections

How well did you know this?
1
Not at all
2
3
4
5
Perfectly
13
Q

A 4-year-old girl is evaluated in the emergency department for 6 days of bloody diarrhea. Her mother reports that her daughter appears tired and pale. The girl drank only 4 oz of fluids today and has not urinated for more than 24 hours. The girl’s 2 older sisters also have diarrhea, but their symptoms are improving. The family recently vacationed at a cabin on a lake in a rural area. They did not drink any unfiltered lake water but drank the tap water.

The patient has a heart rate of 145 beats/min, a respiratory rate of 20 breaths/min, and blood pressure of 85/55 mm Hg. She is ill-appearing, and her capillary refill time is 4 seconds. The remainder of her physical examination findings are unremarkable.

Of the following, the MOST likely cause of this girl’s illness is

A.	Escherichia coli
B.	Giardia duodenalis
C.	norovirus
D.	Yersinia enterocolitica
A

The girl in the vignette has bloody diarrhea that most likely resulted from drinking contaminated well water, a common source of water in rural areas. The most likely cause of her illness is infection with Escherichia coli (possibly hemolytic uremic syndrome). E coli, including E coli 0157:H7 and other forms of Shiga toxin–producing E coli, are common well-water contaminants. Drinking water contaminated with Giardia duodenalis, norovirus, and Yersinia enterocolitica can also cause diarrhea but is less likely to produce bloody diarrhea.

How well did you know this?
1
Not at all
2
3
4
5
Perfectly
14
Q

A 14-year-old adolescent girl is evaluated in the emergency department for fever of 3 days’ duration, sore throat, dysphagia, and a slightly muffled voice. She is otherwise healthy, fully immunized, and does not have a history of recurrent throat infections. Her family history is unremarkable. On physical examination, her temperature is 39°C, heart rate is 100 beats/min, respiratory rate is 18 breaths/min, and oxygen saturation is 99% in room air. She is in mild discomfort due to her throat pain but is speaking comfortably in complete sentences. Her left tonsil is 3+ enlarged and erythematous, her right tonsil is 1+, and her uvula is deviated to the right (Item Q14). She has enlargement and tenderness of her left cervical lymph nodes. The remainder of her physical examination findings are unremarkable.

Of the following, the MOST appropriate next management step for this adolescent is

A. drainage of the abscess
B. intravenous antibiotics
C. observation only
D. tonsillectomy

A

The adolescent in the vignette has a peritonsillar abscess (PTA); drainage of the abscess is the most appropriate first step in treatment. Intravenous antibiotics may be administered as the first step in treatment for individuals with certain complications resulting from PTAs (eg, septic thrombophlebitis). Observation alone is not appropriate management and may result in complications. Tonsillectomy for an acute infected PTA may be performed in certain individuals, such as those who would require general anesthesia for incision and drainage and those with a history of recurrent tonsil infections, but would not be the appropriate first-line treatment for this otherwise healthy adolescent.

How well did you know this?
1
Not at all
2
3
4
5
Perfectly
15
Q

A 13-year-old adolescent girl is seen in the office for evaluation of lower abdominal pain and hematochezia of 3 months’ duration. Her symptoms have progressed from mild lower abdominal pain and loose stools to severe, crampy lower abdominal pain with bloody, liquid stools 6 to 8 times per day. She has urgency and tenesmus, and passes nocturnal stools. Her family has noted pallor over the last few weeks, and she reports dizziness. The girl is otherwise healthy and does not take any medication. Her mother has rheumatoid arthritis and thyroid disease.

The girl is pale and quiet. She is afebrile, with a heart rate of 150 beats/min, blood pressure of 110/65 mm Hg, weight of 38 kg (13th percentile for age), height of 146 cm (4th percentile for age), and body mass index of 18 kg/m2 (30th percentile for age). She has conjunctival pallor, dry mucous membranes, and a soft systolic murmur. Her abdomen is soft, tender to palpation diffusely without rebound or guarding, and grossly bloody stool is noted on the rectal examination.

Laboratory evaluation in the office demonstrates a hemoglobin level of 7.8 g/dL (78 g/L) and positive stool occult blood test.

Of the following, the BEST next step in the management of this adolescent is

A.	immediate referral to the local emergency department for evaluation and care
B.	oral iron supplementation with repeat hemoglobin test in 1 week
C.	referral to pediatric gastroenterology for appointment within 1 week
D.	stool infection studies with follow-up office appointment in 24 hours
A

The adolescent in the vignette has significant anemia, evidence of ongoing lower gastrointestinal bleeding, and vital signs that are concerning for hypovolemia (tachycardia). She needs to be urgently evaluated and stabilized in the emergency department.

The initial step in the evaluation of children with gastrointestinal bleeding (either through vomitus or stool) is the assessment of hemodynamic stability based on vital signs (tachycardia, hypotension, orthostatic changes), capillary refill, and mental status. If hemodynamic instability is noted, appropriate intravenous (IV) access and IV fluid will be necessary, with consideration of transfusions of packed red blood cells, platelets, and/or coagulation factors as clinically appropriate. Laboratory data, including hemoglobin level, platelet count, coagulation studies, liver function tests, albumin level, and type and cross-match (in preparation for potential blood transfusions), should be obtained. The presence of blood should be confirmed with a stool occult blood test, as stool or vomitus can appear red with the ingestion of certain foods, dyes, and medications. Urgent consultation with a pediatric gastroenterologist is indicated because children with refractory gastrointestinal bleeding and ongoing hemodynamic instability despite the administration of IV fluids and blood products may require emergent therapeutic endoscopy.

PREP Pearls
Initial assessment of a child with gastrointestinal bleeding includes evaluation of hemodynamic stability.
Children with gastrointestinal bleeding and hemodynamic instability need urgent intravenous access and intravenous fluid administration, and may require transfusion of blood products.
Upper gastrointestinal bleeding may manifest as hematemesis, melena, or hematochezia. Lower gastrointestinal bleeding may present as melena or hematochezia.

How well did you know this?
1
Not at all
2
3
4
5
Perfectly
16
Q

A 4-month-old, full-term, developmentally normal infant is evaluated in the outpatient office for paroxysmal episodes for the past 2 weeks. During the episodes, his body crunches up repeatedly, “as though he is doing baby sit-ups.” Initially, it occurred once or twice per day, usually in the evening, but it now occurs in clusters, at which point he becomes difficult to console. His mother suspects that he is uncomfortable and might have gastroesophageal reflux. She is concerned because the episodes are becoming more frequent.

The infant’s vital signs are normal. His physical examination findings, including neurological, are normal. During the visit, the infant had a typical episode in which he stiffened, his arms extended out suddenly and he bent forward. This occurred in a short cluster during which he appeared uncomfortable and cried.

Of the following, the BEST next step in this infant’s management is

A.	referral to a gastroenterologist for outpatient evaluation
B.	referral to a neurologist for outpatient evaluation
C.	reflux precautions and close follow-up in the pediatrician’s office
D.	urgent neurological evaluation through the emergency department
A

The infant in the vignette’s episodes are suggestive of infantile spasms (IS), a condition requiring prompt diagnosis and treatment to optimize developmental outcome. Infantile spasms, the most common epilepsy syndrome in infancy, is clinically characterized by a triad of:

Epileptic spasms
Electroencephalogram background of hypsarrhythmia
Accompanying developmental plateau and regression

PREP Pearls
Infantile spasms, the most common epilepsy syndrome in infancy, are clinically characterized by a triad of: 1) epileptic spasms, 2) electroencephalogram background of hypsarrhythmia, and 3) accompanying developmental plateau and regression.
A high index of suspicion and urgent evaluation with expedited electroencephalography are necessary to confirm the diagnosis of infantile spasms and quickly initiate treatment.
Targeted evaluation for underlying causes of infantile spasms, focusing on treatable conditions, should occur concomitantly with treatment initiation and is tailored toward the individual patient. Long-term developmental outcome is impacted by time to treatment and underlying etiology.

How well did you know this?
1
Not at all
2
3
4
5
Perfectly
17
Q

A 5-year-old, previously healthy boy is seen in the clinic for evaluation of testicular pain and fever that started this morning. He had a cough and runny nose 2 weeks ago, which have resolved. He has no other symptoms and no history of trauma. On physical examination, the boy has a temperature of 38.5°C, heart rate of 117 beats/min, respiratory rate of 22 breaths/min, and oxygen saturation of 98% in room air. Both testicles are palpated in the scrotum and the cremasteric reflex is intact. The testicles appear swollen, with no abnormal coloring or masses palpated. Ultrasonography with Doppler shows normal blood flow to both testicles.

Of the following, this boy’s MOST likely diagnosis is

A.	inguinal hernia
B.	postinfectious orchitis
C.	testicular torsion
D.	torsion of the appendix testis
A

The boy in the vignette has fever, painful testicular swelling, and a normal cremasteric reflex. Of the response choices, these signs and symptoms are most consistent with orchitis. In this age group, orchitis is most commonly seen after viral infections. An inguinal hernia can cause scrotal pain and swelling but would be palpated during the genitourinary examination. Testicular torsion, a surgical emergency, is an unlikely diagnosis for the boy in the vignette given his normal blood flow on Doppler ultrasonography and normal cremasteric reflex. In the case of testicular torsion, the normal cremasteric reflex (retraction of the testis in response to touch on the upper thigh) is usually absent. Torsion of the appendix testis is also unlikely given the normal findings on ultrasonography and absence of a “blue dot sign” on physical examination.

Orchitis (inflammation of the testis) and epididymitis (inflammation of the epididymis) have infectious or inflammatory etiologies. Symptoms and signs of orchitis and epididymitis include testicular swelling and tenderness, dysuria, urinary frequency and urgency, and often systemic symptoms (eg, fever). The etiology of orchitis and epididymitis varies by age. Children ages 2 to 13 years often have a postinfectious cause (most commonly Mycoplasma, enterovirus, or adenovirus). Vasculitis (eg, Henoch-Schönlein purpura) is another common cause of orchitis and epididymitis in this age group. Older children and adults are more likely to have an infectious etiology with organisms that cause urinary tract or sexually transmitted infections. A urinalysis may show the presence of nitrites and/or leukocyte esterase because the infection can start in the urine and ascend into the epididymis and testes.

PREP Pearls
All children and adolescents with testicular pain should have ultrasonography with Doppler performed to evaluate for testicular torsion.
Symptoms and signs of orchitis and epididymitis typically include testicular swelling and tenderness, dysuria, urinary frequency and urgency, and often systemic symptoms.

How well did you know this?
1
Not at all
2
3
4
5
Perfectly
18
Q

A 9-month-old girl is seen for follow-up 3 weeks after hospital admission for a febrile Escherichia coli urinary tract infection. She received intravenous ceftriaxone in the hospital and subsequently completed treatment with oral cefdinir. The girl is currently asymptomatic, feeding well, and having regular bowel movements. At 6 months of age she had a febrile urinary tract infection. She is alert and active, and has vital signs that are normal for age. Her physical examination findings, including those of the genitourinary examination, are unremarkable.

Renal ultrasonography performed in the hospital was normal. A voiding cystourethrogram showed bilateral grade III vesicoureteral reflux.

Of the following, the MOST appropriate next management step for this infant is

A.	polyethylene glycol treatment, orally
B.	repeat voiding cystourethrogram in 3 months
C.	trimethoprim-sulfamethoxazole prophylaxis, orally
D.	ureteral reimplantation surgery
A

The infant in the vignette has recurrent febrile urinary tract infection (UTI) and a voiding cystourethrogram (VCUG) that shows bilateral vesicoureteral reflux (VUR). The most appropriate next management step for this girl is trimethoprim-sulfamethoxazole prophylaxis to prevent recurrent UTI.

Vesicoureteral reflux is the retrograde passage of urine from the bladder to the upper urinary tract. Vesicoureteral reflux is divided into primary and secondary types. Primary VUR occurs due to a congenitally short segment of ureter within the bladder wall, which results in incomplete closure of the ureterovesical junction during bladder contraction. Secondary VUR occurs as a result of high bladder pressure associated with conditions such as posterior urethral valves, bladder-bowel dysfunction, and neurogenic bladder.

Voiding cystourethrogram is the test of choice to confirm the presence of VUR and grade its severity. Indications to perform a VCUG in neonates with a history of antenatal hydronephrosis include presence of moderate to severe hydronephrosis or ureteral dilation on the postnatal ultrasonography. A VCUG is also indicated in a child with a first febrile UTI with abnormal renal ultrasonography, or a child with recurrent febrile UTI.

PREP Pearls
Vesicoureteral reflux may be diagnosed during the evaluation of a febrile urinary tract infection, antenatally diagnosed hydronephrosis, or screening in children with family history.
Spontaneous resolution of vesicoureteral reflux is common when it is low grade, unilateral, and in those with asymptomatic antenatally diagnosed hydronephrosis.
Antibiotic prophylaxis is indicated for vesicoureteral reflux in non–toilet-trained children, those with low-grade vesicoureteral reflux with recurrent urinary tract infection, and those with high-grade vesicoureteral reflux.

How well did you know this?
1
Not at all
2
3
4
5
Perfectly
19
Q

A 12-year-old boy is seen in the office for evaluation of bilateral anterior knee pain. His pain began about 3 months ago during wrestling practice. There was no acute injury. He reports no swelling, locking, or instability. The pain is worse with kneeling, running, and jumping. There is tenderness over the inferior aspect of both patellae. He also has pain with resisted knee extension. The remainder of the boy’s physical examination findings are unremarkable.

Of the following, the MOST likely cause of this boy’s pain is

A.	Osgood-Schlatter disease
B.	patellar tendinopathy
C.	prepatellar bursitis
D.	Sinding-Larsen-Johansson syndrome
A

The most likely diagnosis for the boy in the vignette is Sinding-Larsen-Johansson syndrome (SLJ) or inferior patellar pole apophysitis. An apophysis is a bony ossification center adjacent to a minor growth plate. Apophyses are found at sites where tendons attach to bone. Contraction of a muscle causes the tendon to pull on the apophysis. The physis, the growth area adjacent to the apophysis, is made of soft bone that has not yet calcified and is especially vulnerable to injury. With repetitive activity, traction on the apophysis can lead to pain. Overuse of the quadriceps muscles and direct pressure on the bottom of the patella (eg, kneeling) cause irritation of the apophysis, known as apophysitis.

PREP Pearls
Apophysitis occurs because the physis, the growth area adjacent to the apophysis, is made of soft bone and is especially vulnerable to injury.
Sinding-Larsen-Johansson syndrome is an apophysitis of the inferior patellar pole apophysis. Osgood-Schlatter disease is an apophysitis of the tibial tubercle.
Tendinopathy and bursitis, while frequently seen in adults, are uncommon causes of knee pain in children.

How well did you know this?
1
Not at all
2
3
4
5
Perfectly
20
Q

A 20-month-old, previously healthy girl is brought to the urgent care center with a 1-day history of left otalgia and increased fussiness. She has had rhinorrhea and cough for the past week, and developed a fever of 38.8°C 6 hours ago that resolved after 1 dose of acetaminophen. There has been no otorrhea. She is drinking, eating, stooling, and voiding well. She has no past history of otitis media. On physical examination, she is afebrile and has clear rhinorrhea in both nares and 2 tender, mobile, soft, 1-cm posterior cervical lymph nodes. The appearance of her left tympanic membrane is shown in Item Q21 (only ear effusion). The remainder of her physical examination findings are normal.

Of the following, the BEST next step in this girl’s management is

A.	high-dose amoxicillin for 5 days
B.	high-dose amoxicillin for 7 days
C.	high-dose amoxicillin for 10 days
D.	observation for 48 to 72 hours
A

The girl in the vignette has acute otitis media (AOM), which is an acute bacterial infection of the middle ear with fluid, otherwise known as suppurative otitis media. According to the American Academy of Pediatrics Acute Otitis Media Clinical Practice Guidelines, the best treatment for children younger than 24 months of age without severe signs or symptoms is observation for 48 to 72 hours with analgesic administration as needed followed by an antibiotic prescription if symptoms worsen or persist over that time frame.

The diagnostic criteria for AOM include:

Recent onset of ear pain (<48 hours)
Bulging tympanic membrane
Reduced mobility of the tympanic membrane with pneumatic otoscopy or tympanometry
Severe disease is defined as one of the following:

Otalgia that is moderate or severe in nature lasting for at least 48 hours
Temperature of 39°C (102.2°F) or higher

PREP Pearls
The diagnostic criteria for acute otitis media include an acute onset of ear pain (<48 hours), a bulging tympanic membrane, and reduced mobility of the tympanic membrane with pneumatic otoscopy.
Severe acute otitis media is defined as otalgia that is moderate or severe in nature lasting for at least 48 hours, or a temperature of 39°C (102.2°F) or higher.
For children aged 6 months to less than 2 years with unilateral acute otitis media that is not severe, observation for 48 to 72 hours is appropriate, after which antibiotic treatment is indicated for persistent or worsening symptoms.

How well did you know this?
1
Not at all
2
3
4
5
Perfectly
21
Q

A newborn is examined 4 hours after birth in the normal newborn nursery. She was born after 39 weeks’ gestation in an uncomplicated vaginal delivery to a 22-year-old woman with no significant medical history. Apgar scores were 8 and 9 at 1 and 5 minutes, respectively. On physical examination, the neonate has a soft boggy swelling over the occipital area that crosses suture lines with bruising and scattered petechiae on the scalp. The remainder of the physical examination findings are normal.

Of the following, the BEST next step in the management of this condition is

A.	head ultrasonography
B.	serial hemoglobin levels
C.	serial physical examinations
D.	serum bilirubin levels
A

c

PREP Pearls
Caput succedaneum is a common newborn scalp swelling found immediately after birth.
Caput succedaneum typically crosses suture lines and resolves spontaneously within 48 to 72 hours.
Due to the risk of severe blood loss associated with subgaleal hemorrhage, which results from tearing of the bridging veins connecting the scalp and the intradural sinuses, it is important to differentiate this condition from caput succedaneum.

How well did you know this?
1
Not at all
2
3
4
5
Perfectly
22
Q

A 3-year-old girl is admitted to the hospital for evaluation and treatment of acute-onset systemic hypertension with an arterial blood pressure of 130/88 mm Hg. She is otherwise healthy. A parenteral antihypertensive medication is prescribed for administration every 6 hours in order to achieve optimum efficacy once the medication serum concentration is at steady state. The elimination half-life of the medication is 6 hours and the medication follows first-order kinetics. After the second dose, the girl remains hypertensive with a blood pressure of 120/65 mm Hg but is otherwise asymptomatic.

Of the following, the MOST likely time-frame, after administration of the initial dose, for this medication to achieve optimum efficacy is

    A.	12 hours
B.	18 hours
C.	24 hours
D.	30 hours
A

The medication administered to the girl in the vignette follows first-order kinetics and has a 6-hour elimination half-life. When given in regular intervals, this medication will achieve steady state concentration around 30 hours after the initial dose (6-hour elimination half-life × 5 half-lives). Elimination half-life is defined as the time it takes for the plasma concentration of the drug or the total amount of drug in the body to be reduced by 50%. It is only applicable to drugs that exhibit first-order kinetics, in which a constant fraction of drug is eliminated per unit time. The steady state of a medication concentration is achieved after the fifth half-life. It takes 5 elimination half-lives for ~97% of the bioavailable medication dose to be eliminated from the body. Thus, in this vignette, it is not surprising that after the second dose this girl has not achieved optimal blood pressure control.

How well did you know this?
1
Not at all
2
3
4
5
Perfectly
23
Q

A 2-month-old infant is brought to the emergency department in January for trouble breathing. He has had progressively worsening rhinorrhea and cough for the past 2 days. Over the past 8 hours, he has not been tolerating his breast milk feedings due to congestion and rapid breathing. The infant was born at 31 weeks’ gestation and has otherwise been healthy.

On physical examination, he has a temperature of 38.0°C, heart rate of 149 beats/min, respiratory rate of 46 breaths/min, and oxygen saturation of 89% in room air. He appears to be in mild respiratory distress. Copious clear rhinorrhea and subcostal retractions are present. End expiratory wheezing is heard in both lung fields. The remainder of his physical examination findings are normal.

Of the following, the MOST likely pathogen causing this infant’s illness is

A.	coronavirus
B.	influenza virus
C.	parainfluenza virus
D.	respiratory syncytial virus
A

The history and physical examination findings of the infant in the vignette, with an antecedent upper respiratory tract infection, low-grade fever, and respiratory distress, are consistent with the diagnosis of acute bronchiolitis. Bronchiolitis is a common inflammatory illness of the lower respiratory tract in infants and young children. Although many community-acquired respiratory viruses can cause bronchiolitis, respiratory syncytial virus (RSV) is the most common cause in infants followed by human rhinovirus, human metapneumovirus, and parainfluenza virus. Other viral pathogens such as influenza and coronavirus rarely cause bronchiolitis. In addition, influenza is associated with high-grade fever.

How well did you know this?
1
Not at all
2
3
4
5
Perfectly
24
Q

A 16-year-old adolescent girl is brought to the emergency department for evaluation of frequent voiding for 2 months. She has 10 to 12 large-volume voids in a day. She has no burning with urination or urinary incontinence. She typically drinks 5 to 6 L of water per day. The girl’s heart rate is 82 beats/min, respiratory rate is 16 breaths/min, and blood pressure is 110/70 mm of Hg. Her weight is at the 10th percentile and height is at the 25th percentile for age. She has no neurologic deficits, and the rest of the examination findings are unremarkable.

Laboratory data are shown:

Blood

Result

Sodium

133 mEq/L (133 mmol/L)

Potassium

4.1 mEq/L (4.1 mmol/L)

Chloride

98 mEq/L (98 mmol/L)

Bicarbonate

24 mEq/L (24 mmol/L)

Blood urea nitrogen

7 mg/dL (2.5 mmol/L)

Creatinine

0.6 mg/dL (53 μmol/L)

Glucose

72 mg/dL (4 mmol/L)

Serum osmolality

270 mOsm/kg
(reference range, 275-305 mOsm/kg)

Urine

Result

Specific gravity

1.005

Leukocyte esterase

Negative

Nitrite

Negative

Blood

Negative

Protein

Negative

Glucose

Negative

Urine osmolality

150 mOsm/kg
(reference range, 300-900 mOsm/kg)

Of the following, the MOST likely diagnosis for this adolescent girl is

A.	diabetes insipidus
B.	diabetes mellitus
C.	primary polydipsia
D.	syndrome of inappropriate antidiuretic hormone
A

The girl in the vignette has polyuria, polydipsia, low serum sodium, low serum osmolality, and dilute urine, favoring a diagnosis of primary polydipsia.

Polyuria is defined as urine volume production of more than 2 L/m2 or 40-50 mL/kg in 24 hours. Primary polydipsia is characterized by the consumption of an excessive amount of fluids, leading to polyuria and dilute urine. It is commonly seen in individuals with psychiatric conditions (eg, schizophrenia, depression, and bipolar disorders) and is often referred to as psychogenic polydipsia. Primary polydipsia can be seen in healthy individuals who drink large quantities of water out of habit; this condition is referred to as habitual polydipsia or compulsory water drinking.

The differential diagnosis of primary polydipsia includes diabetes insipidus, which also presents with polyuria, polydipsia, and dilute urine. The child in the vignette has hyponatremia (hypo-osmolality), whereas the serum sodium level is high or normal in diabetes insipidus. A water deprivation test can be performed to differentiate primary polydipsia from diabetes insipidus when the diagnosis is not clear. In primary polydipsia, polyuria decreases, and the urine osmolality increases after water deprivation. However, in diabetes insipidus, polyuria persists, and the urine osmolality is low after water deprivation.

PREP Pearls
Primary polydipsia is characterized by consumption of an excessive amount of fluids, leading to polyuria, low serum sodium level, low serum osmolality, and dilute urine.
Primary polydipsia can be asymptomatic or associated with symptoms of hyponatremia (eg, nausea, vomiting, lethargy, confusion, ataxia, and seizures).
The treatment for primary polydipsia is restriction of water intake.

How well did you know this?
1
Not at all
2
3
4
5
Perfectly
25
Q

A 10-month-old female infant is brought for a health supervision visit. She was last seen 4 months ago. She has been well since that visit, with no significant illness. She has been feeding well, taking 4 to 5 ounces of formula 4 to 5 times/day plus pureed foods and yogurt. At birth, her weight was at the 30th to 40th percentile; at her previous health supervision visit, her weight was at the 15th to 20th percentile. She says “mama” and “dada,” and is able to pull up to stand but does not crawl. She is followed by cardiology for a heart murmur. The girl’s temperature is 36.5°C, heart rate is 100 beats/min, respiratory rate is 28 breaths/min, blood pressure is 96/45 mm Hg, and oxygen saturation is 100% in room air. Her weight is 7.3 kg (8th percentile) and length is 65 cm (<3rd percentile). She appears pale and is in no apparent distress. There is a 3/6 systolic ejection murmur, lungs are clear to auscultation bilaterally, and her abdomen is soft and nontender with no hepatosplenomegaly. There are no focal neurologic deficits. The remainder of her physical examination findings are normal.

Laboratory data are shown:

Laboratory Test

Result

White blood cell count

7,400/µL (7.4 × 109/L)

Hemoglobin

3.2 g/dL (32 g/L)

Platelet count

314 × 103/µL (314 × 109/L)

Mean corpuscular volume

115 fL

Neutrophils

22%

Lymphocytes

71%

Monocytes

7%

Reticulocyte

0.8%

Absolute reticulocyte count

6 × 103/µL

Hemoglobin A

81.6%

Hemoglobin A2

2.4%

Hemoglobin F

16%

Adenosine deaminase

1.98 (reference range, 0.33-0.95 U/g of hemoglobin)

Of the following, the MOST appropriate treatment for this girl’s condition is

A.	erythropoietin administration
B.	long-term transfusion therapy
C.	no intervention with continued observation
D.	single transfusion with continued observation
A

The girl in the vignette has a history and laboratory findings that are consistent with a pure red blood cell aplasia, most likely Diamond-Blackfan anemia (DBA). This diagnosis is supported by severe macrocytic anemia without reticulocytosis, elevated fetal hemoglobin level, and elevated erythrocyte adenosine deaminase (eADA) level. Her cardiac murmur may be evidence of a congenital heart defect (CHD); there is an increased prevalence of congenital anomalies (eg, cardiac, genitourinary, thumb) in children with DBA. Genetic testing may reveal a mutation in a ribosomal protein, however, not all mutations have been identified.

The treatment for DBA is long-term blood transfusion therapy, which is required to support the child’s growth. Leucodepleted red blood cells should be administered to avoid febrile reactions, cytomegalovirus transmission, and human leukocyte antigen (HLA) alloimmunization. It is important to obtain a blood sample for hemoglobin electrophoresis and eADA testing before administering a blood transfusion, as those tests must be performed on the child’s native blood cells and would otherwise not be accurate until 3 months after a blood transfusion.

PREP Pearls
Diamond-Blackfan anemia is a rare form of pure red blood cell aplasia with severe macrocytic anemia without reticulocytosis, elevated fetal hemoglobin level, and elevated erythrocyte adenosine deaminase level.
There is an increased prevalence of congenital anomalies (eg, cardiac, genitourinary, thumb) in children with Diamond-Blackfan anemia.
Transient erythroblastopenia of childhood is a form of pure red blood cell aplasia that most often occurs in very young children (typically >1 year of age) after a viral infection. The condition spontaneously resolves within a few weeks.

How well did you know this?
1
Not at all
2
3
4
5
Perfectly
26
Q

A 13-year-old adolescent boy is being seen for follow-up after an urgent care visit several days ago for 1 week of fatigue preceded by 2 days of fever and sore throat. Results of the laboratory tests obtained at that visit are shown:

Laboratory Test

Result

Complete blood cell count with differential

Normal with mildly elevated atypical lymphocytes

Rapid streptococcal antigen

Negative

Heterophile antibody spot test for infectious mononucleosis

Positive

Epstein-Barr virus capsule antigen IgG

Positive

Epstein-Barr virus capsule antigen IgM

Positive

Epstein-Barr virus early antigen

Negative

Epstein-Barr virus nuclear antigen

Negative

Antinuclear antibody (ANA) titer

1:40

Thyrotropin

Normal

The boy’s mother is very concerned about the ANA titer result because her husband’s sister was recently diagnosed with systemic lupus erythematosus. The boy’s symptoms are significantly improved. He has no significant medical history, and the results of a review of systems are negative. His vital signs and findings on a physical examination today are normal.

Of the following, the BEST next step in this boy’s management is to

A.	advise that the test result is likely due to a viral infection
B.	obtain an antinuclear antibody profile evaluation
C.	refer him to a pediatric rheumatologist
D.	treat him with a 5-day course of corticosteroids
A

The adolescent in the vignette has a low positive antinuclear antibody (ANA) titer most likely due to his recent Epstein-Barr virus infection. Low ANA titers (<1:320) are usually not indicative of an autoimmune process, so he should not be referred to a rheumatologist at this time and should not have an ANA profile evaluation performed, unless he exhibits other specific signs of rheumatologic disease. He also does not need to receive corticosteroids for mononucleosis because his symptoms have significantly improved.

PREP Pearls
Up to one-third of healthy children may have a positive antinuclear antibody titer result.
Low antinuclear antibody titers (<1:320) are usually not indicative of an autoimmune process and may be due to viral illnesses, such as mononucleosis or parvovirus.

How well did you know this?
1
Not at all
2
3
4
5
Perfectly
27
Q

A 3-year-old girl is brought to the emergency department after drinking an unknown amount of mouthwash. The girl’s mother found her with an empty bottle of mouthwash in the bathroom. She is unsure how much had been in the bottle. The girl had 4 episodes of nonbloody, nonbilious vomiting prior to arriving at the emergency department. She has a temperature of 36.7°C, heart rate of 138 beats/min, respiratory rate of 12 breaths/min, and blood pressure of 108/66 mm Hg. She appears lethargic and is poorly responsive to painful stimuli. The remainder of the girl’s physical examination findings are unremarkable. Her airway, breathing, and circulation are stabilized.

Of the following, the BEST next step in this girl’s management is to

A.	administer activated charcoal
B.	administer fomepizole
C.	order an arterial blood gas analysis
D.	order a bedside blood glucose level
A

The girl in the vignette has ingested an unknown amount of mouthwash, which contains ethanol (ethyl alcohol). In addition to beer, wine, and liquor, ethanol is found in a wide variety of household items including hand sanitizer, liquid cough and cold medications, cooking extracts (eg, vanilla), mouthwash, and perfumes/colognes. Because ethanol inhibits hepatic gluconeogenesis, children (especially young children) who ingest ethanol can become profoundly hypoglycemic. Therefore, the best next management step for the girl in the vignette is to obtain a bedside blood glucose level.

How well did you know this?
1
Not at all
2
3
4
5
Perfectly
28
Q

A pediatrician is called to an urgent cesarean delivery at 38 weeks’ gestation due to fetal bradycardia. The 36-year-old mother’s first prenatal visit was at 37 weeks’ gestation, when ultrasonography showed severe oligohydramnios; the fetal kidneys could not be visualized. The findings were discussed with the mother, and she was scheduled to see the maternal-fetal specialist but went into labor before the appointment. The clinician arrives for the delivery and prepares for resuscitation. There was no opportunity to talk to the mother before delivery about the fetus’ condition and plans for care.

Of the following, the BEST next management step after delivery will most likely be

A.	administration of intravenous antibiotics
B.	endotracheal intubation for positive-pressure ventilation
C.	performance of renal ultrasonography
D.	umbilical line placement for intravenous glucose
A

Of the response choices, the best next management step after delivery of this neonate will most likely be endotracheal intubation to provide positive-pressure ventilation. The fetal ultrasonography findings are consistent with renal agenesis. Neonates born with bilateral renal agenesis have significant pulmonary hypoplasia resulting from severe oligohydramnios. Fetal urine contributes significantly to the amniotic fluid volume that is needed for fetal lung development. Immediate resuscitative efforts include positive-pressure ventilation through an endotracheal tube.

PREP Pearls
Fetal urine contributes significantly to amniotic fluid volume.
Amniotic fluid is necessary for fetal lung development.
Fetal renal agenesis results in severe oligohydramnios which leads to pulmonary hypoplasia.

How well did you know this?
1
Not at all
2
3
4
5
Perfectly
29
Q

A 16-year-old adolescent boy is seen for follow-up the morning after an emergency department visit for an asthma exacerbation. The adolescent has known asthma and allergies to grass and ragweed. Although prescribed daily, he takes his controller medication only as needed and has not felt the need to use any recently. Yesterday, he helped a friend mow a large field. About 15 minutes after starting to cut the tall grass he felt chest tightness and began to cough. He used his albuterol inhaler, which brought some relief, and took 2 to 4 puffs several times over the next 4 hours until the job was done. After going home and showering he felt better for a while. About 7 to 8 hours after he started cutting the grass he developed wheezing that was not fully relieved by his albuterol inhaler, prompting a visit to the emergency department. In the emergency department he was treated with oral corticosteroids and bronchodilators. Today, he feels much better, and his physical examination findings are normal.

Of the following, the BEST next step in this adolescent’s management is

A.	an albuterol inhaler as needed and before known allergen exposure
B.	as-needed use of inhaled corticosteroid with an albuterol inhaler
C.	daily and as-needed use of a combined budesonide/formoterol inhaler
D.	daily use of inhaled tiotropium and formoterol
A

The adolescent in the vignette had a biphasic asthma response to allergen exposure, with early bronchospasm amenable to bronchodilator treatment followed hours later by a more intense airway inflammatory reaction responsive only to corticosteroids. This biphasic response is characteristic of IgE-mediated allergic asthma. The best next management step for this adolescent is daily and as-needed use of a combined budesonide/formoterol inhaler. A combined inhaled corticosteroid/long-acting β-agonist (ICS/LABA) inhaler used daily and additionally before allergen exposure prevents both early and late responses in those with IgE-mediated allergic asthma. Formoterol is an LABA appropriate for use in asthma management only when combined with an ICS. It is the only LABA with a short enough onset of action to provide acute relief, making it appropriate for as-needed administration (unlike salmeterol). Under the Same Maintenance And Rescue Therapy (SMART) paradigm from the 2020 National Asthma Education and Prevention Program guidelines, the combination of an ICS with formoterol as the LABA is appropriate for both daily maintenance and as-needed treatment.

PREP Pearls
IgE-mediated allergic asthma is characterized by a biphasic response including early bronchospasm and delayed airway inflammation.
The early response in IgE-mediated allergic asthma is amenable to prevention and treatment with bronchodilators, but corticosteroids are the most effective therapy for both prevention and treatment of the delayed inflammatory response.
Injectable anti-IgE monoclonal antibodies (eg, omalizumab) can prevent both the early and late phases of allergic asthma by binding circulating IgE and preventing its binding to allergens on mast cells.

How well did you know this?
1
Not at all
2
3
4
5
Perfectly
30
Q

A 14-year-old adolescent girl is seen in the office for a health supervision visit. Her menarche was 1 year ago; her cycles are irregular, with her last menstrual period occurring 5 weeks before this visit. After her first cycle, she did not have any bleeding for about 40 days; since then, her cycle intervals have ranged from 25 to 45 days. Her blood flow lasts for 4 to 6 days with the use of 2 to 3 pads per day. The adolescent is otherwise healthy with no history of medical problems. She is concerned about her irregular cycles. Her height is 165 cm (75th percentile) and weight is 50 kg (50th percentile); the percentiles are similar to those documented at her annual visit a year ago. Her physical examination findings are normal.

Of the following, the BEST explanation for this adolescent’s irregular menstrual cycles is

A.	anorexia nervosa
B.	hypothyroidism
C.	immature hypothalamic-pituitary axis
D.	polycystic ovarian syndrome
A

It is common for adolescents to have irregular menstrual cycles, especially in the first 2 years after menarche. This phenomenon is due to immaturity of the hypothalamic-pituitary axis, which leads to anovulatory cycles. The interval between menstrual cycles can range from 21 to 45 days; 5% to 10% of girls have intervals shorter than 20 days or longer than 60 days. Prolonged intervals are most commonly seen between the first and second cycle. Bleeding normally occurs for 3 to 7 days. After the initial 2 years, most adolescents begin to have more regular cycles with intervals of 21 to 34 days, typical of adult women.

How well did you know this?
1
Not at all
2
3
4
5
Perfectly
31
Q

An 8-year-old boy with attention-deficit/hyperactivity disorder, combined type, is brought to the office for medication follow-up. His mother reports that the stimulant medication is working well, but she and his teacher are concerned because the boy’s grades have dropped and his frustration with school has increased since entering the third grade. He takes up to 2 hours to complete his homework. His mother tries to limit distractions by having the television off and sitting with him while he works, but it has not helped. The boy’s teacher allows extra time for him to complete his work, and he takes tests in a quiet office to limit distractions. He has been much less hyperactive and impulsive since starting the medication. The boy’s physical examination findings are unremarkable.

Of the following, the BEST next step in this boy’s management is to

A.	increase the dose of his stimulant medication
B.	recommend a school evaluation for a learning disability
C.	recommend parent behavior-management training
D.	request a 504 plan through the school for additional accommodations
A

The boy in the vignette, with known attention-deficit/hyperactivity disorder (ADHD), is exhibiting signs of a learning disability. When he advanced to a higher grade in school, he began to have significant academic difficulty (poor grades, frustration with school, and prolonged time to complete homework despite behaviorally effective treatment with a stimulant medication). Common comorbidities in children with ADHD include learning disabilities, anxiety, depression, and oppositional defiant disorder. To assess the boy for a learning disability, an evaluation that includes both cognitive function and academic achievement should be performed.

How well did you know this?
1
Not at all
2
3
4
5
Perfectly
32
Q

A 3-month-old male infant born at term is admitted to the hospital with a 2-week history of fevers without a source.

Laboratory data are shown:

Laboratory test

Result

White blood cell count

1,500/µL (1.5 × 109/L)

Hemoglobin

8.0 g/dL (80 g/L)

Platelet count

50 × 103/μL (50 × 109/L)

Alanine aminotransferase

5,500 U/L

Aspartate aminotransferase

4,500 U/L

Serum bilirubin

6 mg/dL (102.6 µmol/L)

Serum triglycerides

500 mg/dL (5.7 mmol/L)

Erythrocyte sedimentation rate

150 mm/h

Serum ferritin

12,000 ng/mL (12,000 µg/L)

Interleukin 2 soluble receptor levels

2,000 pg/mL (reference range, 175.3-858.2 pg/mL)

Of the following, the MOST likely physical examination finding expected in this condition is

A.	hearing loss
B.	snuffles
C.	splenomegaly
D.	vesicular rash
A

The infant in the vignette has prolonged fevers, cytopenia, evidence of inflammation, elevated transaminases, and elevated soluble interleukin 2 (IL-2) receptor levels. This clinical presentation is consistent with hemophagocytic lymphohistiocytosis (HLH). Although seen in all age groups, the highest incidence is in infants younger than 3 months. This condition can be familial or acquired. The most common infectious trigger of acquired HLH is Epstein-Barr virus (EBV), and the most common physical examination finding is splenomegaly (89% of cases).

How well did you know this?
1
Not at all
2
3
4
5
Perfectly
33
Q

6-year-old boy is brought to the office for school concerns. After an evaluation by the school his mother was told that he has an intellectual disability. She was surprised to hear this diagnosis as she thought the testing was “just for his reading.”

Review of the boy’s medical record reveals that he received early intervention services (speech and occupational therapy) beginning at age 24 months to address limited speech, difficulties with independent feeding, and fine motor skills. The boy’s mother decided to “take a break from therapies to let him catch up” after he aged out of early intervention at 3 years of age. During all subsequent health supervision visits, global impairments in speech, fine motor skills, problem solving, and adaptive skills were documented, and the boy’s mother was strongly advised to have him evaluated by the school when he entered prekindergarten.

The boy is not able to dress independently, is unable to tell a story, struggles with verbal instructions involving multiple steps, and prefers to play with younger children. He was toilet trained at age 5 years. The boy’s teacher raised academic concerns including his inability to identify letters or numbers, or write his name. Behavioral concerns at school include inattention, hyperactivity, frequent emotional outbursts, and rare aggressive behaviors.

The boy’s mother reports that her brother did not start talking until he was 5 years old, struggled with learning to read, required special education services, and did not graduate from high school. The boy’s mother does not want to “let the same thing happen” to her son. She plans to have him retained in the first grade because she feels that he is “not trying hard enough” and “only needs help with reading.”

Of the following, the finding that MOST supports the school’s diagnosis is the

A.	family history of speech and learning difficulties
B.	history of inattention, hyperactivity, and externalizing behaviors
C.	receipt of speech therapy at an early age
D.	significant impairments in cognitive and adaptive functioning from an early age
A

The boy in the vignette has an intellectual disability. He has a history of developmental delays from a young age, and these impairments have persisted in all areas, including cognitive and adaptive functioning, thus supporting the diagnosis made with the formal school evaluation. The other response choices are not specifically associated with intellectual disability.

When a child has school performance concerns, information should be obtained regarding attainment of developmental milestones, social history, birth history, past medical history, and family history (family members with similar difficulties or concerns). For the boy in the vignette, it is unclear from the history provided if his uncle had a reading disability or intellectual disability.

How well did you know this?
1
Not at all
2
3
4
5
Perfectly
34
Q

A 4-day-old male infant is brought to the emergency department for evaluation of fast breathing and poor feeding. He was born at term via vaginal delivery and went home from the hospital the next day. His mother’s breast milk came in a couple of days ago, and he had been eating well until 8 hours before presentation. He latches well but seems to be breathing too quickly to suck and swallow. On physical examination, his heart rate is 170 beats/min, respiratory rate is 70 breaths/min, blood pressure is 70/40 mm Hg in the right arm, and oxygen saturation is 65% in room air measured in the right hand. He is tachypneic but not in distress. His lungs are clear to auscultation bilaterally, heart sounds are a normal S1 and a single S2 with a II/VI holosystolic murmur at the left sternal border, abdomen is benign, and capillary refill is brisk. A chest radiograph is obtained (Item Q36). boot shaped heart

Of the following, the BEST next management step for this neonate is to

A.	initiate prostaglandin infusion
B.	intubate for impending respiratory failure
C.	perform echocardiography
D.	perform electrocardiography
A

The best next management step for the neonate in the vignette, who has tachypnea, desaturation, and poor feeding, is to initiate treatment with prostaglandin. His physical examination findings are significant for tachycardia, tachypnea, clear lungs, a single second heart sound, and a holosystolic murmur. His chest radiograph demonstrates a right aortic arch and a boot-shaped heart. These findings are concerning for congenital heart disease with insufficient pulmonary blood flow. Prostaglandin needs to be initiated urgently to open the closing ductus arteriosus to increase pulmonary blood flow and increase the oxygen saturation. Echocardiography and electrocardiography will be required to understand the details of his cardiac diagnosis, but they can be performed after the prostaglandin infusion is started. There is no indication for intubation of this child at this time.

PREP Pearls
Congenital cyanotic heart disease occurs when there is insufficient pulmonary blood flow, intracardiac mixing, or transposition physiology.
Prostaglandin infusion maintains patency of the ductus arteriosus. Higher doses are needed when the ductus has closed.
The side effects of prostaglandin infusion include apnea, fever, tachycardia, vasodilation, and hypotension.

How well did you know this?
1
Not at all
2
3
4
5
Perfectly
35
Q

A 14-year-old adolescent boy is seen in the emergency department with acute flaccid paralysis of the lower extremities, ascending numbness to the mid-trunk, urinary retention, and constipation. His vital signs are normal for age. On physical examination, his mental status and cranial nerve function are normal. He has normal upper extremity strength and reflexes. Strength in both lower extremities is 0/5, reflexes are brisk at the patella and ankle, and toes are upgoing to plantar stimulation. There is a sensory level at the mid-thorax below which he has diminished sensation to light touch and pinprick. The adolescent has normal coordination in his upper extremities. His lower extremity coordination and gait cannot be assessed due to his weakness. The remainder of his physical examination findings are unremarkable.

Magnetic resonance imaging (MRI) of the spinal cord shows an extensive longitudinal T2 hyperintensity extending from T3 to the conus with no enhancement. Brain MRI is normal. Lumbar puncture is performed, and cerebrospinal fluid analysis shows an elevated white blood cell count of 100 cells/μL with a lymphocytic predominance. Antibody tests are ordered.

The adolescent is admitted to the intensive care unit and treated with intravenous methylprednisolone daily for 5 days with minimal improvement. He is subsequently treated with plasma exchange. Rehabilitation services are engaged early in his disease course.

Of the following, the BEST additional supportive measure(s) in the acute management of this adolescent is/are

A.	anticoagulation with therapeutic intravenous heparin infusion
B.	bladder ultrasonography with intermittent catheterization
C.	permissive hypertension with intravenous phenylephrine
D.	prophylactic neuropathic pain management with oral gabapentin
A

The adolescent in the vignette has acute transverse myelitis, a disorder of the spinal cord characterized by acute onset of motor, sensory, and sphincter deficits with a range of underlying etiologies. Urinary retention is a common complication, which increases the risk of urinary tract infection. Therefore, of the response choices, the best additional supportive measures in this adolescent’s acute management are bladder ultrasonography and intermittent catheterization.

PREP Pearls
Acute transverse myelitis is a disorder of the spinal cord characterized by acute onset of motor, sensory, and sphincter deficits with a range of underlying etiologies, most commonly inflammatory or autoimmune. Radicular or back pain may precede the acute onset of neurologic symptoms.
Urgent evaluation of suspected transverse myelitis should include spine magnetic resonance imaging (MRI) to exclude cord compression. Most commonly, imaging shows centrally located hyperintensity spanning multiple contiguous vertebral levels; up to 50% have normal findings on MRI.
In addition to disease-specific treatment of transverse myelitis with high-dose intravenous corticosteroids or plasma-exchange, supportive management includes bowel and bladder care, deep vein thrombosis prophylaxis, decubitus ulcer prevention, and treatment of autonomic instability.

How well did you know this?
1
Not at all
2
3
4
5
Perfectly
36
Q

An 11-year-old girl is being evaluated for an injury to her left ankle that she sustained while playing basketball. On the day before her visit to the clinic, she inverted her ankle when she landed awkwardly from a jump. On physical examination, she has moderate swelling over the lateral aspect of her left ankle. There is tenderness to palpation over the anterior aspect of the distal lateral malleolus and just anterior and inferior to this spot. She has mild laxity with anterior drawer testing. She has an antalgic gait and reports mild pain when walking.

Of the following, the BEST next step in this girl’s management is

A.	cast immobilization
B.	gradual return to basketball as tolerated
C.	limiting weight-bearing with the use of crutches
D.	use of an air stirrup splint
A

The most likely diagnosis for the girl in the vignette is a sprain of the anterior talofibular ligament (ATFL). Item C39A illustrates the position of the ATFL, calcaneofibular ligament (CFL) and posterior talofibular ligament (PTFL) on the lateral aspect of the ankle. She had an ankle inversion injury and has tenderness over the anterior aspect of the distal lateral malleolus, where the ligament inserts, and over the ligament itself.

Following an ankle sprain, recovery is faster with early mobilization of the ankle. A stirrup style ankle brace (Item C39B) permits dorsiflexion and plantarflexion at the ankle, which allows for a more normal gait, while limiting eversion and inversion.

Use of crutches with limited weight-bearing is rarely indicated. Physicians should encourage early mobilization with the use of a less restrictive brace when injured athletes can tolerate it.

PREP Pearls
Early mobilization after an ankle sprain leads to a faster recovery.
The Ottawa ankle rules for obtaining radiographs after an ankle injury have been validated in children.

How well did you know this?
1
Not at all
2
3
4
5
Perfectly
37
Q

A male infant is seen for a routine health supervision visit. His parents have no concerns. He sits unassisted on the examination table. He becomes excited when his mother hands him a favorite toy, reaches up to shake it, and places it in his mouth. While the examiner is talking to his mother, the infant babbles and smiles when his mother looks at him.

Of the following, this infant’s age is MOST likely

A.	3 months
B.	4 months
C.	6 months
D.	9 months
A

The infant in the vignette is exhibiting developmental milestones most consistent with a 6-month-old. He responds to his mother, babbles, sits without support, and brings an object to his mouth.

PREP Pearls
By 6 months of age, infants should be able to sit independently or with their own hands for support.
By 6 months of age, infants should babble with vowel sounds and begin to add consonant sounds.
By 6 months of age, infants should be able to cross midline to reach for objects.

How well did you know this?
1
Not at all
2
3
4
5
Perfectly
38
Q

PerformanceConfidenceAssessment History

Search
All Questions
Question View:
All
41 of 269Print Add Bookmark
ASSESSMENT PROGRESS:Total Questions: 269 Questions Answered: 40 Correct Answers: 26
Question 41
A 6-year-old boy is brought to the emergency department with tea-colored urine for 1 day. He has had no flank pain, burning, or passage of blood clots with urination. He had a sore throat 4 weeks ago. On physical examination, the boy’s heart rate is 110 beats/min, respiratory rate is 18 breaths/min, and blood pressure is 136/86 mm Hg. He has mild puffiness over his eyes; the rest of the physical examination findings are unremarkable.

Results of a urinalysis with microscopy are shown:

Urine

Result

Specific gravity

1.030

Leukocyte esterase

Negative

Nitrite

Negative

Blood

3+

Protein

3+

Red blood cells

> 100/HPF

White blood cells

< 5/HPF

Of the following, the MOST likely diagnosis for this boy is

A.	cystitis
B.	glomerulonephritis
C.	nephrolithiasis
D.	rhabdomyolysis
A

The child in the vignette had a sore throat 4 weeks before presentation with macroscopic hematuria (tea-colored urine without blood clots), hypertension, and edema. Urinalysis with microscopy shows hematuria and proteinuria favoring a diagnosis of acute poststreptococcal glomerulonephritis (PSGN).

PREP Pearls
Brown, tea, or cola-colored urine without blood clots favors a glomerular etiology of macroscopic hematuria.
Bright red, red, or pink urine with blood clots is suggestive of hematuria originating from the urinary collecting system.
A urine that is dipstick test–positive for blood without red blood cells on urine microscopy indicates myoglobinuria (eg, rhabdomyolysis) or hemoglobinuria (eg, intravascular hemolysis).
ABP Content Specifications(s)
Formulate a differential diagnosis of gross hematuria
Plan the appropriate clinical evaluation of gross hematuria
Recognize the disorders associated with hematuria

How well did you know this?
1
Not at all
2
3
4
5
Perfectly
39
Q

A 3-day-old male infant is seen in the emergency department for decreased feeding, vomiting, and lethargy. He has been taking 2 ounces of formula every 2 hours but vomited his last feed and has been difficult to arouse. His vital signs include a blood pressure of 68/40 mm Hg, temperature of 37.3°C, heart rate of 170 beats/min, and respiratory rate of 80 breaths/min. His newborn metabolic screen results are pending.

Laboratory data are shown:

Laboratory Test

Result

Glucose

38 mg/dL (2.1 mmol/L)

Ammonia

35 µg/dL (25 µmol/L)

Carbon dioxide

16 mEq/L (16 mmol/L)

Lactic acid

3 mmol/L

Urine ketones

Positive

Urine reducing substances

Positive

Serum total bilirubin

14 mg/dL (239.4 µg/dL)

Aspartate aminotransferase

90 U/L

Alanine aminotransferase

60 U/L

White blood cell count

8,000/µL (8.0 × 109/L)

C-reactive protein

0.2 mg/dL (2 mg/L)

Of the following, the neonate’s MOST likely diagnosis is

A.	fatty acid oxidation disorder
B.	galactosemia
C.	organic acidemia
D.	urea cycle defect
A

The neonate in the vignette has galactosemia as evidenced by hypoglycemia, hyperbilirubinemia, elevated liver enzymes, and the presence of reducing substances in the urine. The diagnosis of classic galactosemia is made through newborn screening or after the onset of clinical signs and symptoms consistent with the condition. In an untreated neonate, classic galactosemia presents with a life-threatening illness that can involve the liver, brain, and kidney. Neonates are normal at birth; symptoms develop after ingestion of breast milk or standard formula, which leads to accumulation of galactose and its by-products (galactose-1-phosphate, galactitol, and galactonate) in the body.

Urea cycle defects present in the neonatal period with hyperammonemia and respiratory alkalosis. Common urea cycle defects include ornithine transcarbamylase deficiency, citrullinemia, and argininosuccinic aciduria.

How well did you know this?
1
Not at all
2
3
4
5
Perfectly
40
Q

A 5-week-old male infant is seen in the clinic for a routine follow-up visit. He has had worsening emesis over the past 2 weeks that occurs after all feedings and is yellow in color. He has had no fever or respiratory symptoms, but he is progressively becoming more fussy. The infant has lost 110 g since his last visit 2 weeks ago. He is too fussy to obtain an accurate abdominal examination.

Of the following, this infant’s diagnostic evaluation would MOST likely show

A.	abnormal upper gastrointestinal series with small bowel follow through
B.	elevated total body potassium
C.	esophageal erosion
D.	hypochloremic metabolic alkalosis
A

The infant in the vignette has a clinical presentation that is consistent with a diagnosis of hypertrophic pyloric stenosis (HPS). On abdominal examination of an infant with pyloric stenosis, an “olive-sized” epigastric mass is palpable in 60% to 80% of cases, and peristaltic waves may be visible.

PREP Pearls
Abdominal examination of an infant with hypertrophic pyloric stenosis may demonstrate a palpable epigastric “olive” or visible peristaltic waves.
Hypertrophic pyloric stenosis causes a hypochloremic metabolic alkalosis.
Abdominal ultrasonography is the imaging study of choice to diagnose hypertrophic pyloric stenosis.
ABP Content Specifications(s)
Recognize the acid-base changes associated with pyloric stenosis, and manage appropriately
Recognize the clinical features associated with pyloric stenosis, and manage appropriately

How well did you know this?
1
Not at all
2
3
4
5
Perfectly
41
Q

A 2-week-old male infant, born at 35 weeks’ gestation, is seen at the office for a weight check. He is breastfeeding well with normal urination and stooling. He has gained 5 ounces since birth. On physical examination, the neonate is comfortable in room air, with a respiratory rate of 40 breaths/min and a temperature of 37.5°C. There is bilateral watery eye discharge, palpebral and bulbar injection, and mild eyelid swelling. The neonate received eye prophylaxis at birth. The conjunctiva and nasopharynx are swabbed for bacterial culture and a polymerase chain reaction test for gonorrhea and chlamydia.

Of the following, the BEST next step in management of this neonate is

A.	azithromycin eye ointment
B.	azithromycin oral suspension
C.	ceftriaxone, intramuscular
D.	external digital massage of the tear ducts
A

The neonate in the vignette has findings consistent with chlamydial conjunctivitis. Neonatal chlamydial infection is acquired during passage through the infected birth canal. Early in the course, chlamydial conjunctivitis can be watery, and later it becomes mucopurulent. There may be swelling of the eyelids and chemosis (swelling and redness of conjunctiva). Eye prophylaxis with erythromycin ointment given at birth is not effective in preventing chlamydial infection. Oral azithromycin or oral erythromycin is the treatment of choice for both chlamydial conjunctivitis and pneumonia in neonates.

A single dose of intramuscular ceftriaxone, 50 mg/kg, is recommended for treatment of gonococcal ophthalmia, but is not effective for treatment of a chlamydial infection. Although increased tearing and matting of the eyelashes is commonly seen with nasolacrimal duct obstruction, conjunctival erythema, edema, and swelling are not common. The presence of eyelid swelling and chemosis should prompt further investigation and consideration of treatment for neonatal conjunctivitis.

PREP Pearls
Chlamydia, an obligate intracellular bacterium, causes the most common sexually transmitted notifiable infection in the United States.
Neonates exposed to Chlamydia through an infected birth canal develop conjunctivitis typically 5 to 14 days after birth; chlamydial pneumonia occurs between 3 and 12 weeks of age.
The recommended treatment for any form of neonatal chlamydial infection is oral azithromycin or erythromycin

How well did you know this?
1
Not at all
2
3
4
5
Perfectly
42
Q

A 6-week-old infant with no significant birth or past medical history is brought to the emergency department by ambulance for decreased responsiveness. He appears lethargic, pale, and cyanotic with poor respiratory effort. His capillary refill time is greater than 5 seconds. His vital signs include a temperature of 36.5°C, heart rate of 198 beats/min, respiratory rate of 8 breaths/min, blood pressure of 65/42 mm Hg, and oxygen saturation of 86% in room air.

Of the following, the BEST next step in this infant’s management is to

A.	administer a 20 mL/kg normal saline fluid bolus
B.	administer broad-spectrum antibiotics intravenously
C.	begin bag-valve mask ventilation and prepare for intubation
D.	obtain a point-of-care glucose level
A

The infant in the vignette is displaying altered mental status, for which the initial management is the ABCs (Airway, Breathing, Circulation) of resuscitation. Assessment and stabilization of the airway is the first step, followed by stabilization of breathing and circulation. The infant in the vignette is cyanotic with poor respiratory effort and a respiratory rate of 8 breaths/min; thus, bag-valve mask ventilation and preparing for intubation is the best next management step. Although obtaining a point-of-care glucose level and administration of intravenous fluids and broad-spectrum antibiotics will be important steps in the infant’s care, addressing the airway and breathing concerns must take priority.

PREP Pearls
The initial management of altered mental status is the ABCs (Airway, Breathing, Circulation) of resuscitation.
The signs and symptoms of altered mental status in a child can range from subtle to overt and vary depending upon the age of presentation.
The mnemonic AEIOU TIPS is helpful for remembering the broad differential diagnosis for causes of altered mental status.

How well did you know this?
1
Not at all
2
3
4
5
Perfectly
43
Q

A 3-day-old neonate is seen in the office after his newborn screening test showed a thyroxine (T4) level of 1.2 µg/dL (15.5 nmol/L) (reference range, ≥6.5 µg/dL [≥83.7 nmol/L]. The reflex thyroid-stimulating hormone (TSH) level is 2.5 mIU/L (reference range, ≤20 mIU/L ). The neonate was born at term via uncomplicated labor and delivery and had no problems in the newborn nursery. He is exclusively breastfed, and his weight today is 3% below birth weight. The neonate appears vigorous and has mild jaundice. He does not have a goiter or umbilical hernia. The remainder of his physical examination findings are normal.

Of the following, the BEST next step in the management of this neonate is to

A.	begin levothyroxine therapy
B.	obtain thyroid-stimulating hormone and free thyroxine levels
C.	order magnetic resonance imaging of the pituitary gland
D.	repeat the newborn screening test when the neonate is 5 to 7 days old
A

The neonate in the vignette has an abnormal newborn screening test with a low thyroxine (T4) and normal thyroid-stimulating hormone (TSH) level. Based on the neonate’s physical examination findings, the next best step in management is to obtain confirmatory serum TSH and free thyroxine (FT4) levels. The most likely diagnosis for this healthy male neonate is congenital thyroxine-binding globulin (TBG) deficiency, a benign X-linked condition. Confirmatory testing would demonstrate normal TSH and FT4 levels.

Neonates who have newborn screening performed too soon after birth may have elevated TSH levels, given the expected surge of thyroid hormones that occurs after delivery. In such a case, repeating the newborn screening test may be recommended. However, this would not be true for the neonate in the vignette, as his pattern of thyroid function results is not consistent with the newborn screening test being drawn too soon after birth.

PREP Pearls
Neonates with an abnormal thyroid newborn screening test result should be evaluated promptly with a physical examination and confirmatory serum thyroid-stimulating hormone and free thyroxine testing.
A newborn screening test result showing a low thyroxine and normal thyroid-stimulating hormone level is consistent with thyroxine-binding globulin (TBG) deficiency or, less frequently, central hypothyroidism. The confirmatory serum free thyroxine level distinguishes these conditions (normal in TBG deficiency and low in central hypothyroidism).
Thyroxine-binding globulin deficiency is a benign, X-linked condition that does not require treatment.

How well did you know this?
1
Not at all
2
3
4
5
Perfectly
44
Q

A 38-year-old, gravida 7, para 5 woman who is 38 weeks’ pregnant is brought to the labor and delivery unit for severe abdominal pain. She has a history of diabetes and hypertension. She received limited prenatal care. Ultrasonography 1 month ago showed that the placenta was abnormally placed. The woman’s blood pressure is 230/140 mm Hg. The health care team is preparing to perform an urgent cesarean section because of the fetal heart tracing shown in Item Q49. Before delivery, the team is planning for the neonate’s potential management needs beyond standard resuscitation.

Of the following, in the delivery room this neonate is MOST likely to require

A.	blood transfusion
B.	chest radiography
C.	intravenous calcium gluconate
D.	surfactant administration
A

The clinical scenario described in the vignette is suggestive of a placental abruption with fetal blood loss. The neonate in the vignette is most likely to require a blood transfusion of uncrossmatched O-negative blood.

PREP Pearls
Most neonates transition to the extrauterine environment without difficulty. Five percent require some form of resuscitation.
Anticipation and planning are key components of neonatal resuscitation.
Ventilation is the most important step in neonatal resuscitation.

How well did you know this?
1
Not at all
2
3
4
5
Perfectly
45
Q

A 7-year-old boy is seen in the emergency department for evaluation of a 2-day history of abdominal pain, watery diarrhea, malaise, and loss of appetite. Five days ago, he visited a family-owned farm with his siblings. He swam in a chlorinated pool located on the farm and had exposure to livestock, including calves. Other members of the family are also ill with diarrhea. The boy’s temperature is 37.4°C; his vital signs are normal for age. His physical examination findings are unremarkable except for dry oral mucous membranes.

Of the following, the MOST likely cause of this boy’s illness is

A.	Clostridium perfringens
B.	Cryptosporidium parvum
C.	enteropathogenic Escherichia coli
D.	norovirus
A

The boy in the vignette had onset of watery diarrhea 3 days after exposure to treated (ie, chlorinated, filtrated) recreational water. This presentation is most suggestive of Cryptosporidium parvum gastroenteritis. The incubation period of cryptosporidiosis ranges from 3 to 14 days.

Recreational water-associated outbreaks can be caused by a variety of pathogens including Escherichia coli O157:H7, Shigella, norovirus, and Giardia. Sporadic outbreaks of norovirus gastroenteritis are common. However, the incubation period of norovirus infection is short (12 to 24 hours) and characterized by sudden onset of vomiting followed by watery diarrhea.

PREP Pearls
Cryptosporidium is a leading cause of diarrheal outbreaks associated with exposure to chlorinated recreational water (eg, swimming pools, waterslides).
Fecal-oral transmission of Cryptosporidium infection occurs via ingestion of oocysts excreted by infected hosts.
Manifestations of cryptosporidiosis include non-bloody, watery diarrhea with abdominal pain, nausea, vomiting, low-grade fever, anorexia, and weight loss; asymptomatic gastrointestinal infection can occur resulting in failure to thrive.

How well did you know this?
1
Not at all
2
3
4
5
Perfectly
46
Q

An 8-month-old boy is evaluated for a 3-day history of worsening cough, nasal congestion, and low-grade fever. His mother reports that he is irritable, has difficulty breathing, and is eating and drinking less than usual. He has good urine output.

On physical examination, the infant is ill-appearing and fussy but consolable. His temperature is 38.5°C, heart rate is 102 beats/min, respiratory rate is 55 breaths/min, and oxygen saturation is 95% in room air. He has mild intercostal retractions and nasal flaring. Breath sounds are coarse throughout with crackles in the right base extending into the right axilla; no wheezing is heard. The remainder of the physical examination findings are unremarkable.

Of the following, the BEST next step in this infant’s management is to

A.	administer antibiotics
B.	obtain computed tomography scan of the chest
C.	perform rapid intubation and start mechanical ventilation
D.	recommend he return for reevaluation tomorrow
A

The infant in the vignette with fever, tachypnea, increased work of breathing, and focal crackles on physical examination most likely has community-acquired pneumonia (CAP). Although most CAP in children is caused by viruses, this infant’s focal findings and worsening symptoms make a bacterial etiology more likely. Based on the 2011 Infectious Diseases Society of America (IDSA) guidelines for management of CAP, administration of antibiotics is the best next intervention for this infant with likely bacterial pneumonia.

The infant in the vignette appears ill and meets the IDSA guideline criteria for respiratory distress (Item C52A). Admission to the hospital should be considered, and chest radiography would be appropriate; however chest computed tomography is not indicated as a first-line imaging study. Although the infant is in mild distress, he has normal oxygen saturation and does not show signs of imminent respiratory failure; therefore, there is no indication for intubation and mechanical ventilation. However, given his level of distress and the concern for bacterial pneumonia, he does need intervention beyond observation and reevaluation. An important consideration regarding his need for hospital admission is his risk for dehydration, as his mother reports poor oral intake.

PREP Pearls
Normal respiratory rate varies widely across ages; infants show greater variability than older children.
Respiratory rate is a sensitive indicator of illness; abnormal sleeping respiratory rate is the best measure of true dysfunction.
Any child with unexplained tachypnea or hypopnea should undergo careful evaluation for underlying illness.

How well did you know this?
1
Not at all
2
3
4
5
Perfectly
47
Q

A 6-month-old infant, born at 27 weeks’ gestation, with grade 2 vesicoureteral reflux, gastroesophageal reflux, and chronic lung disease is seen in the emergency department for lethargy. His daily medications include fluticasone, furosemide, lansoprazole, and nitrofurantoin. His temperature is 36.8°C, heart rate is 120 beats/min, respiratory rate is 30 breaths/min, blood pressure is 85/65 mm Hg, and oxygen saturation is 99% on his home oxygen support of 0.5 L/min via nasal cannula. The infant’s anterior fontanelle is slightly sunken, and his mucous membranes are moist and pink. On lung auscultation, fine rhonchi are heard throughout without wheezing or rales. There are no retractions or other signs of increased work of breathing. The remainder of his physical examination findings are normal.

Laboratory data are shown:

Laboratory Test

Result

Sodium

130 mEq/L (130 mmol/L)

Potassium

2.8 mEq/L (2.8 mmol/L)

Chloride

89 mEq/L (89 mmol/L)

Carbon dioxide

38 mEq/L (38 mmol/L)

Blood urea nitrogen

10 mg/dL (3.6 mmol/L)

Creatinine

0.3 mg/dL (26.5 µmol/L)

Of the following, the medication MOST likely to have led to this infant’s findings is

A.	fluticasone
B.	furosemide
C.	lansoprazole
D.	nitrofurantoin
A

The infant in the vignette, with chronic lung disease related to prematurity, has signs of mild dehydration (mildly sunken anterior fontanelle), and electrolyte abnormalities including hyponatremia, hypokalemia, hypochloremia, and an elevated bicarbonate level. These findings are consistent with a contraction metabolic alkalosis induced by chronic diuretic therapy. Thus, of the response choices, furosemide (used to control excess fluid in the lungs) is the most likely medication to have contributed to this infant’s condition.

Furosemide is a loop diuretic that inhibits reabsorption of sodium and chloride in the ascending loop of Henle and the proximal and distal renal tubules by interfering with the chloride-binding co-transport system. Both natriuresis (sodium loss) and diuresis (water loss) result. Diuresis decreases extracellular fluid volume and concentrates extracellular serum bicarbonate, contributing to metabolic alkalosis. Historically, this process was known as “contraction alkalosis,” and the primary mechanism was thought to be due to decreased fluid volume and the resulting concentration of serum bicarbonate. However, newer studies have demonstrated that chloride plays a major role in the development of metabolic alkalosis, and some have suggested that “chloride depletion alkalosis” replace the term “contraction alkalosis” to more accurately reflect the underlying mechanism.

PREP Pearls
Metabolic alkalosis is often caused by hypochloremia.
Infants and children on chronic diuretic therapy with loop diuretics (eg, furosemide) are at risk for developing a chloride depletion alkalosis.
Bartter syndrome can cause hypokalemic, hypochloremic, metabolic alkalosis; this diagnosis should be suspected in infants with hyponatremia, hypokalemia, failure to thrive, and dehydration.

How well did you know this?
1
Not at all
2
3
4
5
Perfectly
48
Q

greatest risk for perinatal transmission of hep b

A

Perinatal hepatitis B transmission occurs at the time of labor and delivery. The risk of transmission to the newborn is determined based on the presence of maternal hepatitis B surface antigen (HBsAg) and hepatitis B e antigen (HBeAg). The presence of HBsAg implies either an acute (<6 months) or chronic (≥6 months) infection in the mother. The presence of HBeAg indicates a high level of transmissibility. For a mother positive for both HBsAg and HBeAg, the risk of perinatal transmission is as high as 90%. For a mother that is only HBsAg positive, the risk is around 30%. Other risk factors associated with high transmissibility include a high viral DNA level >2,000 IU/mL, age <25 and <3 doses of maternal hepatitis B vaccine. Detection of hepatitis B surface antibody (>10 mIU/ML) indicates disease immunity, either from immunization or natural infection. Detection of hepatitis B e antibody along with hepatitis B surface antibody indicates past infection.

How well did you know this?
1
Not at all
2
3
4
5
Perfectly
49
Q

A 12-year-old girl is seen in the clinic for evaluation of a 3-month history of abdominal pain and diarrhea. She has 5 to 8 loose, malodorous, floating, greasy-appearing stools daily. She lost 5 kg unintentionally during the 3-month period. Physical examination reveals a weight of 36.3 kg (24th percentile for age), height of 150 cm (44th percentile for age), and a body mass index of 16 kg/m2 (19th percentile for age). She appears pale. Her abdomen is distended but soft and nontender. The remainder of the girl’s physical examination findings are unremarkable.

Laboratory data are shown:

Laboratory Test

Result

Stool

pH

7.0

Occult blood

Negative

Reducing substances

Negative

Ova and parasite

Negative

Bacterial pathogen testing

Negative

Blood

Hemoglobin

10.1 g/dL (101 g/L)

Erythrocyte sedimentation rate

2 mm/h

25-Hydroxyvitamin D

9 ng/mL (22.46 nmol/L)

Albumin

4.3 g/dL (43 g/L)

Of the following, the BEST treatment for this girl is

A.	albumin 25% infusion
B.	gluten-free diet
C.	lactase enzyme replacement therapy
D.	low-sucrose diet
A

The girl in the vignette has evidence of increased intestinal fat losses (steatorrhea and fat-soluble vitamin D deficiency), resulting from fat maldigestion and/or fat malabsorption. Of the response choices, a gluten-free diet is the only treatment for a fat malabsorption disorder.

On the basis of the girl’s laboratory and physical examination findings, a diagnosis of celiac disease was suspected. Further testing revealed an elevated tissue transglutaminase IgA antibody level, and findings on endoscopy with biopsy confirmed the diagnosis. A gluten-free diet is the treatment for celiac disease. Albumin infusions are used to treat symptomatic hypoalbuminemia caused by severe protein-losing enteropathy. The albumin level of the girl in the vignette is within the reference range, and an albumin infusion would not address her fat malabsorption and steatorrhea. Lactase enzyme replacement therapy can be used to treat diarrhea that results from malabsorption of lactose, and a low-sugar diet may resolve diarrhea caused by malabsorption of sucrose, but neither treatment would decrease this girl’s fat malabsorption and steatorrhea.
PREP Pearls
Steatorrhea (Increased fecal fat) occurs when ingested fat is maldigested and/or malabsorbed.
Complications of fat maldigestion or malabsorption include diarrhea, abdominal pain, diaper dermatitis in infants and diapered children, weight loss or poor weight gain, and fat-soluble vitamin deficiencies.
Treatment of steatorrhea includes identifying and managing the underlying cause while supporting nutrition and growth through adequate caloric intake and fat-soluble vitamin supplementation.

How well did you know this?
1
Not at all
2
3
4
5
Perfectly
50
Q

A 4-year-old boy with trisomy 21 is seen in the emergency department with a femur fracture. His mother brought him to the emergency department because he would not walk when he awoke from his nap. There is no reported history of trauma. The boy’s past medical history is significant for an atrial septal defect that was repaired 9 months ago. He has an individualized education program and receives speech therapy through the school district twice weekly. He is incontinent of urine and stool. The boy lives with his mother and 2 older developmentally typical siblings. His mother states that she is unemployed because she must care for the boy during the day. The boy’s grandparents sometimes help with childcare. The family receives public assistance and has Medicaid insurance. Further evaluation in the emergency department reveals multiple healed rib fractures.

Of the following, the MOST significant risk factor for this boy’s injuries is

A.	family need for public assistance
B.	male sex
C.	single-parent household
D.	trisomy 21
A

The boy in the vignette has injuries consistent with non-accidental trauma (NAT) or physical abuse (unexplained femur fracture, multiple healed rib fractures). Having trisomy 21 is his most significant risk factor for experiencing physical abuse. Children with developmental disabilities and chronic medical conditions are at increased risk for abuse. One study (Paul) found that children with disabilities are nearly 3 times more likely to be physically abused than children without a disability. Children with mild cognitive disabilities and without motor disabilities were found to be at the highest risk.

How well did you know this?
1
Not at all
2
3
4
5
Perfectly
51
Q

A 12-month-old, full-term male infant is being evaluated for gross motor delay. His mother first became concerned when he was 7 months of age because he was not yet rolling over. At that time laboratory testing demonstrated a serum creatine kinase level of 50 U/L and a thyroid-stimulating hormone level of 1.1 mIU/L. Newborn screening results were re-verified as normal. The infant was enrolled in physical therapy. He rolled (supine to prone and prone to supine) at age 9 months and was able to sit with support at 10 months of age. At age 12 months, he cannot sit independently. He reaches for toys with both hands, transfers objects, and is beginning to use 2 fingers in a pincer grasp to pick up cereal. He smiles, laughs, and babbles with multiple consonants. He enjoys playing peek-a-boo and with toys that he causes to light up and play music. There has been no developmental regression.

On physical examination, the infant’s vital signs are normal. His head circumference is 46 cm (41st percentile for age). On neurological examination, he is alert and playful with age-appropriate stranger anxiety. His cranial nerve findings are normal. He has truncal hypotonia with head lag when pulled to sit, slippage at the shoulders on vertical suspension, and mild appendicular hypotonia in all 4 extremities. His strength appears normal and symmetric. When placed in a supported sit, he has slight titubation of the head. Deep tendon reflexes are 2+ in the biceps, brachioradialis, and triceps, 3 at the patella bilaterally, and 2+ at the achilles bilaterally. His toes are downgoing to plantar stimulation. He reaches for toys without dysmetria/ataxia. The remainder of the infant’s physical examination findings are normal.

Of the following, the BEST next step in this infant’s evaluation is

A.	brain magnetic resonance imaging
B.	electromyography
C.	nerve conduction studies
D.	survival motor neuron 1 gene testing
A

The infant in the vignette has an isolated gross motor delay in the setting of predominantly axial hypotonia with preserved reflexes and strength. These examination findings are suggestive of a central etiology for his hypotonia. Magnetic resonance imaging (MRI) of the brain would be an appropriate next step in his diagnostic evaluation to evaluate for cerebral malformations or neuroimaging markers of specific genetic and/or metabolic disorders. The other response choices would be more appropriate when testing for an infant with findings suggestive of hypotonia secondary to a peripheral etiology. In this age group, however, central etiologies are more common than peripheral.

The initial diagnostic testing for infants with hypotonia suspected to have a peripheral etiology is focused on time-sensitive and treatable conditions. Evaluation usually includes a serum creatinine kinase (CK) level, SMN1 gene deletion testing, and thyroid studies to evaluate for an underlying myopathy, spinal muscular atrophy, or thyroid disease. If the results of initial laboratory testing are negative, electromyography and nerve conduction studies should be performed for more precise localization within the peripheral nervous system prior to proceeding to targeted disease tests or muscle biopsy.

PREP Pearls
The signs and symptoms accompanying hypotonia provide important clues for accurate localization, which can encompass any part of the nervous system.
In infants, central etiologies of hypotonia are more common than peripheral.
Tone, the resistance of the muscle to passive range of motion/stretch, is distinct from strength, the amount of force a muscle group can generate. The presence or absence of weakness can be a helpful localization indicator.

How well did you know this?
1
Not at all
2
3
4
5
Perfectly
52
Q

A 14-year-old adolescent boy is seen for a preparticipation physical examination prior to his high school soccer season. The boy’s medical history is significant for a seizure 2 years ago that occurred during a middle school basketball game. His family history is remarkable for a sibling who died of sudden infant death syndrome and his mother and maternal uncle who have type 1 diabetes mellitus. The adolescent’s physical examination findings are normal.

Of the following, the BEST next management step for this boy is

A.	clearance for soccer participation
B.	referral to a cardiologist
C.	referral to a endocrinologist
D.	referral to a neurologist
A

The boy in the vignette had an apparent seizure while exercising. Myoclonic activity associated with an arrhythmia can be mistaken for seizure activity. Seizures can also be associated with dysrhythmia events and can be the initial presenting sign of an underlying channelopathy, such as long QT syndrome. The boy had a sibling who died of sudden infant death syndrome, which can also be the sentinel event for a child with a congenital arrhythmia syndrome. The boy’s personal and family history are very concerning for a genetic heart condition, and he should be evaluated by cardiology prior to clearing him for sports participation.

PREP Pearls
A history of unexplained seizure activity could signify arrhythmogenic heart disease.
A child or adolescent with a well-controlled seizure disorder should be cleared for participation in most sports without qualification.

How well did you know this?
1
Not at all
2
3
4
5
Perfectly
53
Q

A 3-month-old infant is seen for a weight check. Her weight today is 5 kg, which is the same as her weight at an appointment 2 weeks ago. She is scheduled for surgical repair of a large ventricular septal defect at the end of the month and is on a high-dose diuretic regimen. She has persistent comfortable tachypnea and is otherwise asymptomatic. Her diet consists of expressed breast milk by bottle. Attempts at increasing the volume of feedings have failed secondary to disinterest from the infant.

Of the following, the BEST next dietary intervention for this infant is to

A.	add electrolyte solution to aid in hydration
B.	change to a 30-kcal/oz formula
C.	fortify the breast milk
D.	limit feeding time to 20 minutes
A

The infant in the vignette has a large ventricular septal defect with increased pulmonary blood flow, leading to pulmonary edema and tachypnea. Her symptoms are managed with high-dose diuretics to decrease the pulmonary edema. Ultimately, she needs to undergo surgical repair of this lesion. Until then, the pediatrician and pediatric cardiologist can improve her surgical candidacy by maintaining appropriate weight gain. Fortifying the expressed breast milk is the best option to limit fluid intake while maximizing calories. Neither adding an electrolyte solution for hydration nor limiting feeding time to minimize energy use will provide the needed additional calories. Changing the infant’s diet to a 30-kcal/oz formula may result in diarrhea and excessive stool losses.

PREP Pearls
Children with heart disease have increased caloric needs and often require limitation of fluid intake.
Increased caloric density formula or fortified breast milk is often needed for appropriate weight gain in infants with heart disease.
A multidisciplinary approach is needed to optimize nutrition and growth for many children with heart disease.

How well did you know this?
1
Not at all
2
3
4
5
Perfectly
54
Q

A 15-year-old adolescent boy is evaluated in the office for right flank pain and bright red urine with blood clots for 1 day. His pain is intermittent and colicky, and is radiating to the right lower abdomen. He has no dysuria, urgency, or fever. The boy’s heart rate is 120 beats/min, respiratory rate is 16 breaths/min, and blood pressure is 110/70 mm Hg. His weight is greater than the 95th percentile, and his height is at the 50th percentile for age. He has right costovertebral angle tenderness. The remainder of his physical examination findings are unremarkable.

A urinalysis with microscopy is shown:

Laboratory Test

Result

Specific gravity

1.030

Leukocyte esterase

Positive

Nitrite

Negative

Blood

3+

Protein

Negative

Red blood cells

> 100/HPF

White blood cells

10-20/HPF

Of the following, the BEST next step in confirming this adolescent’s suspected diagnosis is

A.	abdominal computed tomography with contrast
B.	abdominal magnetic resonance imaging without contrast
C.	plain abdominal radiography
D.	renal and bladder ultrasonography
A

The adolescent boy in the vignette has symptoms consistent with renal colic, macroscopic hematuria with blood clots, and right costovertebral tenderness, favoring a diagnosis of nephrolithiasis. Renal ultrasonography is the first-line imaging modality recommended to detect nephrolithiasis. Advantages of ultrasonography over the other options listed include the avoidance of exposure to ionizing radiation and the potential to detect obstruction in the urinary system. A noncontrast spiral computed tomography (CT) scan is the most sensitive test to diagnose nephrolithiasis, especially in symptomatic patients, however, the significant radiation exposure makes this a less optimal choice. A CT scan with contrast is not commonly recommended as the contrast may decrease the sensitivity for stone detection and extend the time required to perform the test. Renal calculi are often not detected on magnetic resonance imaging. Abdominal radiography has a lower sensitivity and specificity than does ultrasonography and detects only radio-opaque renal stones.

PREP Pearls
Factors that increase the risk for nephrolithiasis include decreased urine volume, increased solute excretion (calcium, uric acid), and decreased levels of stone inhibitors (citrate, magnesium).
Renal and bladder ultrasonography is the first-line imaging modality recommended to diagnose nephrolithiasis in children.
A noncontrast spiral computed tomography scan is the most sensitive test to diagnose nephrolithiasis in symptomatic children.

How well did you know this?
1
Not at all
2
3
4
5
Perfectly
55
Q

A 12-year-old girl is seen for a health supervision visit. Developmental milestones were delayed for speech, and she received speech therapy from age 2 to 4 years. She is in a 7th-grade, regular education classroom with an individualized education plan for reading and comprehension. Menarche was achieved at age 11 years. Physical examination findings are unremarkable except that she is a tall girl. Family history is noncontributory.

Of the following, the MOST likely diagnosis for this girl is

A.	Klinefelter syndrome
B.	Marfan syndrome
C.	trisomy X
D.	Turner syndrome
A

Of the response choices, the most likely diagnosis for the girl in the vignette is trisomy X. Trisomy X or triple X (47,XXX) is a sex chromosome aneuploidy with an incidence of approximately 1 in 1,000 liveborn females. Clinical features are highly variable without clearly defined malformations.

Common reported features include:

Tall stature
Developmental delay: Primarily speech delay; intellectual disability in the mild-to-moderate range (average IQ of 55); learning disabilities in verbal and expressive language
Behavior: Conduct disorder, depression, anxiety, autism spectrum disorder, mood disorder

PREP Pearls
Trisomy X or triple X (47,XXX) is a sex chromosome aneuploidy with highly variable clinical features. The most common features include tall stature, speech delay, and behavioral issues.
Triple X syndrome arises when a nondisjunction event results in an extra chromosome X in the embryo; approximately 60% occur during maternal meiosis I and are associated with advanced maternal age.

How well did you know this?
1
Not at all
2
3
4
5
Perfectly
56
Q

A previously healthy, 8-year-old, fully immunized boy is seen in the office for malaise, headache, a 2-day history of fever, and a 10-day history of worsening, nonproductive cough. One week ago he was seen in the emergency department and diagnosed with a viral illness. The boy has no sick contacts, no pets at home, and has not traveled recently.

The boy appears non-toxic. His temperature is 38.2°C, heart rate is 102 beats/min, respiratory rate is 38 breaths/min, and oxygen saturation is 92% in room air. His blood pressure is normal. He has pharyngeal erythema and crackles auscultated in both lung fields. The remainder of his physical examination findings are unremarkable.

Of the following, the MOST likely pathogen causing this boy’s illness is

A.	Legionella pneumophila
B.	Mycoplasma pneumoniae
C.	Staphylococcus aureus
D.	Streptococcus pneumoniae
A

The clinical presentation of the well-appearing boy in the vignette with a gradual onset of illness characterized by prolonged cough, low-grade fever, malaise, pharyngitis, and bilateral crackles on lung examination is suggestive of atypical pneumonia caused by Mycoplasma pneumoniae.

It can be difficult to reliably distinguish bacterial etiologies of community-acquired pneumonia (CAP) from viral etiologies based on clinical findings alone. However, CAP caused by Staphylococcus aureus and Streptococcus pneumoniae is associated with high fever, chills, marked tachypnea, and focal auscultatory findings, which is not consistent with the clinical findings for the boy in the vignette. Atypical infection due to Legionella pneumophila is rare in children, accounting for ≤1% of all pneumonia cases, making this diagnosis unlikely for the boy in the vignette.

PREP Pearls
Mycoplasma pneumoniae is a frequent cause of upper respiratory tract infection and community-acquired pneumonia in school-aged children, adolescents, and young adults.
In ambulatory settings, routine administration of antimicrobial therapy for school-aged children for respiratory tract infection due to suspected Mycoplasma is not indicated.
In children with sickle cell disease, trisomy 21, immunodeficiency disorders, and chronic cardiac or pulmonary disease, Mycoplasma pneumoniae can cause severe, complicated pneumonia.
In hospitalized children with moderate to severe Mycoplasma pneumoniae pneumonia or other severe manifestations, antimicrobial therapy with a macrolide (eg, azithromycin, clarithromycin) may be indicated.

How well did you know this?
1
Not at all
2
3
4
5
Perfectly
57
Q

A 12-month-old girl is seen for a health supervision visit. She has been thriving, and her parents do not have any concerns. On physical examination, there is a white pupillary reflex in the right eye. The remainder of her physical examination findings are normal.

Of the following, the BEST next step in this child’s management would be referral to a

A.	geneticist
    B.	pediatric oncologist
C.	pediatric ophthalmologist
D.	radiation oncologist
A

The child in the vignette has a white pupillary reflex, known as leukocoria (Item C65A). Leukocoria is a concerning finding, suggestive of retinoblastoma (RB; an abnormal growth of retinoblasts), and must be confirmed by an ophthalmologist.

The geneticist, radiation oncologist, and pediatric oncologist play a role in caring for a child with RB. However, the specialist who must evaluate the child first and make the diagnosis of RB is the pediatric ophthalmologist.

PREP Pearls
Leukocoria is the presenting finding in 50% to 60% of cases of retinoblastoma, and new-onset strabismus may be the presenting finding of retinoblastoma.
A pediatric ophthalmologist must promptly and thoroughly examine a child with leukocoria to evaluate for retinoblastoma.
All children should have screening for a bilateral red reflex at every health supervision visit. The American Academy of Pediatrics guidelines recommend screening from the newborn to 6 months of age, 6 to 12 months of age, 1 to 3 years of age, 4 to 5 years of age, and 6 years and older visits.

How well did you know this?
1
Not at all
2
3
4
5
Perfectly
58
Q

A 16-year-old adolescent boy is brought to the emergency department by ambulance after he was involved in a motor vehicle collision. He was the unrestrained driver of a vehicle that hit a tree traveling approximately 40 mph. He complains of nasal and right-sided facial pain. He did not lose consciousness during the accident and was able to self-extricate from the vehicle. He denies neck or back pain. Physical examination reveals swelling and ecchymosis of his right cheek and nasal bridge, and a small amount of blood in his nares. His pupils are equal and reactive to light, and his extraocular movements are intact bilaterally. His dentition is intact, and there is no bleeding from his mouth. Head computed tomography scan does not demonstrate any skull fractures or intracranial hemorrhage. Computed tomography scan of his face shows displaced, comminuted fractures of his right zygomatic arch and ethmoid bones.

Of the following, the BEST next step in this boy’s management is

A.	intravenous ampicillin/sulbactam
B.	magnetic resonance imaging with contrast
C.	oral analgesia, cold compresses, and nasal decongestant
D.	urgent evaluation by an otolaryngologist
A

The boy in the vignette has displaced, comminuted facial fractures and requires urgent evaluation by a pediatric otolaryngologist.

PREP Pearls
Comminuted or displaced facial fractures require urgent evaluation by an otolaryngologist.
Maxillofacial computed tomography (facial bones including orbits, sinuses, mandible, teeth) is the gold standard imaging modality for facial trauma.
For isolated, non-displaced, or minimally displaced facial fractures, conservative management with cold compresses, oral analgesia, and nasal decongestants is appropriate.

How well did you know this?
1
Not at all
2
3
4
5
Perfectly
59
Q

A 2-day-old, 38 weeks’ gestation male neonate is seen in the emergency department. This morning, while changing his diaper, his mother noticed blood mixed in the stool. The infant has been fussy for a few hours. He was delivered vaginally during a planned home delivery attended by a neighbor who has experience in performing home births but is not a qualified midwife. The neonate did not receive any medications at the time of or after delivery. He is exclusively breastfed and has been feeding well. The infant’s heart rate is 156 beats/min and blood pressure is 88/35 mm Hg (mean = 43 mm Hg). The infant is awake and fussy. He appears pink and well perfused, with scattered petechiae over the abdomen and blood oozing from the umbilical cord. His abdomen is soft and nontender. Laboratory tests are ordered, and a peripheral intravenous line is placed.

Of the following, the BEST next step in this neonate’s management is to

A.	administer ampicillin and gentamicin, intravenously
B.	administer vitamin K, intravenously
C.	infuse fresh frozen plasma
D.	infuse platelets
A

The neonate in the vignette most likely has hemorrhagic disease of the newborn, and it is evident from the perinatal history that he did not receive 1 mg intramuscular vitamin K after delivery. To prevent further bleeding, it is important to give this patient vitamin K as soon as possible. If no intravenous line is available, intramuscular injection is an appropriate alternative. Coagulation factor replacement and blood product administration, if needed, can occur later.

PREP Pearls
Hemorrhagic disease of the newborn due to vitamin K deficiency can be easily prevented with intramuscular injection of 1 mg of vitamin K after birth.
Vitamin K deficiency should be strongly considered for a newborn presenting with any form of bleeding, especially those exclusively breastfeeding, those delivered at home, and those who did not receive vitamin K prophylaxis after birth.

How well did you know this?
1
Not at all
2
3
4
5
Perfectly
60
Q

A 13-year-old adolescent boy with newly diagnosed type 1 diabetes is seen for follow-up. He is doing well with his insulin regimen of multiple daily subcutaneous injections. He takes 12 units of long-acting basal insulin analog once daily at bedtime and boluses of a rapid-acting insulin analog before meals, dosed based on carbohydrate counts (1 unit for every 15 g of carbohydrate) and blood glucose measurements (1 unit for every 50 mg/dL over 150 mg/dL). The boy’s physical examination findings are unremarkable.

His blood glucose log is shown (glucose levels are in mg/dL):

Breakfast

Lunch

Dinner

Bedtime

Day 1

253

118

142

117

Day 2

322

165

133

154

Day 3

196

122

102

125

Day 4

217

131

95

111

Day 5

284

110

118

132

Of the following, the BEST next step in this boy’s management is to

A.	ask the boy to check his blood glucose at 3 AM
B.	ask the boy to check for urine ketones each morning
C.	increase the basal insulin dose to 13 units
D.	split the basal insulin dose into 6 units twice daily
A

The boy in the vignette has newly diagnosed type 1 diabetes, and his blood glucose log demonstrates that he is experiencing elevated blood glucoses in the morning. This pattern is most commonly due to too little (dawn effect) or too much (Somogyi effect) bedtime basal insulin. A blood glucose level checked in the middle of the night (usually between 2-3 AM) can help distinguish between these two effects. An elevated overnight glucose level indicates too little basal insulin, and a low level indicates too much basal insulin.

PREP Pearls
In type 1 diabetes, elevated morning blood glucose levels may be due to either too little (dawn effect) or too much basal insulin (Somogyi effect).
Checking a blood glucose level in the middle of the night (2-3 AM) can help distinguish between the dawn and Somogyi effects.
Understanding the pharmacodynamics of insulin analogs is essential for making effective insulin dose adjustments

How well did you know this?
1
Not at all
2
3
4
5
Perfectly
61
Q

A 6-year-old, previously healthy boy is brought to the urgent care center with a 2-week history of cough and low-grade fever. On physical examination, his temperature is 38°C, his oxygen saturation is 94% in room air, and he has crackles over his left lung base. Chest radiography shows a left lower lobe consolidation and a radiolucent lesion with a thin cortical rim with a fallen leaf fragment in the metaphyseal region of the right proximal humerus (Item Q70). The boy has no history of arm pain. Right upper extremity physical examination findings are normal.

Of the following, the BEST next step in management of this boy’s incidental finding is

A.	bone biopsy
B.	bone scan
C.	magnetic resonance imaging
D.	observation
A

The boy in the vignette has an incidental radiographic finding of a simple bone cyst, also known as a solitary bone cyst or unicameral bone cyst (UBC). This is a benign pediatric bone tumor representing 3% of primary bone tumor biopsies. Unicameral bone cysts occur most often between ages 4 and 10 years; the etiology remains unknown. Most UBCs are located in the metaphysis of the proximal humerus or femur but can occur in other appendicular or axial bones. They are generally painless and discovered incidentally but can be mildly painful, reflecting a microscopic fracture. Trauma to the area may result in an abrupt pathologic fracture presenting with pain, edema, erythema, warmth, decreased range of motion, and deformity.

PREP Pearls
The diagnosis of a unicameral bone cyst can be made using radiography; typical features include location in the metaphysis that extends to, but does not cross, the physis; radiolucency with a thin cortical rim; and the pathognomonic sign of the “fallen leaf” fragment representing a portion of fractured cortical bone settling to the dependent part of the fluid-filled cyst.
The differential diagnosis of a unicameral bone cyst includes an aneurysmal bone cyst, fibrous dysplasia, and enchondroma.
Observation is reasonable for asymptomatic and small unicameral bone cysts. Indications for surgical intervention include persistent pain, recurrent fracture, and prevention of deformity.

How well did you know this?
1
Not at all
2
3
4
5
Perfectly
62
Q

A 14-year-old adolescent girl is seen in the office for follow up of dysmenorrhea. Based on the results of laboratory evaluation performed at her initial visit for this condition 9 months ago, she was diagnosed with iron-deficiency anemia and type 2 diabetes (hemoglobin A1C, 6.6%). Today, she reports ongoing heavy and painful menstrual cycles as well as increasing fatigue; she is otherwise well. She has never had sexual intercourse. The girl’s vital signs are normal. Her body mass index has increased over the past 3 months from 98% to 99% for age and sex. Her current prescribed medications are ferrous sulfate 325 mg every other day, naproxen sodium 440 mg twice daily at the time of her menses, and metformin 1,000 mg twice daily.

Laboratory data are shown:

Laboratory Test

Result

Hemoglobin

9.3 g/dL (93 g/L)

Mean corpuscular volume

78 fL

Mean corpuscular hemoglobin concentration

34.2 g/dL (342 g/L)

Ferritin

2 ng/mL (2 µg/L)

Hemoglobin A1C

6.7%

Of the following, the BEST next step in this girl’s management is

A.	assess adherence by asking open-ended questions
B.	decrease the dose of metformin for 2 weeks
C.	increase her dose of ferrous sulfate to 325 mg daily
D.	refer her to a hematologist for possible iron infusion therapy
A

The adolescent in the vignette has recently been diagnosed with chronic medical conditions that require daily medications for long-term management. Medication adherence is difficult for adolescents, especially if parents are not monitoring medications closely to increase accountability. The best next step in the management of the girl in the vignette is to assess medication adherence by asking her open-ended questions. For example, “Are you in charge of your own medications?” “Do you ever accidentally forget to take them?” “Do any of them cause you problems?” The physician should not increase or decrease the dosage of her medications until it is assessed if the patient is taking their medications correctly. While referral to hematology for iron infusion therapy may be needed at some point, this step should not occur before open-ended inquiry regarding her medication adherence.

PREP Pearls
Adherence to medications and medical treatment plans can be challenging for adolescents due to various factors including peer factors, striving for “normalcy,” forgetting the medication, physical well-being (eg, side effect concerns), and parent conflict.
Physicians can assess medication adherence with open-ended, nonjudgmental questions.
Physicians should participate as team members with parents and adolescents to problem solve how to improve adherence.

How well did you know this?
1
Not at all
2
3
4
5
Perfectly
63
Q

A 16-month-old boy is seen for a routine health supervision visit. He has been well since his last visit 4 months ago and has not had any recent illness. The boy was breastfed until 12 months of age, when he transitioned to cow milk. He started eating solid foods at 6 months of age, and occasionally eats crackers, but mainly consumes milk (about 48 oz/day). He also drinks juice. The boy’s activity level, stools, and urine output are normal. His heart rate is 100 beats/min. He looks well nourished. His lips, tongue, and conjunctiva are pale. There is no scleral icterus, and his mucous membranes are moist. He has a 2/6 systolic ejection murmur at the left sternal border. The remainder of his physical examination findings are normal.

Laboratory data are shown:

Laboratory Test

Result

White blood cell count

6,800/µL (6.8 × 109/L)

Hemoglobin

5.5 g/dL (55 g/L)

Platelet count

480 × 103/µL (480 × 109/L)

Mean corpuscular volume

65 fL

Red cell distribution width

18%

Neutrophils

41.2%

Lymphocytes

54.3%

Monocytes

4.2%

Eosinophils

0.3%

Reticulocyte

1%

Absolute reticulocyte count

44 × 103/µL (44 × 109/L)

Iron

20 µg/dL (3.6 µmol/L) (reference range, 45-165 µg/dL [8.1-30 µmol/L])

Total iron-binding capacity

530 µg/dL (95 µmol/L) (reference range, 220-430 µg/dL [39.4-77 µmol/L)

Unsaturated iron-binding capacity

507 µg/dL (90.8 µmol/L) (reference range, 110-370 µg/dL [19.7-66.2 µmol/L])

Iron saturation

3.7% (reference range, 14-50%)

Ferritin

<5 ng/mL (<5 µg/L) (reference range, 30-400 ng/mL [30-400 µg/L])

Of the following, the BEST next best step in this boy’s management is to

A.	give a 20-mL/kg normal saline bolus, intravenously
B.	give a 20-mL/kg transfusion of packed red blood cells
C.	give a dose of iron, intravenously
D.	recommend dietary changes and close follow-up
A

The boy in the vignette has severe iron-deficiency anemia (hemoglobin level of 5.5 g/dL [55 g/L]) due to inadequate dietary intake of iron. The best next management step is to give a dose of iron intravenously. Parenteral iron is indicated for children who require a rapid and/or short treatment course, such as the boy in the vignette, and for children who do not tolerate oral iron supplementation or have ongoing blood loss or gastrointestinal disease.

The boy in the vignette is hemodynamically stable so he does not require an immediate increase in his hemoglobin level. A transfusion of packed red blood cells would be indicated to rapidly increase the hemoglobin level if the boy had symptoms such as severe fatigue, syncope or near syncope, or significant tachycardia. An intravenous normal saline bolus would be indicated if the boy was hemodynamically unstable. Normal saline boluses should be given with caution in the setting of anemia because the dilutional effect causes worsening of the anemia and decreased oxygen-carrying capacity, and could lead to heart failure. Although dietary changes should be recommended for the boy, his severe anemia necessitates more rapid iron replacement.

Oral iron supplementation would also be appropriate for the boy in the vignette. The ferrous form of iron, at a minimum dose of 3 mg/kg/day, is sufficient to replenish iron stores over several months (3 or more). Very high doses of oral iron (eg, 6 mg/kg/day) can cause bowel irritation and constipation, which may lead to decreased adherence.

PREP Pearls
Parenteral iron is indicated for children with iron deficiency who are hemodynamically stable and require a rapid and/or short treatment course, do not tolerate oral iron supplementation, or have ongoing blood loss or gastrointestinal disease.
A packed red blood cell transfusion is indicated for the treatment of symptomatic anemia (eg, severe fatigue, syncope or near syncope, or significant tachycardia).
An increase in the reticulocyte percentage is an early indicator of recovery during the treatment of iron-deficiency anemia.

How well did you know this?
1
Not at all
2
3
4
5
Perfectly
64
Q

A 4-year-old boy is seen in the office with fever, rash, and decreased oral intake. He has had fevers for 5 days, and a new skin rash was noted this morning. He has had no nasal congestion, cough, vomiting, or diarrhea. His temperature is 38.5°C, heart rate is 130 beats/min, respiratory rate is 30 breaths/min, blood pressure is 85/55 mm Hg, and oxygen saturation is 99% in room air. He is irritable but consolable. His conjunctivae are injected without drainage bilaterally, lips and tongue are erythematous without blisters or exudate, and he has cervical lymphadenopathy on the right. A diffuse maculopapular erythematous rash is noted on his trunk and extremities as well as erythema of his hands and feet. The remainder of his physical examination findings are unremarkable.

Of the following, the BEST next management step for this boy is to

A.	administer acetylsalicylic acid orally at 10 mg/kg per day
B.	administer intravenous immunoglobulin at 2 g/kg
C.	obtain a complete echocardiogram
D.	obtain laboratory data including C-reactive protein level
A

The boy in the vignette has a history and physical examination findings that meet criteria for classic Kawasaki disease (KD). The best next step for him is treatment with 2 mg/kg of intravenous immunoglobulin to reduce the risk of coronary artery aneurysm formation. Although children with KD are treated with acetylsalicylic acid at the time of diagnosis, a dose of 10 mg/kg per day is not adequate. Echocardiography and laboratory evaluation are important to the complete evaluation of KD, but they are not as urgent as the administration of immunoglobulin.

PREP Pearls
Classic Kawasaki disease is diagnosed when a child has fever for 5 or more days and at least 4 of the following 5 clinical findings:
Erythema and cracking of the lips, strawberry tongue, and/or erythema of oral and pharyngeal mucosa
Bilateral bulbar conjunctival injection without exudate
Maculopapular diffuse erythematous or erythema multiforme-like rash
Erythema and edema of the hands and feet in acute phase and/or periungual desquamation in subacute phase
Cervical lymphadenopathy (≥1.5-cm diameter), usually unilateral
Children with a prolonged fever with fewer than 4 of the classic features may have atypical or incomplete Kawasaki disease; this diagnosis may be supported by laboratory and echocardiographic findings.
Treatment of children with Kawasaki disease with 2 mg/kg of intravenous immunoglobulin is required to reduce the incidence of coronary artery aneurysms; acetylsalicylic acid is used to treat inflammation and block platelet function.

How well did you know this?
1
Not at all
2
3
4
5
Perfectly
65
Q

A 14-year-old adolescent boy is admitted to the hospital for management of anemia, hematochezia, and hypovolemia. He has a 6-month history of recurrent hematochezia and diffuse abdominal pain with worsening symptoms over the past 3 weeks, now with 5 to 6 episodes of bloody diarrhea daily. He reports bilateral knee pain, a 5-lb weight loss, fevers, and mouth sores over the past 2 weeks. Physical examination reveals a pale and tired-appearing adolescent. His heart rate is 120 beats/min, blood pressure is 130/70 mm Hg, weight is 57 kg (60th percentile for age), height is 160 cm (18th percentile for age), and body mass index is 25 kg/m2 (90th percentile for age). He has a soft systolic murmur, a diffusely tender abdomen without rebound or guarding, and his rectal examination findings are notable for grossly bloody stools without other abnormality.

After the administration of intravenous fluids and packed red blood cells, his heart rate normalizes. Endoscopy with colonoscopy is performed after a bowel cleanout. The upper endoscopy findings are normal. The colonoscopy demonstrates severe mucosal erythema, friability and edema, with exudate from the rectum to the cecum (Item Q74); pathology demonstrates severe chronic colitis without granulomas with a normal terminal ileum. Magnetic resonance enterography reveals a thickened colonic bowel wall with a normal-appearing small intestine.

Of the following, the BEST treatment for the adolescent’s condition is

A.	exclusive enteral nutrition
B.	intravenous corticosteroids
C.	oral azathioprine
D.	oral tacrolimus
A

The adolescent in the vignette has findings consistent with ulcerative colitis (UC); the constellation of signs, symptoms, and laboratory findings meets the criteria for acute severe colitis (ASC). The initial therapy recommended for ASC includes intravenous corticosteroids and supportive care (including treatment of anemia and nutritional support as clinically warranted).

How well did you know this?
1
Not at all
2
3
4
5
Perfectly
66
Q

A 3-year-old girl (Item Q113) is seen for a health supervision visit with her adoptive parents. Prenatal history is not known. Gross motor skills are mildly delayed. Her other developmental milestones are appropriate for age. Her weight and height are at the 3rd percentile, and her head circumference is at the 5th percentile.

Of the following, the girl’s clinical findings are MOST likely due to

A. congenital cytomegalovirus infection
B. folate deficiency during pregnancy
C. maternal alcohol consumption during pregnancy
D. uncontrolled maternal diabetes during pregnancy

A

The girl in the vignette demonstrates features of fetal alcohol spectrum disorder (FASD) including growth restriction, gross motor delay, and the physical findings shown in Item C113A (midface hypoplasia, a relatively smooth philtrum, small palpebral fissures, and a narrow upper lip with a poorly defined “cupid’s bow”). Fetal alcohol spectrum disorder results from maternal alcohol consumption during pregnancy. There is no safe limit for alcohol consumption during pregnancy; FASD can occur with any level of alcohol intake. Both the American Academy of Pediatrics and the American College of Obstetricians and Gynecologists recommend complete abstinence from alcohol for preconceptional and pregnant women. The clinical features of FASD include:

Growth restriction (height and/or weight ≤10%)
Dysmorphic features (small palpebral fissures, thin vermillion border, flat nasal bridge, smooth philtrum, railroad track appearance to the ear, hockey stick palmar crease, and midface hypoplasia)
Central nervous system abnormalities
Congenital cytomegalovirus infection can present with prematurity, intrauterine growth retardation, microcephaly, poor feeding, thrombocytopenia, hepatosplenomegaly, intracranial calcifications, sensorineural hearing loss, and chorioretinitis. While developmental delay may be seen, there are no dysmorphic facial features associated with the condition.

How well did you know this?
1
Not at all
2
3
4
5
Perfectly
67
Q

A 5-year-old boy who recently emigrated from a refugee camp in Burma is brought to the office with complaints of abdominal pain, anorexia, and occasional watery diarrhea for more than 3 weeks. He is afebrile with normal vital signs. He has abdominal distention but no hepatomegaly or splenomegaly. The remainder of his physical examination findings are normal.

Laboratory data are shown:

Laboratory Test

Result

White blood cell count

12,000/μL (12.0 × 109/L)

Neutrophils

68%

Lymphocytes

26%

Monocytes

2%

Eosinophils

4%

Hemoglobin

11.7 g/dL (117 g/L)

Platelet count

274 × 103/μL (274 × 109/L)

A stool specimen is obtained for microscopic examination (Item Q114).

Of the following, the MOST likely pathogen causing this boy’s illness is

A.	Ancylostoma duodenale
B.	Ascaris lumbricoides
C.	Necator americanus
D.	Trichuris trichura
A

The boy in the vignette is infected with Ascaris lumbricoides, an intestinal roundworm commonly found in locations with tropical climates (eg, Burma). The boy’s symptoms (abdominal pain, anorexia, watery diarrhea), physical examination finding (abdominal distention), and characteristic appearance of the fertilized eggs on microscopic examination of a stool specimen (oval shape surrounded by a thick shell with an external proteinaceous layer) are consistent with A lumbricoides infection.

How well did you know this?
1
Not at all
2
3
4
5
Perfectly
68
Q

A previously healthy, 7-year-old girl is seen in the office for right knee pain and swelling that developed and has been worsening over the past several days. She also has pain while walking. She had an episode of acute pharyngitis 3 weeks ago that resolved without intervention. On physical examination, the girl has normal vital signs for her age. She has erythema and swelling of the right knee with pain on motion. On chest auscultation, there is a long, high-pitched systolic murmur at the apex. There is an erythematous, serpiginous rash throughout her trunk that blanches. The remainder of her examination findings are unremarkable.

Of the following, the BEST test to establish the child’s diagnosis is

A.	antistreptolysin O antibody titer
B.	C-reactive protein
C.	echocardiography
D.	rheumatoid factor
A

The child in the vignette meets several of the modified Jones criteria for diagnosing acute rheumatic fever (ARF): arthritis, erythema marginatum, and carditis. To definitively make the diagnosis, however, one needs to also demonstrate evidence of a prior group A streptococcal (GAS) infection. The best test to establish the diagnosis of ARF is antistreptolysin O titer. A C-reactive protein level and echocardiography may be useful in the complete assessment and management of the condition but will not clarify the underlying diagnosis. In general, a rheumatoid factor level is unlikely to be helpful in the diagnosis of knee pain and swelling in a young child and is not recommended to make the diagnosis in this child with a clinical picture suggestive of ARF.

How well did you know this?
1
Not at all
2
3
4
5
Perfectly
69
Q

A 2-year-old, previously healthy boy is brought to the emergency department for difficulty walking. Two weeks ago he developed an upper respiratory infection that self-resolved. Over the past 4 days, he has had difficulty standing and walking, and requires support to take steps such as holding onto furniture or his parents’ hands. When he sits, he nods his head and has trouble feeding himself. His parents deny recent trauma or ingestion.

On physical examination, the boy is alert and playful in his mother’s lap, resting against her. He orients when his name is called. His pupils are equal, round, and reactive to light, extraocular movements are intact with a few beats of end gaze nystagmus bilaterally, facies are symmetric, and tongue is midline. Muscles have normal bulk, tone, and strength with symmetric, full movements. Deep tendon reflexes are initially difficult to elicit but are 2 throughout; toes are downgoing to plantar stimulation. Sensation is intact. When reaching for objects he has mild dysmetria bilaterally. He maintains a seated posture without support but has titubation. He can walk with significant support (holding his mother’s hands) with a wide-based gait.

Results of a complete blood cell count, comprehensive metabolic panel, toxicology screen (urine and serum), and brain magnetic resonance imaging are normal.

Of the following, the MOST likely diagnosis for this boy is

A.	acute cerebellar ataxia
B.	acute disseminated encephalomyelitis
C.	developmental coordination disorder
D.	Miller-Fisher syndrome
A

The boy in the vignette has acute cerebellar ataxia, a self-limited, postinfectious disorder primarily affecting children younger than 5 years. Affected children exhibit truncal and gait ataxia, often coming to medical attention due to the inability to stand or sit without support. Dysmetria and titubation (head nodding) while sitting are further evidence of cerebellar dysfunction. Affected children are otherwise well appearing with normal mental status. Symptoms are usually maximal at onset and gradually improve over days to weeks, rarely months. Management is supportive. There is no evidence that immune therapy (eg, corticosteroids or intravenous immunoglobulin [IVIG]) affects outcome.

PREP Pearls
The differential diagnosis of balance disturbances in childhood is broad and can localize anywhere within the nervous system. A careful history and physical examination are critical for appropriate localization, evaluation, and diagnosis.
Ataxia refers to an absence of coordination and can be acute, subacute, or chronic depending on the time course of symptoms.
Acute cerebellar ataxia is a self-limited, post-infectious disorder presenting with acute onset of truncal and gait ataxia, often in young children. Treatment is supportive.

How well did you know this?
1
Not at all
2
3
4
5
Perfectly
70
Q

A 13-year-old adolescent girl is seen in the office for concern about absent menses. She has been healthy except for multiple ear infections as an infant. The girl is doing well in school and participates in club soccer. Her sexual maturity rating is 1 for breast development and 3 for pubic hair. The remainder of the girl’s physical examination findings are unremarkable. Her growth charts are shown in Item Q119A and Item Q119B.

Of the following, the test/assessment MOST likely to reveal this girl’s diagnosis is a

A.	bone age radiograph
B.	calorie intake and expenditure assessment
C.	karyotype
D.	tissue transglutaminase IgA antibody level
A

The girl in the vignette has Turner syndrome. The best test to reveal her diagnosis is a karyotype, which would show an absent or missing part of a sex chromosome (45,X or a variant). Her short stature, absent breast development, and history of multiple ear infections as an infant are all features of Turner syndrome.

Delayed puberty in girls is defined as lack of breast development by age 13 years. In adolescents with delayed puberty, measurement of gonadotropins (luteinizing hormone and follicle-stimulating hormone [FSH]) is indicated to determine whether the pubertal failure is primary (ovarian/testicular) or secondary (hypothalamic-pituitary). In adolescent girls with Turner syndrome, the FSH is often elevated due to primary ovarian failure. The girl in the vignette has a sexual maturing rating of 3 for pubic hair, which is secondary to adrenal androgen production.

Bone age radiography is an important part of the evaluation of short stature and delayed puberty. Typically, conditions that delay puberty also delay bone age as estradiol is the primary hormone responsible for growth plate maturation. Although bone age radiography may help in treatment planning for the girl in the vignette, it is not specific enough to determine the etiology of her delayed puberty. A delayed bone age in an otherwise healthy child or adolescent with a normal growth rate aids in the diagnosis of constitutional delay of growth and puberty.

How well did you know this?
1
Not at all
2
3
4
5
Perfectly
71
Q

A 2-day-old neonate is transferred to the neonatal intensive care unit for a dusky appearance. On physical examination, he has a heart rate of 160 beats/min, respiratory rate of 60 breaths/min, blood pressure of 70/55 mm Hg in the right arm, and right upper and lower extremity oxygen saturations of 78% in room air. He is tachypneic but comfortable, without increased work of breathing. Breath sounds are clear. He has a normal S1, a single S2, and a continuous murmur heard in the left side of the chest; the liver edge is palpable 2 cm below the right costal margin; and his extremities are warm and well perfused. A chest radiograph is obtained (Item Q120).

Of the following, the BEST next step in this neonate’s management is to

A.	initiate epinephrine infusion
B.	initiate inhaled nitric oxide C.	initiate prostaglandin infusion
D.	intubate for respiratory failure
A

he neonate in the vignette is cyanotic, with a room air oxygen saturation of 78% measured in the right upper and lower extremities. He is comfortably tachypneic with good perfusion. On auscultation, he has clear breath sounds, a single S2, and a continuous murmur. The chest radiograph (Item C120A) shows oligemic lungs without infiltrate, a boot-shaped heart, and a right aortic arch. These findings are consistent with tetralogy of Fallot with pulmonary atresia. Although this diagnosis will need to be confirmed by echocardiography, for this ductal-dependent condition it is important to first initiate prostaglandin infusion to prevent further closure of the patent ductus arteriosus (PDA). The neonate does not have evidence of low cardiac output, so administration of epinephrine is not indicated. He does not have evidence of respiratory failure, so intubation is not indicated. Inhaled nitric oxide would not correct his hypoxemia.

The mnemonic 5 Ts and H is helpful for remembering many of the types of cyanotic congenital heart disease:

Tetralogy of Fallot
Total anomalous pulmonary venous connection
Transposition of the great arteries
Tricuspid atresia
Truncus arteriosus
Hypoplastic left heart syndrome

How well did you know this?
1
Not at all
2
3
4
5
Perfectly
72
Q

A 15-year-old adolescent girl is brought to the emergency department for a 48-hour history of blurry vision and pain with movement of the left eye. Today, she is unable to see anything out of her left eye. She complains of mild, left frontal headache but denies fever or eye discharge. There is no history of recent illness or eye injury. Her vital signs are normal, and she is in no acute distress. Her cornea and conjunctivae are clear without discharge or injection. She has mild pain with movement of her left eye but has full range of extraocular movement. When light is directed at her right pupil, there is normal constriction of both pupils. When light is directed at her left pupil, both pupils remain dilated. Fundoscopy shows blurred optic disc margins on the left with sharp margins on the right. The remainder of her physical examination findings are normal.

Of the following, the MOST likely diagnosis for this adolescent is

A.	migraine variant
B.	optic neuritis
C.	orbital cellulitis
D.	pseudotumor cerebri
A

The adolescent in the vignette has findings consistent with left optic neuritis. The swinging flashlight test, also known as the Marcus Gunn pupil test, is performed to detect an afferent pupillary defect. The test is performed by shining a light on one eye for 2 seconds and then alternating the light from eye to eye. Normally, both pupils constrict in response to shining the light on one pupil. If the pupils remain dilated, an afferent pupillary defect is likely, indicating disease of the retina or optic nerve. The adolescent has a lack of pupillary response on the left side with a normal direct and consensual response on the right side, indicating a left afferent pupillary defect. This finding, along with blurring of her left optic disc, makes optic neuritis the most likely diagnosis.

The first-line treatment of pediatric optic neuritis is high-dose, pulsed methylprednisolone for 3 to 5 days followed by a taper over 2 weeks. Children and adolescents resistant to steroid therapy may benefit from treatment with intravenous immunoglobulin and plasmapheresis.

PREP Pearls
Optic neuritis can present with acute vision loss and findings of an afferent pupillary defect with blurring of the optic disc margins.
Evaluation of children and adolescents with suspected optic neuritis should include magnetic resonance imaging of the brain and orbits.
Optic neuritis can be the initial clinical presentation of demyelinating disorders such as multiple sclerosis.

How well did you know this?
1
Not at all
2
3
4
5
Perfectly
73
Q

A 3-year-old girl is brought to the emergency department (ED) for difficulty breathing. She developed a sudden, severe sore throat and fever today. Her vital signs include a temperature of 40.1°C, heart rate of 176 beats/min, respiratory rate of 26 breaths/min, blood pressure of 76/59 mm Hg, and oxygen saturation of 91% in room air. On arrival at the ED, the girl appears toxic and has severe respiratory distress and difficulty swallowing. She is sitting upright and holding her head in an extended position while resting her arms on her knees. She has stridorous respirations associated with tachypnea and suprasternal retractions.

Of the following, the BEST next step in this girl’s management is

A.	administration of intravenous ceftriaxone
B.	administration of intravenous corticosteroid
C.	administration of nebulized racemic epinephrine
D.	rapid sequence induction and intubation
A

The girl in the vignette is displaying signs and symptoms of airway obstruction concerning for epiglottitis. The best next step in her management is rapid sequence induction and intubation. Other interventions, such as nebulizer treatments, intravenous corticosteroids or antibiotics, or imaging studies should not be undertaken until a definitive airway is secured. Every effort should be made to keep the child as calm and quiet as possible until the airway is secured.

PREP Pearls
Epiglottitis is a rare life-threatening condition that results in severe swelling of the epiglottis, which can lead to complete airway obstruction.
The clinical presentation of epiglottis is an overtly toxic-appearing child with sudden onset and rapid progression of fever, drooling, stridor, muffled voice, and difficulty swallowing. The classic diagnostic triad, the “3 Ds,” includes drooling, dysphagia, and distress.
The critically important first step in the management of a child with suspected epiglottitis is obtaining a definitive and secure airway with an endotracheal tube

How well did you know this?
1
Not at all
2
3
4
5
Perfectly
74
Q

A 14-year-old adolescent boy is seen for a preparticipation physical examination to play soccer. He is healthy with no concerns voiced by him or his parents. His physical examination findings are normal except for a systolic ejection click heard best at the right upper sternal border. Further inquiry reveals that the boy’s paternal grandfather had aortic valve surgery at age 55 years. The boy is referred to a pediatric cardiologist.

Of the following, the boy’s MOST likely diagnosis is

A.	bicuspid aortic valve
B.	partial anomalous pulmonary venous return
C.	pulmonary valve stenosis
D.	ventricular septal defect
A

The boy in the vignette has a systolic ejection click heard at the right upper sternal border, suggestive of aortic valve pathology, and a family history (paternal grandfather) of aortic valve pathology. Of the response choices, bicuspid aortic valve (BAV) is the most likely diagnosis. He should be evaluated by a pediatric cardiologist; echocardiography is needed to confirm the diagnosis. If there is no significant valve dysfunction or aortic dilation, children with BAV can often play sports and exercise without restriction.

PREP Pearls
Bicuspid aortic valve, in which the aortic valve develops with only 2 leaflets instead of the normal 3, is one of the most common congenital heart defects.
Bicuspid aortic valve occurs in males more than females and can be familial.
Individuals with bicuspid aortic valve can develop valve dysfunction (stenosis, regurgitation, or both), aortic dilation, or aortic aneurysm.

How well did you know this?
1
Not at all
2
3
4
5
Perfectly
75
Q

A 7-year-old girl is evaluated for a 1-week history of a dry, hacking cough. Today, she had an episode of posttussive emesis. The girl appears well. She has a temperature of 37.5°C, heart rate of 90 beats/min, respiratory rate of 18 breaths/min, and oxygen saturation of 100% in room air. Her lungs are clear on auscultation. Bilateral subconjunctival hemorrhages and a few scattered petechiae are noted over the neck. Her parents are concerned that the girl’s siblings, ages 6 weeks and 5 years, will catch her illness. The children are all up to date with their immunizations.

Of the following, over the next 21 days, the MOST appropriate postexposure management is to

A.	monitor only the 5-year-old sibling for symptoms
B.	monitor only the 6-week-old sibling for symptoms
C.	start azithromycin treatment for both siblings
D.	start trimethoprim-sulfamethoxazole treatment for both siblings
A

Pertussis postexposure prophylaxis with azithromycin is recommended for all close contacts of the index case irrespective of immunization status. Individuals are considered close contacts if they are either household members or asymptomatic persons who had face-to-face exposure within 3 feet of a symptomatic individual in a childcare facility, school, or health care setting. Exposure is defined as direct contact with respiratory, oral, or nasal secretions or sharing the same space with the symptomatic individual for 1 hour or more. Prompt postexposure prophylaxis is recommended, given the high transmission risk (secondary attack rate of 80% in susceptible persons) and effectiveness of azithromycin. Trimethoprim-sulfamethoxazole is reserved for prophylaxis in children who cannot tolerate azithromycin; it is contraindicated in infants younger than 2 months. All exposed individuals should be monitored for symptoms for 21 days.

PREP Pearls
Pertussis is a highly contagious infection with a typical duration of 6 to 10 weeks.
There are 3 stages of pertussis infection: catarrhal (upper respiratory infection symptoms), lasting 1 to 2 weeks; paroxysmal (repeated coughing spells), lasting 2 to 4 weeks; and convalescent (improving cough), lasting 4 to 12 weeks.
Household members and close contacts of individuals diagnosed with pertussis infection should receive postexposure prophylaxis with azithromycin, irrespective of their immunization status.

How well did you know this?
1
Not at all
2
3
4
5
Perfectly
76
Q

A 4-day-old male neonate is seen in the office for a health supervision visit. The neonate was born at term, had an uneventful prenatal and nursery course, and was discharged 2 days after birth. His mother is planning to exclusively breastfeed and asks if any nutrient supplements are recommended for her son.

Of the following, the BEST recommendation is

A.	start vitamin D and iron supplementation at age 4 months
B.	start vitamin D and iron supplementation now
C.	start vitamin D supplementation now and iron supplementation at age 4 months
D.	vitamin D and iron supplementation are not necessary
A

The breastfed neonate in the vignette should start vitamin D supplementation now and iron at age 4 months. Although breast milk provides excellent nutrition during the first year after birth, the vitamin D and iron content in breast milk is not adequate to meet the infant’s nutritional needs.

How well did you know this?
1
Not at all
2
3
4
5
Perfectly
77
Q

A 1-day-old newborn was delivered at 39 weeks’ gestation via normal spontaneous vaginal delivery to a 21-year-old primigravida mother. The pregnancy was complicated by oligohydramnios. Prenatal ultrasonography revealed bilateral hydronephrosis. The neonate has had 1 small void since birth. His vital signs are normal for age, and his physical examination findings are unremarkable.

Postnatal renal ultrasonography reveals severe bilateral hydroureteronephrosis with a thick-walled urinary bladder.

Of the following, the BEST next step in this neonate’s evaluation is to order

A.	a diuretic renal scan
B.	magnetic resonance imaging of the kidneys and bladder
C.	repeat renal ultrasonography after 1 week
D.	voiding cystourethrography
A

The male neonate in the vignette most likely has posterior urethral valves (PUVs). This diagnosis is supported by the history of bilateral antenatal hydronephrosis, maternal oligohydramnios, and severe bilateral hydroureteronephrosis with a thick-walled urinary bladder on postnatal renal ultrasonography. The best next step in his evaluation is to perform voiding cystourethrography (VCUG). The VCUG would likely demonstrate the classic findings of a dilated posterior urethra during the voiding phase, trabeculated bladder, and perhaps vesicoureteral reflux (VUR) (Item C126).

PREP Pearls
Posterior urethral valves are abnormal mucosal folds within the lumen of the posterior urethra that obstruct the flow of urine.
Children with suspected posterior urethral valves should have voiding cystourethrography (VCUG) performed. A dilated posterior urethra on VCUG is consistent with the diagnosis.
Children with posterior urethral valves are at increased risk of chronic kidney disease, even with prenatal diagnosis and early intervention.

How well did you know this?
1
Not at all
2
3
4
5
Perfectly
78
Q

A 2-year-old girl is seen for hives of 24 hours’ duration. The girl woke up yesterday with scattered lesions on her torso. Since onset, the lesions have come and gone in various areas of her torso. Two weeks ago, she was diagnosed with a viral upper respiratory tract infection and acute otitis media for which she was prescribed amoxicillin. She finished the amoxicillin 4 days ago. For the past 2 days, the girl has had a temperature of 38°C, mild cough, and rhinorrhea. She has continued at her usual activity level and is eating a wide variety of foods. Her medical history is unremarkable. Her mother has hypothyroidism and her older brother has a peanut allergy. The girl’s vital signs are normal. Physical examination findings are remarkable only for clear rhinorrhea and scattered 1- to 5-cm diameter hives on her torso.

Of the following the MOST likely cause of this girl’s hives is

A.	amoxicillin
B.	food allergy
C.	hypothyroidism
D.	viral illness
A

The most likely cause for the hives exhibited by the girl in the vignette is a viral illness. Viral infections cause almost 60% of acute urticaria seen in children, and this girl has clear symptoms of a viral illness. Amoxicillin is a less likely cause of this girl’s urticaria because urticaria caused by amoxicillin usually does not have a delayed onset. Urticaria from food allergies usually presents within 2 hours of exposure. Hypothyroidism, specifically autoimmune hypothyroidism, may predispose some children to urticaria, but in these cases, the urticaria is usually chronic, lasting more than 6 weeks.

PREP Pearls
The most common cause of acute urticaria in children is a viral infection.
The first-line treatment for both acute and chronic urticaria is a nonsedating antihistamine such as cetirizine or loratadine.
Children with cold-induced urticaria should be prescribed an epinephrine auto-injector because of the risk of anaphylaxis.

How well did you know this?
1
Not at all
2
3
4
5
Perfectly
79
Q

A 14-year-old adolescent boy is brought to the emergency department by ambulance after his parents found him unresponsive in his bedroom. Upon arrival at the home, the emergency medical services team witnessed a 5-minute generalized tonic clonic seizure for which the paramedic administered diazepam 2 mg intravenously. The boy has a history of depression and attention-deficit/hyperactivity disorder. His current medications include melatonin, amitriptyline, and dexmethylphenidate.

In the emergency department, the boy has a temperature of 38.3°C, heart rate of 188 beats/min, respiratory rate of 10 breaths/min, blood pressure of 72/45 mm Hg, and oxygen saturation of 90% in room air. He appears sleepy, without apparent seizure activity. His physical examination findings are significant for dilated pupils; flushed, dry skin; and dry mucous membranes. He has shallow breathing and decreased respiratory effort. The remainder of his physical examination findings are unremarkable.

The boy is intubated, and his airway is secured. Results of a complete blood count and comprehensive metabolic panel are normal. Shortly after intubation he is noted to have rhythm changes on the cardiac monitor; 12-lead electrocardiography is obtained and shown (Item Q128).

Of the following, the BEST next step in this boy’s management is to administer intravenous

A.	flumazenil
B.	magnesium sulfate
C.	physostigmine
D.	sodium bicarbonate
A

The adolescent in the vignette is demonstrating classic anticholinergic findings (seizure; respiratory depression; dilated pupils; dry, flushed skin; tachycardia; and arrhythmia) caused by an overdose of amitriptyline, a tricyclic antidepressant (TCA). The treatment for TCA-induced, wide complex ventricular tachycardia and resulting hypotension is administration of intravenous sodium bicarbonate, with the goal of raising the serum pH to approximately 7.5.

Flumazenil, the antidote for a benzodiazepine overdose, is contraindicated in individuals with TCA overdose as it may induce seizures. Flumazenil should also not be administered to individuals with an unknown ingestion, as it may precipitate seizures in those who are benzodiazepine dependent.

PREP Pearls
An electrocardiogram should be obtained immediately in a child with suspected or confirmed ingestion of a tricyclic antidepressant.
Sodium bicarbonate is the mainstay of treatment in children with tricyclic antidepressant toxicity and should be initiated in any child who is hemodynamically unstable, acidotic, hypotensive, or exhibiting seizure activity or a QRS interval prolongation of more than 100 milliseconds.
The goal of sodium bicarbonate therapy for tricyclic antidepressant–induced QRS interval prolongation is to narrow the QRS complex and achieve and maintain a serum pH of 7.5.

How well did you know this?
1
Not at all
2
3
4
5
Perfectly
80
Q

A 3-year-old girl in the pediatric intensive care unit has acute respiratory failure from respiratory syncytial virus bronchiolitis. She is in marked respiratory distress, with a respiratory rate of 60 breaths/min and oxygen saturation of 73% in room air. She is somnolent and lethargic. Auscultation of the lungs demonstrates diffuse, coarse rales, wheezes, and rhonchi. The decision is made to perform rapid-sequence endotracheal intubation, after which her oxygen saturation is 98% on an FiO2 of 45%. Approximately 20 minutes after initiation of mechanical ventilation, the girl has an acute oxygen desaturation to 75% which does not improve when 100% FiO2 is provided by the ventilator. Visual inspection of her chest reveals equal but restricted chest rise, and auscultation reveals diminished breath sounds in all lung fields.

Of the following, the BEST next step in this girl’s management is

A.	chest radiography
B.	manual ventilation with a resuscitation bag
C.	needle thoracostomy of the chest
D.	suctioning of the endotracheal tube
A

The girl in the vignette has acute respiratory failure requiring intubation and mechanical ventilation. Given her ongoing hypoxia despite the provision of 100% FiO2 by the ventilator, equal but diminished breath sounds, and equal chest rise, the best next step in management is to provide manual ventilation with a resuscitation bag via the endotracheal tube to both assess and manage the cause of ventilation failure.

Finally, suctioning of the endotracheal tube can alleviate mucus or secretion-related endotracheal tube obstruction. However, given the ongoing significant oxygen desaturations, manual ventilation with a resuscitation bag is indicated before suctioning to attempt to restore normal oxygenation quickly.
PREP Pearls
When acute hypoxia occurs in mechanically ventilated children, manual ventilation with a resuscitation bag should be performed with 100% FiO2 until the underlying cause of hypoxia can be addressed and corrected.
Endotracheal intubation may be required for children with respiratory failure, impending respiratory arrest, or those with neurologic impairment or severely altered mental status where airway protective reflexes cannot be maintained.
A common mnemonic used to evaluate and manage a successfully intubated patient who subsequently develops hypoxemia is DOPE: Displacement/Dislodgement of the endotracheal tube, Obstruction of the endotracheal tube, Pneumothorax, and Equipment failure.

How well did you know this?
1
Not at all
2
3
4
5
Perfectly
81
Q

A plan is made to treat the adolescent with antibiotics and draw repeat titers for Bartonella henselae in 4 weeks.

Of the following, the BEST next step in management of this adolescent is to administer

A.	amoxicillin B.	azithromycin
C.	cephalexin
D.	penicillin
A

The adolescent in the vignette has unilateral cervical lymphadenopathy associated with fever. This clinical scenario, including exposure to kittens, is suggestive of Bartonella henselae infection or “cat-scratch disease.” The adolescent in the vignette has lymphadenopathy, fever, headache, and abdominal pain as well as exposure to kittens, therefore, empiric treatment with azithromycin would be appropriate. Empiric treatment with azithromycin can be considered in situations compatible with this clinical syndrome (eg, acute lymphadenopathy and exposure to kittens). Penicillin, amoxicillin, and first-generation cephalosporins are not recommended for treatment of B henselae.

PREP Pearls
Cat-scratch disease is a common cause of lymphadenopathy in children, caused by the gram-negative bacillus Bartonella henselae.
The majority of Bartonella henselae infections are asymptomatic. The severity of disease varies based on the age and immune status of the host, ranging from regional lymphadenopathy to systemic disseminated infection.
Uncomplicated cases of lymphadenopathy due to Bartonella henselae do not require treatment. However, a 5-day course of azithromycin results in reduction of lymph node size and hastens resolution of symptoms.

How well did you know this?
1
Not at all
2
3
4
5
Perfectly
82
Q

A 14-year-old adolescent girl is seen for evaluation of a 9-kg weight gain over the past year. Her mother is concerned about a possible hormonal etiology. The adolescent reports fatigue and depression. She often stays up late working on her computer and will snack during this time. She does not sleep well and snores loudly. She reports occasional headaches, but no vision changes, nausea, or vomiting. She has no muscle weakness. Menarche occurred at age 11 years, and her menses are regular. Her mother recently underwent bariatric surgery.

On physical examination, the adolescent’s blood pressure is 132/86 mm Hg and heart rate is 86 beats/min. Her weight is 85 kg (98th percentile), height is 163 cm (65th percentile) and body mass index is 32 kg/m2 (98th percentile). She has hyperpigmentation around her neck and in her skin creases, and thin striae on her abdomen. The remainder of her physical examination findings are normal.

Of the following, the BEST next step in this adolescent’s evaluation is to

A.	gather a detailed dietary history
B.	measure a serum morning cortisol level
C.	obtain magnetic resonance imaging of the abdomen
D.	obtain thyroid function tests
A

detailed diet history

How well did you know this?
1
Not at all
2
3
4
5
Perfectly
83
Q

A 5-day-old male neonate born at 36 weeks’ gestation is brought to the emergency department for evaluation of a 1-day history of poor feeding, decreased activity, and episodes of excessive crying. He is exclusively breastfed. The infant was discharged home 2 days after birth with a bilirubin level of 8 mg/dL (136.83 µmol/L). The mother’s blood group is O positive; she does not know the neonate’s blood type. On physical examination, the neonate has a high-pitched cry, marked jaundice, increased extensor tone in the upper and lower extremities, intermittent arching, and an exaggerated Moro reflex. There is no clonus. Laboratory results include a total bilirubin level of 32 mg/dL (547.33 µmol/L), a blood type of A positive, and a positive direct antiglobulin (Coombs) test result. The neonate is admitted, and phototherapy is started.

Of the following, the MOST important next step in the management of this neonate is

A.	continued phototherapy
B.	double volume exchange transfusion
C.	intravenous fluid therapy
D.	partial volume exchange transfusion
A

The neonate in the vignette is a late preterm infant with severe hyperbilirubinemia. His physical examination findings are consistent with phases 1 to 2 of acute bilirubin neurotoxicity. To prevent further neurologic deterioration and decrease the risk of the long-term neurodevelopmental sequelae of kernicterus, a double volume exchange transfusion should be performed as soon as possible. For this neonate, the total bilirubin level for which exchange transfusion is indicated, is 22.2 mg/dL (379.7 µmol/L); this neonate’s level is well above that (Item C133A).

Almost all preterm neonates and approximately 60% of term neonates will develop physiologic jaundice. This jaundice occurs due to catabolism of red blood cells (RBCs) (the half-life of RBCs in neonates is 90 days compared with 120 days in adults), increased RBC volume, immature hepatic conjugation, and delayed establishment of feedings, leading to reduced excretion of bilirubin and increased enterohepatic circulation of bilirubin. Item C133B lists risk factors for severe hyperbilirubinemia in neonates; the neonate in the vignette’s risk factors include gestational age (late preterm), exclusive breastfeeding, and isoimmune hemolytic disease.

Phototherapy should continue before and after exchange transfusion but will not rapidly decrease the bilirubin level, which is critical to minimizing the neurotoxic effects of severe hyperbilirubinemia. Intravenous fluid is helpful to address dehydration, if present, but it is not a treatment for hyperbilirubinemia. Partial volume exchange transfusion is performed in neonates with chronic anemia for whom heart failure is a concern and in neonates with symptomatic polycythemia with a central hematocrit greater than 65% to 70% (0.65-0.70); it is not an appropriate treatment for neonatal hyperbilirubinemia.

PREP Pearls
A high level of bilirubin or an insufficient amount of serum protein to bind the bilirubin leads to circulating free bilirubin, which can easily penetrate the blood–brain barrier.
To prevent further neurologic deterioration and decrease the risk of the long-term neurodevelopmental sequelae of kernicterus, a double volume exchange transfusion should be performed as soon as possible when exchange transfusion is indicated.
Some risk factors for significant neonatal hyperbilirubinemia include predischarge total serum or transcutaneous bilirubin level close to phototherapy threshold, lower gestational age, exclusive breastfeeding, jaundice observed in the first 24 hours, inherited red blood cell disorders, phototherapy before discharge, hemolysis from any cause, macrosomic infant, and Down syndrome.

How well did you know this?
1
Not at all
2
3
4
5
Perfectly
84
Q

A new patient is seen for a health supervision visit. The child is healthy with a normal birth history. He is crawling and pulling himself up to stand using the couch. Occasionally, he will stand up unassisted in the middle of the floor. He uses his thumb and index finger to pick up a piece of breakfast cereal. He is able to find his pacifier when his mother hides it under a blanket. He says ”mama” and “uh-oh” appropriately. He will stop and look at his parents when they say “no.”

Of the following, this child’s development is MOST consistent with that of a

A.	9-month-old
B.	12-month-old
C.	15-month-old
D.	18-month-old
A

The child in the vignette is at a developmental level that is most consistent with that of a 12-month-old.

The fine motor development of a 12-month-old child should include the ability to pick things up between thumb and pointer finger, like small bits of food. The gross motor development of a 12-month-old child should include cruising and taking first steps. Often a child this age will stand independently, demonstrating age-appropriate tone and balance, but may not yet be willing to attempt the actual motion of walking. At 15 to 18 months of age we would expect a child to be walking well independently and beginning to run and climb.

How well did you know this?
1
Not at all
2
3
4
5
Perfectly
85
Q

A 7-month-old boy is admitted to the hospital for dehydration, weight loss, vomiting, and diarrhea. The diarrhea began approximately 4 weeks ago. He has 8 to 10 nonbloody, watery stools daily and frequent diaper rashes, which have been difficult to clear despite frequent application of zinc oxide diaper cream. His diet consists of cow milk formula, pureed fruits and vegetables, and oat and wheat infant cereal. He recently began to have intermittent, nonbilious, nonbloody emesis and decreased oral intake (both solids and formula). On physical examination, the infant is pale and fussy but consolable. His weight is at the third percentile, length is at the 50th percentile, and weight for length is fifth percentile for age. He has tachycardia, a capillary refill time of 2 seconds, a soft and nondistended abdomen without hepatosplenomegaly, and perianal erythema. The remainder of his physical examination findings are normal.

The infant is given intravenous fluids and made nil per os. Stool infectious studies, including viral and bacterial testing, are negative. His diarrhea significantly improves after 24 hours of bowel rest.

Of the following, the BEST next step in this infant’s management is

A.	fiber supplementation
B.	loperamide
C.	serum vasoactive intestinal peptide concentration
D.	upper endoscopy
A

The infant in the vignette has chronic diarrhea (duration >14 days) associated with failure to thrive and therefore requires further evaluation. For a child with this clinical presentation, without evidence of secretory diarrhea, upper endoscopy is the best management option. With further evaluation, he was diagnosed with allergic enteropathy.

Chronic diarrhea may occur because of the following mechanisms:

Osmotic diarrhea: unabsorbed intestinal solutes drive luminal retention of water (via osmotic gradient)
Examples: lactose intolerance, exocrine pancreatic insufficiency
Diarrhea improves when unabsorbed nutrients are removed from the diet (or in nil per os [NPO] status)
Secretory diarrhea: excessive intestinal electrolyte and/or fluid secretion
Examples: neuroendocrine tumors (eg, VIPoma, neuroblastoma), congenital chloride diarrhea
Diarrhea persists despite NPO status
Low stool osmolar gap:
Stool Total Osmolarity – 2 × [Stool Na+ + Stool K+]
Inflammatory diarrhea: may include both secretory and osmotic mechanisms because intestinal fluid secretion may be excessive and the inflamed bowel may not completely absorb intestinal solutes
Examples: infectious diarrhea (excessive intestinal fluid secretion), inflammatory bowel disease, celiac disease, and allergic enteropathy
Diarrhea may improve with NPO status
Diarrhea caused by dysmotility: occurs because of decreased intestinal transit time
Examples: chronic nonspecific diarrhea, irritable bowel syndrome

PREP Pearls
Chronic diarrhea may be classified based on the mechanism: osmotic, secretory, inflammatory, and dysmotility associated.
Infants with chronic diarrhea and failure to thrive require additional evaluation, including blood and stool studies; endoscopic evaluation may be useful to determine the cause.

How well did you know this?
1
Not at all
2
3
4
5
Perfectly
86
Q

An 8-year-old girl is seen for a health supervision visit. She was recently diagnosed with a left optic pathway glioma that is being monitored by her health care team. She is doing well in the third grade in a general education classroom. The girl’s physical examination findings are significant only for multiple café au lait macules (at least 6 greater than 5 mm). Sexual maturity rating is 1 for breast and pubic hair development.

Of the following, this girl’s MOST likely diagnosis is

A.	Li-Fraumeni syndrome
B.	neurofibromatosis type 1
C.	neurofibromatosis type 2
D.	tuberous sclerosis complex
A

The girl in the vignette has neurofibromatosis type 1 (NF1). She meets the clinical criteria for NF1 with her optic pathway glioma and at least 6 café au lait macules larger than 5 mm. Neurofibromatosis type 1 is a tumor predisposition syndrome inherited in an autosomal dominant manner with complete penetrance. An example of a NF1 pedigree is shown in Item C136A. Clinical criteria for NF1, recently revised, are below:

How well did you know this?
1
Not at all
2
3
4
5
Perfectly
87
Q

An 8-month-old infant undergoing treatment for acute lymphoblastic leukemia is seen in the emergency department for evaluation of fever, diarrhea, and a generalized macular, erythematous, blanching, nonpruritic rash. Laboratory evaluation findings are significant for transaminitis. After further evaluation, the infant’s findings are determined to be secondary to a packed red blood cell transfusion administered 2 weeks ago.

Of the following, the blood product preparation strategy MOST likely to have prevented this infant’s findings is

A.	irradiation
B.	leukoreduction
C.	selection of the freshest product
D.	washing
A

Fever, transaminitis, diarrhea, and generalized macular erythematous, blanching, nonpruritic rash, as seen in the infant in the vignette, are common findings in many conditions, including viral infections and medication reactions. However, in the context of a recent blood product (packed red blood cells [PRBCs]) transfusion, transfusion-associated graft-vs-host disease (TA-GVHD) is this infant’s most likely diagnosis.

PREP Pearls
Transfusion-associated graft-vs-host disease has a high mortality rate.
Irradiation of blood products administered to an immunodeficient individual is paramount in preventing transfusion-associated graft-vs-host disease.

88
Q

A 17-year-old adolescent boy is evaluated for burning with urination for the past 3 days. He has had 3 lifetime sexual partners and is interested in women only. His last sexual encounter was 3 weeks ago when he had vaginal and oral sex without a condom. He has never been tested for sexually transmitted infections. His physical examination findings are normal except for a scant amount of mucoid urethral discharge. Urine and oral swabs are obtained for nucleic acid amplification testing (NAAT) for Neisseria gonorrhoeae and Chlamydia trachomatis.

Of the following, the BEST next step in this adolescent’s management is to

A.	have him follow-up for treatment in 2 days when the test results are available
B.	instruct him to notify all of his sexual partners from the past year
C.	provide presumptive antibiotic treatment for gonorrhea and chlamydia
D.	send a urinalysis and urine culture
A

The adolescent in the vignette has symptoms and signs of urethral inflammation including dysuria and urethral mucoid discharge. After obtaining urine and oral swabs for nucleic acid amplification testing (NAAT) for Chlamydia trachomatis and Neisseria gonorrhoeae, the best next step in management is to treat him with an antibiotic regimen effective against chlamydia and gonorrhea (eg, doxycycline, orally, and ceftriaxone, intramuscularly). Treatment should begin right away; it should not be held until test results are available.

PREP Pearls
Urethritis is commonly caused by sexually transmitted infections including gonorrhea and chlamydia. Mycoplasma genitalium causes almost 25% of nongonococcal urethritis cases in the United States.
Health care professionals should treat individuals with objective signs of urethral inflammation with antibiotics effective against gonorrhea and chlamydia, even before test results are available.

89
Q

A 14-year-old adolescent boy is seen for concerns about a worsening chest deformity. He is self-conscious about his appearance and has experienced difficulty exercising. He has no other known health problems. On physical examination, his oxygen saturation in room air is 100% by pulse oximetry. There is a moderately severe depression of his sternum with some molding of the anterior ribs (Item Q139). Breath sounds are equal with no stridor or wheeze; there is good air exchange. Cardiac sounds are displaced slightly to the left. The remainder of the adolescent’s physical examination findings are normal.

A chest radiograph shows a long, narrow thoracic cavity with the heart slightly displaced to the left on anterior-posterior view and the sternum displaced posteriorly on the lateral view. Complete lung function testing including spirometry and lung volumes is normal for age and height.

Of the following, the BEST test to evaluate the impact of this adolescent’s condition on his respiratory function is

A.	cardiopulmonary exercise testing
B.	computed tomography scan of the chest
C.	echocardiography
A

The adolescent in the vignette has a pectus excavatum chest deformity that is causing him concern about his appearance and exercise capacity. The most common chest wall deformity, pectus excavatum often has a significant negative impact on body image and perceived exercise capacity. Static pulmonary function testing results are rarely abnormal, as is the case for the boy in the vignette. Cardiopulmonary exercise testing may uncover subtle deficits in exercise capacity and is the best test to further evaluate his respiratory function.
A lung ventilation-perfusion scan is not indicated for the adolescent in the vignette. Chest wall deformities do not cause ventilation-perfusion mismatch.
PREP Pearls
Severe chest wall deformities can cause restrictive pulmonary physiology. The severity of the restrictive lung disease is directly related to the complexity of the chest deformity.
Pectus excavatum can have a significant negative impact on self-image and perceived exercise capacity; static pulmonary function testing is usually normal.
Surgical repair of scoliosis is not likely to improve lung function but can prevent it from worsening.

90
Q

A 1-month-old infant is seen for a health supervision visit. His mother is concerned about his head shape. The infant was born at term after an unremarkable pregnancy and delivery. He has been well and is meeting age-appropriate developmental milestones. His family history is unremarkable. The infant’s growth parameters are normal, including head circumference. His physical examination findings are significant only for the head shape shown in Item Q140.
bathrocephaly

Of the following, the BEST next management step for this infant is

A.	helmet therapy B.	neurosurgery referral
C.	physical therapy
D.	reassurance
A

The infant in the vignette has bathrocephaly resulting from a persistent mendosal suture (Item C140A). This suture usually disappears by the time of birth but when persistent can cause a prominent occiput. Bathrocephaly usually resolves without intervention, so the infant’s mother can be reassured that her son’s head shape will likely normalize. He does not require helmet therapy, referral to neurosurgery, or physical therapy for correction.

PREP Pearls
Bathrocephaly, which is caused by a persistent mendosal suture and results in a prominent occiput, usually resolves without intervention.
Early identification of concerning head shape is crucial so that infants receive treatment before the window of opportunity closes.
Craniosynostosis is a rare cause of microcephaly.

91
Q

A 15-year-old adolescent boy is brought to the office for a health supervision visit. His maternal grandfather received a kidney transplant secondary to autosomal dominant polycystic kidney disease at age 58 years. One year ago, the adolescent underwent renal ultrasonography screening that showed multiple bilateral cysts, the largest measuring 2 cm in the left kidney and 1.5 cm in the right kidney. The boy and his parents have no concerns today. He is doing well in school. He has not had headaches, dizziness, flank pain, swelling in any parts of his body, increased urinary frequency, or dysuria.

The adolescent’s weight is at the 25th percentile, height is at the 40th percentile, and blood pressure is 120/72 mm Hg. The remainder of his physical examination findings are unremarkable.

Of the following, the BEST next step in this adolescent’s evaluation is to perform

A.	computed tomography of the brain
B.	Doppler ultrasonography of the liver
C.	genetic testing
D.	urinalysis with microscopy
A

The adolescent in the vignette most likely has autosomal dominant polycystic kidney disease (ADPKD). He has multiple cysts visualized on renal ultrasonography and a family history of ADPKD. While he remains asymptomatic, he should be screened with annual blood pressure measurements (to monitor for hypertension) and urinalysis with microscopy (to detect proteinuria and hematuria). Annual blood pressure measurement and urinalysis should be performed for all asymptomatic children with a family history of ADPKD. Serum creatinine values are usually normal in children with ADPKD and kidney function does not typically worsen until adulthood.

A computed tomography scan of the brain to screen for cerebral aneurysm is not recommended for asymptomatic children since aneurysmal rupture is extremely rare. Doppler ultrasonography of the liver is helpful in diagnosing portal hypertension secondary to congenital hepatic fibrosis, which is associated with ARPKD. However, hepatic fibrosis is not associated with ADPKD and therefore, liver ultrasonography would not be indicated for the child in the vignette.

PREP Pearls
Autosomal dominant polycystic kidney disease is the most common inherited kidney disease.
Most children with autosomal dominant polycystic kidney disease are asymptomatic and diagnosed incidentally on an imaging study or during screening due to family history.
Annual blood pressure measurement and urinalysis (to monitor for proteinuria and hematuria) should be performed for asymptomatic children with a family history of autosomal dominant polycystic kidney disease.

92
Q

Question 142
A 14-day-old, low-birth-weight neonate undergoing parenteral antibiotic therapy for bacterial sepsis in the neonatal intensive care unit becomes clinically unstable.

Laboratory data are shown:

Laboratory Test

Result

White blood cell count

18,000/µL (18.0 × 109/L)

Hemoglobin

8.7 g/dL (87 g/L)

Hematocrit

25.8%

Platelet count

339 × 103/µL (339 × 109/L)

Sodium

143 mEq/L (143 mmol/L)

Potassium

4.8 mEq/L (4.8 mmol/L)

Chloride

108 mEq/L (108 mmol/L)

Carbon dioxide

23 mEq/L (23 mmol/L)

Blood urea nitrogen

4.0 mg/dL (1.4 mmol/L)

Creatinine

<0.2 mg/dL (<17.9 µmol/L)

Glucose

174 mg/dL (9.7 mmol/L)

Gram stain of the neonate’s positive blood culture reveals fungi concerning for Candida albicans.

Parenteral antifungal therapy is started. Twenty-four hours after initiation of antifungal therapy, the following laboratory data are obtained:

Laboratory Test

Result

White blood cell count

13,000/µL (13.0 × 109/L)

Hemoglobin

8.0 g/dL (80 g/L)

Hematocrit

24.2%

Platelet count

275 × 103/µL (275 × 109/L)

Sodium

136 mEq/L (136 mmol/L)

Potassium

2.6 mEq/L (2.6 mmol/L)

Chloride

105 mEq/L (105 mmol/L)

Carbon dioxide

18 mEq/L (18 mmol/L)

Blood urea nitrogen

25 mg/dL (8.9 mmol/L)

Creatinine

0.7 mg/dL (61.9 µmol/L)

Glucose

99 mg/dL (5.5 mmol/L)

Of the following, the MOST likely antifungal agent that was started is

A.	amphotericin B
B.	fluconazole
C.	flucytosine
D.	micafungin
A

The neonate in the vignette developed fungemia (candidemia) requiring parenteral antifungal therapy. Within 24 hours after starting treatment the neonate appears to have significant nephrotoxicity with acute kidney injury as evidenced by elevated serum blood urea nitrogen and creatinine levels and hypokalemia. Of the response choices, amphotericin B is most likely to cause acute and significant nephrotoxicity. Fluconazole, flucytosine, and micafungin are not associated with kidney injury.

PREP Pearls
The primary toxic side effect of amphotericin B, including liposomal preparations, is nephrotoxicity.
Antifungals may be associated with hepatotoxicity or inhibition of the cytochrome p450 enzyme system, which can result in elevated levels of other medications cleared by the liver.
With the exception of flucytosine, therapeutic drug level monitoring is rarely necessary when prescribing antifungal therapy.

93
Q

A 4-year-old boy with a noncontributory medical history is brought to the emergency department. He awoke around 3 AM crying and complaining of severe abdominal pain. The pain persisted throughout the night. This morning he continues to complain, and his oral intake has been poor. When asked where he hurts, the boy points to the periumbilical region. He has not had any emesis, fever, constipation, diarrhea, rash, joint pain, cough, shortness of breath, sore throat, testicular pain, or recent trauma.

The boy has a temperature of 36.7°C, heart rate of 106 beats/min, respiratory rate of 22 breaths/min, and blood pressure of 109/72 mm Hg. He appears uncomfortable, lying in his mother’s arms and groaning. His abdomen is tender to palpation in the periumbilical area without rebound tenderness or guarding. The remainder of his physical examination findings are unremarkable.

Findings on left lateral decubitus abdominal radiography are shown in Item Q143.

Of the following, the BEST next step in this boy’s management is

A.	admission to the hospital for serial abdominal examinations
B.	close outpatient follow-up with serial abdominal radiography
C.	emergency referral for endoscopic management
D.	emergency referral for surgical management
A

The boy in the vignette’s abdominal radiograph shows 3 circular radiopaque foreign bodies that appear to be touching, raising concern that the boy ingested magnets. Because the boy is symptomatic with multiple magnets that have passed through the stomach, the best next step in management is emergency referral for surgical removal. The algorithm for magnet ingestion in children is shown in Item C143. If the magnets were located within the esophagus or stomach, emergency referral for endoscopic removal would be the appropriate next step. Neither close outpatient follow-up with serial abdominal radiography or admission to the hospital for serial abdominal examinations would be appropriate for this child. With ingestion of multiple magnets, he is at risk for bowel injury or perforation if the magnets are not removed promptly.

94
Q

n 8-month-old female infant is being evaluated in the emergency department for a progressive rash and fever. The infant was seen in an urgent care center 2 weeks ago, diagnosed with gingivostomatitis, and treated with supportive care. The oral lesions resolved over 1 week, and she was doing well until 2 days ago when she developed a low-grade fever and a rash that started on her trunk and spread to her extremities. The infant has a temperature of 38.7°C, a heart rate of 120 beats/min, and a respiratory rate of 30 breaths/min. She appears alert, is not in acute distress, and appears nontoxic. There are multiple annular plaques with dusky centers on her trunk, extensor aspects of all extremities, and palms (Item Q144). There is no conjunctival injection or any oral lesions.

Of the following, the MOST accurate statement regarding this infant’s rash is that it is

A.	associated with arthritis
B.	characterized by intense pruritus
C.	commonly triggered by infections
D.	expected to resolve in 24 hours
A

The infant in the vignette has an annular rash with dusky center following a herpes simplex virus (HSV) gingivostomatitis infection that is suggestive of erythema multiforme (EM). Erythema multiforme is commonly triggered by infections. It is not associated with arthritis; any associated pruritus is mild, not intense; and erythema multiforme is expected to resolve in 2 to 3 weeks, not 24 hours.

Erythema multiforme is an immune-mediated, self-limiting disorder involving the skin and mucous membranes. The EM rash is described as annular targetoid plaques with 3 zones: a dusky center, surrounding pallor, and a rim of erythema. The centers of the lesions can become bullous. The lesions symmetrically involve the extremities, trunk, palms, and soles. Individual lesions last for 1 to 2 weeks; transient lesions are more consistent with urticaria. Pruritus, if present, is usually mild, and systemic symptoms such as fever and myalgias may occur especially in those with mucous membrane lesions.

PREP Pearls
Erythema multiforme is an immune-mediated condition characterized by targetoid lesions on the trunk, extremities, palms, and soles.
Infections, especially herpes simplex virus, are a common trigger for erythema multiforme.
Erythema multiforme is self-limiting and resolves with supportive care within 2 to 3 weeks.

95
Q

A 6-month-old infant born at 29 weeks’ gestation is seen in the neonatal intensive care unit for management of feeding difficulties. She has had a tracheoplasty for complete tracheal rings and has chronic lung disease. The infant reached full enteral feedings 5 weeks ago and has been slowly improving her oral intake. She currently feeds every 3 hours, taking one-third of her caloric requirements by bottle and the remaining administered in bolus feedings through a nasogastric tube. She has not made any progress with oral feedings in the past month despite intensive intervention by the feeding team. The team anticipates that it will take several months before the infant is able to take full caloric requirements by mouth. The medical team and infant’s family, through shared decision-making, discuss plans for discharge and ongoing enteral nutrition support. The infant’s family would like to continue with nasogastric feedings and work on improving oral intake at home before considering placement of a gastrostomy tube. The medical team, however, recommends gastrostomy tube placement.

Of the following, the factor MOST supportive of the medical team’s recommendation is the

A.	anticipated duration of nasogastric feedings
B.	decreased risk for emergency department visits
C.	history of prematurity
D.	history of previous tracheoplasty
A

The infant in the vignette currently requires enteral nutrition support by nasogastric (NG) tube due to inadequate oral intake despite appropriate feeding therapy. Because the medical team anticipates that this level of support will be required for several months, gastrostomy tube (G-tube) placement is indicated. Gastrostomy tube placement is generally recommended when enteral supplementation will be needed for a long period, with recommendations varying from at least 6 weeks to 3 months.

The infant’s history of prematurity and previous tracheoplasty do not support a preference for NG or G-tube feeding. Nasogastric feeding is associated with fewer tube-related emergency department visits compared with G-tube feeding, which would be a factor supporting the family’s preference.

96
Q

A 4-year-old boy (Item Q147) is seen for a health supervision visit. His mother reports reduced sweating compared to his peers. On physical examination, the boy has sparse scalp hair, dry skin, and a total of 2 teeth which are conical in shape. The family history is significant for oligodontia in the mother and the similarity of the boy’s physical features to those of his maternal grandfather.

Of the following, the MOST appropriate recommendation for this child at this visit is

A.	avoidance of overheating
B.	placement of dentures by age 20 years
C.	use of methotrexate to treat alopecia
D.	use of prophylactic antibiotics to prevent recurrent sinus infections
A

The boy in the vignette has a type of ectodermal dysplasia (ED). Children with ED are at high risk for hyperthermia and hyperpyrexia because of their decreased ability to sweat. Anticipatory guidance regarding avoidance of overheating is critically important.

PREP Pearls
Ectodermal dysplasias are a group of disorders that involve abnormalities in at least 2 ectodermal structures—hair, teeth, nails, or sweat glands.
The main clinical features of hypohidrotic ectodermal dysplasia include hypotrichosis, hypodontia, and hypohidrosis.
A critical feature of the management of hypohidrosis is avoidance of overheating.

97
Q

A 6-year-old girl is brought to the office for breathing problems. She has had repeated episodes of wheezing which occurred only with viral respiratory infections. Over the past 2 months she has been coughing at night 2 to 3 times weekly, frequently becomes short of breath while running, and must stop and rest while playing with friends. She required a course of oral corticosteroids for wheezing with a viral respiratory illness during the most recent winter season. Her mother and a paternal aunt had asthma as children. The girl’s vital signs are normal. There is no increased work of breathing or use of accessory muscles. Breath sounds are clear and equal to auscultation with good aeration. The remainder of her physical examination findings are normal.

Of the following, the BEST treatment for this girl is a/an

A.	combined inhaled steroid and formoterol inhaler used daily and as needed
B.	medium-dose inhaled corticosteroid as needed
C.	oral leukotriene modifier daily
D.	short-acting bronchodilator inhaler as needed
A

The girl in the vignette has mild to moderate persistent asthma, with symptoms occurring more than twice per week but less than daily. The preferred management for persistent asthma in children 5 years of age and older is daily and as-needed use of a combined inhaler with corticosteroid and formoterol.

Combined inhalers containing an inhaled corticosteroid and formoterol (a long-acting β-agonist [LABA]) are appropriate for what is known as the same maintenance and rescue therapy (SMART) approach to asthma management. Formoterol combined with budesonide used with the SMART approach has been well studied in the United States. This combination became the standard of care with the publication of the 2020 National Institutes of Health (NIH) updated asthma treatment guidelines. For children aged 5 years and older with persistent mild or moderate asthma, using an inhaled corticosteroid (ICS) with the long-acting β-agonist (LABA) formoterol as both maintenance and rescue therapy helps to minimize daily steroid exposure while decreasing the likelihood that oral steroids will be needed with exacerbation. The LABA salmeterol has a slower onset of action and is not appropriate for use as a rescue treatment. Item C148A, Item C148B, and Item C148C outline current treatment recommendations for children with asthma.

PREP Pearls
The most effective treatment for asthma of all severities is an inhaled corticosteroid.
Leukotriene modifiers may be used as alternative first-line therapy in young children with mild persistent asthma. A US Food and Drug Administration boxed warning was added to the prescribing information for montelukast regarding the risk for neuropsychiatric adverse effects.
Combined inhaled corticosteroids with formoterol are appropriate for the same maintenance and rescue therapy in children and adolescents aged 5 years and older.
Long-acting ꞵ-agonists should be used to treat asthma only in conjunction with an inhaled corticosteroid.

98
Q

A 1-year-old girl with moderate atopic dermatitis is seen for a health supervision visit. Her father has been using a low-potency topical steroid sporadically to manage her atopic dermatitis with minimal effect. He does not want to increase the frequency or strength of the topical steroid because he has concerns regarding side effects. He would like to explore dietary modifications to manage her atopic dermatitis. The girl eats a wide variety of foods with no obvious correlation between flares of her skin condition and specific foods. She has been otherwise healthy with normal growth and development. Her physical examination findings are normal except for diffuse patches of moderate atopic dermatitis.

Of the following, the BEST management strategy for this girl’s condition is to

A.	begin vitamin D supplementation
B.	optimize medical management
C.	order food allergy testing
D.	recommend a food elimination diet
A

The best management strategy for the girl in the vignette is optimization of medical treatment of her atopic dermatitis. Without an obvious correlation between flares of her atopic dermatitis and specific foods, a food elimination diet is not indicated, and food allergy testing in this situation has a high false-positive rate. Current evidence does not support the effectiveness of vitamin supplementation in the management of atopic dermatitis. It will be important to share this information with the girl’s father, and respond respectfully to his questions and concerns to engage him willingly in effective skin care management.

PREP Pearls
First-line treatment of atopic dermatitis is to optimize medical management before considering dietary modifications.
Management of atopic dermatitis must include family education that emphasizes the chronic nature of the condition and the importance of maintenance therapy.
Complications of atopic dermatitis include atopic, infectious, and behavioral conditions.

99
Q

A 6-year-old boy is seen for a routine health supervision visit. He has a history of an atrioventricular septal defect repair that resulted in stenosis of his left atrioventricular valve. The stenotic valve was replaced with a mechanical valve. He is maintained on anticoagulation, and his international normalized ratio has been in the appropriate treatment range. He has no known drug allergies and takes oral medications without difficulty. The boy’s physical examination findings are normal except for mechanical cardiac sounds. He is scheduled to see his dentist next week for a routine cleaning for which his mother requests a prescription for antibiotic prophylaxis.

Of the following, the BEST medication to prescribe for this boy is

A.	amoxicillin
B.	clindamycin
C.	nothing; none are indicated
D.	trimethoprim-sulfamethoxazole
A

The boy in the vignette has a mechanical heart valve, an indication for endocarditis prophylaxis with antibiotics prior to routine dental cleaning. The regimen recommended by the American Heart Association 2007 guideline is amoxicillin, orally, in a single dose 30 to 60 minutes prior to the procedure. Amoxicillin is the drug of choice because it is well absorbed by the gastrointestinal (GI) tract and has high and sustained serum levels. Item C151 lists alternative antibiotic options for those with a penicillin allergy or inability to take medication orally. The targeted organisms are viridans group streptococci. Clindamycin is an appropriate alternative for those with a penicillin allergy, which the boy in the vignette does not have. Trimethoprim-sulfamethoxazole is not indicated for endocarditis prophylaxis.

Not all children with congenital heart disease require endocarditis prophylaxis. Prophylaxis is indicated for children with conditions that confer a high risk of an adverse outcome from infective endocarditis. These conditions include:

A prosthetic cardiac valve or prosthetic material used in cardiac valve repair
A previous history of endocarditis
Specific congenital heart disease (CHD)

PREP Pearls
Endocarditis prophylaxis is required for children with specific cardiac risk factors (eg, prior history of endocarditis, residual defects, cyanotic disease, or prosthetic material that is not endothelialized).
Specific dental procedures and those causing respiratory mucosal injury are the most common indications for endocarditis prophylaxis.
Amoxicillin is the antibiotic of choice for endocarditis prophylaxis, administered as a single dose 30 to 60 minutes prior to the procedure.

100
Q

A 16-year-old adolescent boy is seen for a health supervision visit. On routine adolescent screening, he reports that he has been sexually active with 2 males for 9 months and does not use condoms consistently. His last high-risk sexual contact was 4 weeks ago. He denies sick contacts or injection drug use. His physical examination findings are normal. He is counseled on reducing his risk for sexually transmitted infection (STI) and offered routine STI testing including HIV.

Of the following, the BEST HIV test to order for this adolescent is

A.	DNA polymerase chain reaction
B.	HIV-1/HIV-2 antibody immunoassay
C.	HIV-1/HIV-2 antigen/antibody combination assay
D.	RNA polymerase chain reaction
A

The best HIV test to order for the adolescent in the vignette is the HIV-1/HIV-2 antigen/antibody combination assay, a fourth-generation immunoassay recommended by the Centers for Disease Control and Prevention (CDC) as the preferred initial screening test. This combination test detects HIV viral protein (p24 antigen) as well as HIV IgM and IgG antibodies to HIV-1 and HIV-2. The test is highly sensitive and specific in identifying individuals in the early stages of HIV infection. The p24 antigen is detectable between weeks 2 and 3 of infection. A non-reactive test result is reported as negative, and no additional testing is recommended unless early HIV infection (acute retroviral syndrome) is suspected. A reactive test is followed up with a supplemental HIV-1/HIV-2 antibody differentiation assay.

PREP Pearls
The United States Centers for Disease Control and Prevention recommends routine HIV screening at least once for individuals aged 13 to 64 years.
Annual HIV screening is recommended for individuals who are at high risk for HIV acquisition.
The preferred initial HIV screening test recommended by the Centers for Disease Control and Prevention is the fourth-generation HIV-1/HIV-2 antigen-antibody combination immunoassay performed on serum/plasma.

101
Q

A 7-year-old girl is seen for a health supervision visit. Results of urine dipstick testing, performed on the previous 2 visits for a suspicion of urinary tract infection, were positive for blood and negative for protein, and urine microscopy revealed 10 to 20 red blood cells per HPF. She has no recent symptoms of dysuria, urgency, frequency, nocturnal enuresis, abdominal pain, rashes, or joint pain. There is no family history of kidney disease or deafness. The girl’s weight is at the 50th percentile and height is at the 65th percentile for age. Her blood pressure is 100/60 mm Hg. The remainder of the physical examination findings are normal.

Results of today’s urinalysis with microscopy are shown:

Test

Result

Appearance

Yellow

Specific gravity

1.015

Leukocyte esterase

Negative

Nitrite

Negative

Blood

Positive

Protein

Negative

Red blood cells per high-power field

20-50

White blood cells per high-power field

<5

Of the following, the BEST next step in this girl’s evaluation is

A.	24-hour urine protein excretion
B.	computed tomography scan of the abdomen
C.	genetic testing
D.	spot urine calcium:creatinine ratio
A

The girl in the vignette has persistent asymptomatic microscopic hematuria. The most appropriate next step in her evaluation is to order a spot urine calcium:creatinine ratio to evaluate for hypercalciuria. Hypercalciuria, defined as a urine calcium:creatinine ratio of greater than 0.2 mg/mg or calcium excretion of greater than 4 mg/kg daily on 24-hour urine collection, is a risk factor for nephrolithiasis.

PREP Pearls
Common causes of persistent microscopic hematuria include hypercalciuria, thin basement membrane disease, IgA nephropathy, and sickle cell disease.
Initial laboratory evaluation of persistent microscopic hematuria should include urine protein:creatinine ratio, urine calcium:creatinine ratio, urine culture, serum electrolyte levels, renal function tests (blood urea nitrogen and serum creatinine measurement), and hemoglobin electrophoresis (if sickle cell disease is suspected).
Children with microscopic hematuria associated with edema, hypertension, or proteinuria should be promptly referred to a pediatric nephrologist.

102
Q

A 1-month-old infant is seen for a health supervision visit. Her parents are concerned that she breathes faster while eating. She takes 1 to 2 ounces of standard infant formula every 2 to 3 hours and is sleeping well between feedings. The infant was born at term. There were no complications during pregnancy or the delivery, and she was discharged from the hospital 24 hours after birth. She has had consistent growth along the 50th percentile for weight and length. On physical examination, she has a heart rate of 150 beats/min, respiratory rate of 60 breaths/min, holosystolic murmur at the lower left sternal border, and liver edge palpable 3 cm below the right costal margin. The remainder of her physical examination findings are normal.

Of the following, this infant’s MOST likely diagnosis is

A.	aortic stenosis
B.	atrial septal defect
C.	pulmonary stenosis
D.	ventricular septal defect
A

The infant in the vignette has tachypnea while feeding, hepatomegaly, and a holosystolic murmur at the lower left sternal border. These findings are consistent with a ventricular septal defect (VSD). The flow across an atrial septal defect (ASD) itself does not create a murmur, though the increased flow can result in a fixed split S2 and produce a murmur across the pulmonary valve, which would be heard in mid-systole at the left upper sternal border. Pulmonary valve stenosis would produce a systolic ejection murmur in the pulmonic location. An aortic stenosis murmur would be heard in mid-systole, loudest at the right upper sternal border.

PREP Pearls
A ventricular septal defect creates a holosystolic murmur heard best at the left lower sternal border.
Symptoms and signs of a moderate to large ventricular septal defect include tachypnea, sweating while feeding, tachycardia, and hepatomegaly.
A moderate to large ventricular septal defect requires surgical closure and if left open can lead to irreversible pulmonary hypertension (Eisenmenger syndrome)

103
Q

A 16-year-old adolescent girl is seen during the winter for concerns about recurrent episodes of blue hands and feet. These episodes have occurred at various intervals for several months and seem to be related to cold exposure. There is no associated pain or dysfunction, and the events resolve spontaneously in a warm environment. She takes no medications and has no history of cardiovascular or respiratory disease. The adolescent is a ballet dancer and practices for 3 to 4 hours daily after school.

On physical examination, the adolescent is very thin, with little subcutaneous fat on her trunk. Her weight is 45 kg, 8 kg lower than it was 6 months ago, and height is 165 cm. Her temperature is 35.5°C, heart rate is 75 beats/min, respiratory rate is 12 breaths/min, and blood pressure is 100/60 mm Hg in the right arm, sitting; there is no change in blood pressure when supine, sitting, or standing. Oxygen saturation by pulse oximetry is 99% in room air in each hand and foot. The girl’s mucous membranes are pink, and there is no peripheral cyanosis. The remainder of her physical examination findings are normal. The girl’s weight loss is discussed, and plans are made to address this concern.

Of the following, the BEST next step in evaluating the concern for which the adolescent was brought to the office is to

A.	obtain a blood methemoglobin level
B.	obtain a serum antinuclear antibody level
C.	order a 72-hour Holter monitor study
D.	request pictures of the next episode
A

The girl in the vignette has acrocyanosis caused by thermodysregulation accompanying anorexia nervosa. An 8-kg weight loss in 6 months and low body temperature in an otherwise healthy athletic adolescent makes anorexia nervosa a likely diagnosis. Secondary acrocyanosis is a common finding in anorexia nervosa, occurring in up to 80% of affected individuals. Acrocyanosis is characteristically variable without clearly demarcated margins. Cold exposure is the most common trigger. Asking the girl to provide pictures of her extremities at the time of an episode can confirm the diagnosis. She can be reassured that the cyanosis itself is not threatening to her health; the focus of her care can then shift to treatment of her anorexia nervosa.

Methemoglobinemia causes hemoglobin to be oxidized to the ferric (Fe3+) state, which cannot carry oxygen to the peripheral tissues. This pathology results in persistent central cyanosis and low peripheral oxygen saturation. Serum methemoglobin levels above 10% are associated with cyanosis and levels above 20% are associated with dyspnea, tachycardia, and lightheadedness.

PREP Pearls
Primary acrocyanosis is common and benign. Up to 80% of patients with anorexia nervosa may have acrocyanosis.
Raynaud phenomenon is differentiated from acrocyanosis by sharp demarcation of cyanotic skin, pain or discomfort, and a biphasic or triphasic color response (pallor, cyanosis, plethora).
Secondary Raynaud phenomenon can be associated with collagen vascular disease and skin breakdown.

104
Q

A 10-year-old girl with a history of recurrent otitis media is seen for follow-up after a course of antibiotic ear drops for recurrent ear drainage and mild subjective hearing loss. She has had no dizziness, headaches, or fever. On physical examination, her vital signs are within the normal range. Findings on right ear otoscopic examination are shown in Item Q157. The remainder of her physical examination findings are normal.

Of the following, the BEST description of the finding noted on this girl’s right otoscopic examination is (a/the)

A.	cholesteatoma
B.	foreign body
C.	head of the stapes
D.	tympanosclerosis
A

The right ear finding for the girl in the vignette is most likely a cholesteatoma, associated with retraction of a portion of the tympanic membrane (TM). A foreign body is more likely to protrude into the ear canal. The head of the stapes is inferior to the location of this girl’s finding. Areas of tympanosclerosis are not usually retracted.

PREP Pearls
Cholesteatomas are collections of debris that form in retraction pockets of the tympanic membrane, most often secondary to chronic otitis media.
If left untreated, cholesteatomas may grow into the middle ear and erode the ossicles.
Cholesteatomas may recur after surgical removal, requiring lifelong monitoring.

105
Q

A 15-month-old boy born at term is brought to the office for evaluation of seizure-like events. His parents report episodes of shivering movements of his arms and upper body, lasting 5 to 10 seconds, occurring multiple times per day for the past few months. These events seem to happen randomly, although they may be more frequent at mealtimes. There is no loss of consciousness during the events, and they do not distress him or interrupt his activities. There is no significant medical or perinatal history, he is meeting developmental milestones as expected, and there has been no developmental regression. His physical examination findings are normal.

Of the following, the BEST next step in this boy’s evaluation is to obtain (a)

A.	brain magnetic resonance imaging
B.	routine electroencephalography
C.	serum thyroid-stimulating hormone level D.	video recording of a characteristic event
A

D

PREP Pearls
Benign shuddering attacks of infancy is a nonepileptic, self-limited condition of infancy and early childhood characterized by brief shivering movements of the head and upper extremities without alteration of consciousness. Episodes are more frequently noticed during activities such as mealtime, diaper changes, or excitement/frustration.
Distinguishing epileptic from nonepileptic conditions in infancy requires careful history with attention to the clinical features of the events, conditions in which they occur, and provoking factors. Video recording of events can be a useful tool for accurate diagnosis.
When a diagnosis of benign shuddering attacks is suspected, parental reassurance and education is a critical component of management. Investigative studies, such as neuroimaging, electroencephalography, or laboratory studies are not necessary.

106
Q

A 14-year-old adolescent boy is seen for a preparticipation physical examination. He would like to participate in high school volleyball, wrestling, and baseball. The boy has a history of vision impairment following an eye injury sustained as a young child. The corrected vision in his left eye is 20/60, and his uncorrected vision in the right eye is 20/20. Recommendations for participation and the need for protective eyewear are discussed with the boy and his family.

Of the following, it is MOST appropriate to clear this boy for participation in

A.	volleyball and baseball only
B.	volleyball only
C.	volleyball, wrestling, and baseball
D.	wrestling only
A

The adolescent in the vignette is considered to have only 1 functional eye because the corrected visual acuity in his left eye is less than 20/40. The American Academy of Pediatrics (AAP) and American Academy of Ophthalmology (AAO) recommend that athletes who are functionally 1-eyed wear protective eyewear for all sports. An eye injury resulting in vision loss in the unaffected eye puts these athletes at risk for significant disability. Because protective eyewear cannot be worn during wrestling, the boy in the vignette should not participate in this sport. Combat sports (eg, martial arts and boxing) are also contraindicated for athletes who are functionally 1-eyed.

107
Q

A 5-year-old boy is brought to the clinic for evaluation. He has had difficulty walking for the past 2 days. He has bilateral calf pain and prefers not to walk; when he does walk, he does so with a stiff-legged gait. Three days ago, the boy had a fever and cough, both of which have resolved. There is no history of trauma or initiation of a new physical activity. The boy’s physical examination findings are normal except for tenderness to palpation in both calves.

Of the following, the BEST next step in this boy’s evaluation is to order

A.	antinuclear antibody and double-stranded DNA testing
B.	antistreptolysin O testing
C.	a creatinine phosphokinase level
D.	a respiratory viral panel for influenza A/B
A

The boy in the vignette has the classic symptoms and signs of postviral myositis (benign acute childhood myositis [BACM]), including difficulty walking, calf pain, and calf tenderness after a viral illness. Of the response choices, the best next step in his evaluation is to obtain a creatinine phosphokinase (CPK) level. The CPK level is usually moderately elevated in patients with BACM. This test is useful for excluding other diagnoses and in monitoring recovery. In a well-appearing child with BACM, where close follow-up is feasible, laboratory testing may not be needed.

The most dangerous sequela of BACM is rhabdomyolysis, which can lead to myoglobin release and resulting renal failure. Rhabdomyolysis is very rare in children with BACM, and the risk of occurrence does not correlate with the CPK level. Some experts recommend that children with high CPK levels be monitored for dark-colored urine at home and, when noted, use urinalysis to identify myoglobinuria. A reasonable management approach when mild signs and symptoms are present, such as for the boy in the vignette, is observation without laboratory testing as long as follow-up is feasible and anticipatory guidance is provided to return if symptoms worsen or progress, weakness develops, or if dark-colored urine occurs.

PREP Pearls
Benign acute childhood myositis is an acute myositis that occurs after a viral infection, most often influenza.
A child with classic signs and symptoms of benign acute childhood myositis does not require an extensive evaluation. A creatinine phosphokinase level is usually elevated, but rhabdomyolysis is rare.
Rheumatological conditions (eg, polymyositis, dermatomyositis, systemic lupus erythematosus, juvenile idiopathic arthritis), and muscular dystrophies should be considered in a child with chronic progressive muscle pain and weakness.

108
Q

A 5-year-old boy is brought to the clinic for evaluation. He has had difficulty walking for the past 2 days. He has bilateral calf pain and prefers not to walk; when he does walk, he does so with a stiff-legged gait. Three days ago, the boy had a fever and cough, both of which have resolved. There is no history of trauma or initiation of a new physical activity. The boy’s physical examination findings are normal except for tenderness to palpation in both calves.

Of the following, the BEST next step in this boy’s evaluation is to order

A.	antinuclear antibody and double-stranded DNA testing
B.	antistreptolysin O testing
C.	a creatinine phosphokinase level
D.	a respiratory viral panel for influenza A/B
A

The boy in the vignette has the classic symptoms and signs of postviral myositis (benign acute childhood myositis [BACM]), including difficulty walking, calf pain, and calf tenderness after a viral illness. Of the response choices, the best next step in his evaluation is to obtain a creatinine phosphokinase (CPK) level. The CPK level is usually moderately elevated in patients with BACM. This test is useful for excluding other diagnoses and in monitoring recovery. In a well-appearing child with BACM, where close follow-up is feasible, laboratory testing may not be needed.

The most dangerous sequela of BACM is rhabdomyolysis, which can lead to myoglobin release and resulting renal failure. Rhabdomyolysis is very rare in children with BACM, and the risk of occurrence does not correlate with the CPK level. Some experts recommend that children with high CPK levels be monitored for dark-colored urine at home and, when noted, use urinalysis to identify myoglobinuria. A reasonable management approach when mild signs and symptoms are present, such as for the boy in the vignette, is observation without laboratory testing as long as follow-up is feasible and anticipatory guidance is provided to return if symptoms worsen or progress, weakness develops, or if dark-colored urine occurs.

PREP Pearls
Benign acute childhood myositis is an acute myositis that occurs after a viral infection, most often influenza.
A child with classic signs and symptoms of benign acute childhood myositis does not require an extensive evaluation. A creatinine phosphokinase level is usually elevated, but rhabdomyolysis is rare.
Rheumatological conditions (eg, polymyositis, dermatomyositis, systemic lupus erythematosus, juvenile idiopathic arthritis), and muscular dystrophies should be considered in a child with chronic progressive muscle pain and weakness.

109
Q

A 2-year-old boy is evaluated in the emergency department for 5 days of fever, rash, and red eyes. He was seen at an urgent care center 2 days ago, where he was given amoxicillin; there has been no decrease in fever. Similar symptoms are present in several children attending his toddler swim class. There has been no SARS-CoV-2 infection documented among his contacts in the past 6 weeks. On physical examination, he has dry, cracked lips, and dry mucous membranes. His conjunctivae are injected bilaterally. He has tender preauricular lymph nodes, pharyngeal erythema, and tonsillar exudates.

Results of laboratory testing performed at the urgent care center are shown:

Laboratory Test

Result

White blood cell

12,000/μL (12.0 × 109/L)

  Neutrophils

25%

  Lymphocytes

55%

  Monocytes

18%

  Eosinophils

2%

Hemoglobin

12 g/dL (120 g/L)

Platelet count

150 × 103/μL (150 × 109/L)

Group A streptococcal antigen test

Negative

Group A Streptococcus culture

Pending

C-reactive protein

3.5 mg/dL (35 mg/L)

SARS-CoV-2 polymerase chain reaction

Not detected

Epstein–Barr virus VCA IgM

<17 U/mL

Epstein–Barr virus VCA IgG

> 150 U/mL

Epstein–Barr virus nuclear IgG

> 300 U/mL

Abbreviation: VCA, viral capsid antigen.

Of the following, the MOST appropriate test to confirm this boy’s diagnosis is

A.	adenovirus polymerase chain reaction
B.	Epstein-Barr virus polymerase chain reaction
C.	group A Streptococcus antibodies
D.	SARS-CoV-2 spike protein antibodies
A

The boy in the vignette has a clinical constellation of acute follicular conjunctivitis (Item C162A), tender preauricular lymphadenopathy, pharyngitis, and fever, which is most consistent with adenovirus pharyngoconjunctival fever. Adenoviral tonsillitis most often occurs in children younger than 3 years.

PREP Pearls
Adenovirus is responsible for a wide variety of infections in children, most commonly respiratory and gastrointestinal.
Adenovirus pharyngoconjunctival fever presents with follicular conjunctivitis, tender preauricular lymphadenopathy, pharyngitis, and fever.
Adenovirus is diagnosed by polymerase chain reaction performed on a nasopharyngeal swab sample; this test is included in the available respiratory multiplex viral panels.

110
Q

A 7-year-old girl is brought to the clinic for concerns about her balance. For the past few days, the girl has complained of the room spinning and nausea. She has 5 to 6 episodes per day, each lasting about 1 minute, during which she grabs onto a chair or the sofa. Between episodes she acts and feels normal. She is otherwise healthy with no history of trauma or recent viral symptoms. Her physical examination findings are unremarkable, including her tympanic membranes and a detailed neurologic examination.

Of the following, the BEST next step in this girl’s management is

A.	an antihistamine prescription
B.	head and neck imaging
C.	observation
D.	referral to neurology
A

The girl in the vignette most likely has benign paroxysmal vertigo of childhood (BPVC), given her intermittent symptoms and unremarkable physical examination findings. Benign paroxysmal vertigo of childhood is typically a self-limited condition for which observation is recommended. Diagnostic evaluation and referral to neurology are only recommended if symptoms worsen or there is impairment of function. Medication management with anti-emetics or antihistamines may be warranted if episodes are long lasting, which is not yet the case for the girl in the vignette. Although the cause of BPVC is unknown, there is usually a family history of migraine headaches, and children with BPVC may subsequently develop migraines.

PREP Pearls
Benign paroxysmal vertigo of childhood is the second most common cause of vertigo in children aged 2 to 12 years; the most common cause is middle ear disease.
The diagnosis of benign paroxysmal vertigo is based on a history of brief episodes of vertigo associated with vomiting, pallor, nystagmus, ataxia or fearfulness, normal neurologic examination findings, audiometric, and vestibular findings between episodes, symptom-free intervals, and no other attributable disorder.
Benign paroxysmal vertigo is a self-limited condition for which the treatment is observation. Referral to neurology for further evaluation and management may be considered for worsening symptoms or impairment of function.

111
Q

A 16-month-old girl is seen for a health supervision visit. She eats a wide variety of foods and drinks 40 oz of cow milk per day. On physical examination, she looks pale. Her vital signs are normal for age. The remainder of her physical examination findings are normal.

Screening laboratory evaluation reveals a hemoglobin level of 6.4 g/dL (64 g/L) and a mean corpuscular volume of 65 fL.

After review of the laboratory results, a more detailed history is obtained. The family recently renovated their house. The girl often puts items into her mouth (things found on the floor and paper items). She has not had bright red blood in her diaper or darker stools.

Of the following, the MOST likely additional laboratory findings for this girl include

Low or high:
Reticulocytes
RBC distribution width
Ferritin
soluble factor

A

The girl in the vignette has iron-deficiency anemia, the most common cause of anemia in her age group. Her high-volume consumption of cow milk, pica, and pallor support this diagnosis. Laboratory findings consistent with iron-deficiency anemia include a low hemoglobin (Hb) level, low mean corpuscular volume (MCV), low reticulocyte count, high red blood cell distribution width (RDW), high soluble transferrin receptor (sTfR) level, and low ferritin level (the storage form of iron).

Anemia of chronic disease or inflammation is associated with a low reticulocyte count, normal RDW, high ferritin level, and low sTfR level. Gastrointestinal blood loss should be considered in the differential diagnosis of any child with iron-deficiency anemia. There may be a poor response to oral iron supplementation if there is a gastrointestinal condition; evaluation by a gastroenterologist for an inflammatory bowel disorder may be indicated. Markers of inflammation that can be helpful in determining the cause of anemia include elevated ferritin level, erythrocyte sedimentation rate, C-reactive protein level, and platelet count.

112
Q

A neonate born at 31 weeks’ gestation with a birth weight of 1,500 g is admitted to the neonatal intensive care unit. The neonate currently requires continuous positive pressure support of 5 cm H2O and a fraction of inspired oxygen of 25%. To provide ease of access for care, the newborn is placed under a radiant warmer.

Of the following, the MOST likely effect of this care environment on the neonate will be

A.	decreased insensible water loss
B.	excess weight gain
C.	hyperthermia
D.	hypoglycemia
A

The most likely effect of care for a neonate under a radiant warmer is hyperthermia. Radiant warmers, while allowing for easy access to neonates, are associated with increased insensible water losses and, if not used properly, can result in significant deleterious effects.

PREP Pearls
A neutral thermal environment is important for care of newborns.
Premature neonates should ideally be cared for in a double-walled incubator.
If a radiant warmer is used, care must be taken to ensure that the sensor probe is properly placed and the warmer is placed in automatic mode; careful monitoring of the neonate’s temperature is required.

113
Q

A 3-year-old boy is seen in the office to establish care after emigrating from Southeast Asia 2 months ago. He has had a cough for 12 months and has been eating less for the past 6 months. He has a habit of soil pica. The family has 4 dogs and 5 cats. The boy’s physical examination findings are remarkable for scattered wheezing in both lung fields and a liver edge palpable 2 cm below the right costal margin. His weight is less than the third percentile. His parents report that his weight is usually at the 25th percentile.

Laboratory data are shown:

Laboratory Test

Result

White blood cell count

15,000/μL (15.0 × 109/L)

Neutrophils

45%

Lymphocytes

30%

Monocytes

5%

Eosinophils

20%

Hemoglobin

10 g/dL (100 g/L)

Platelet count

200 × 103/μL (200 × 109/L)

Alanine aminotransferase

55 U/L

Aspartate aminotransferase

65 U/L

Of the following, the BEST next step to confirm this boy’s diagnosis is to obtain

A.	serum for Ascaris IgG
B.	serum for Toxocara IgG
C.	stool for bacterial culture
D.	stool for ova and parasites
A

The boy in the vignette has toxocariasis, a tissue roundworm infection. His signs and symptoms (wheezing, hepatomegaly, anorexia, and weight loss), exposure to dogs and cats, habit of soil pica, and eosinophilia are consistent with infection with the visceral larva migrans form of Toxocara. In Southeast Asia, the seroprevalence of toxocariasis is approximately 34%. In the United States, the seroprevalence is 5%.

PREP Pearls
Toxocariasis is a parasitic infection caused by 2 species of roundworm (nematode): Toxocara canis and Toxocara cati, parasites of dogs (puppies) and cats (kittens), respectively.
Toxocara infection causes disease when larvae migrate to tissues and local immune reactions result in injury. Major symptomatic presentations are visceral larva migrans (young children) and ocular larva migrans (older children and adolescents).
The diagnosis of Toxocara is confirmed by Toxocara IgG serologic testing. Stool for ova and parasites are not helpful because the adult worm does not complete its life cycle in the human gastrointestinal tract.

114
Q

A 2-hour-old neonate born at 36 4/7 weeks’ gestation is examined in the newborn nursery. She was born to a 28-year-old gravida 2, para 1 woman in a normal spontaneous vaginal delivery. Maternal prenatal testing is negative for group B Streptococcus, and rupture of membranes occurred 20 hours before delivery. The mother had a temperature of 39.6°C measured 8 hours before delivery and fetal tachycardia was noted at that time. The neonate has a temperature of 37.1°C, heart rate of 130 beats/min, and respiratory rate of 48 breaths/min. She appears vigorous with a good cry and normal tone. The remainder of her physical examination findings are normal. The mother requests early discharge.

Of the following, the BEST next management step for this neonate is to

A.	consider early discharge if the neonate's vital signs are stable with next day follow-up
B.	observe closely for signs of sepsis for at least 48 hours
C.	obtain a complete blood cell count and C-reactive protein level, and perform blood culture
D.	obtain a complete blood cell count, perform blood culture, and start empiric antibiotics
A

The neonate in the vignette is at risk for sepsis due to a combination of factors, late preterm gestation, maternal fever, and prolonged rupture of membranes (PROM, >18 hours). The best next step in management is to obtain a complete blood cell count, perform a blood culture, and start intravenous ampicillin and gentamicin treatment. Early discharge is not the best option because signs of sepsis may be delayed until 72 hours after birth. Close observation for 48 hours without empiric antibiotics would not be a good choice given this neonate’s multiple risk factors for sepsis. Although normal laboratory evaluation may seem reassuring, the white blood cell counts and inflammatory markers may not be a reliable indicator of sepsis in the first 24 to 48 hours after birth.

PREP Pearls
Risk factors for early-onset neonatal sepsis include prematurity, evidence of maternal chorioamnionitis, untreated maternal colonization with group B Streptococcus, and prolonged rupture of membranes.
Risk stratification algorithms may guide clinicians’ management of neonates at risk for sepsis and reduce the use of unnecessary antibiotics.
The use of intrapartum antibiotics has reduced the incidence of early-onset neonatal sepsis.

115
Q

A 27-hour-old neonate in the newborn nursery was noted to have a malformation of her right ear. She was born at 38 weeks’ gestation. The pregnancy was complicated by a history of maternal type diabetes that was not well controlled. The neonate’s physical examination findings are significant only for the appearance of her right ear (Item Q170). double ears

Of the following, the BEST next step in the evaluation of this neonate is

A.	an auditory brainstem response test
B.	a chromosomal microarray
C.	renal ultrasonography
D.	a temporal bone computed tomography scan
A

The neonate in the vignette has isolated right microtia, a malformation of the external ear. Microtia commonly affects both the external ear canal and middle ear, so hearing can be affected. Thus, the best next step is to evaluate this neonate’s hearing with an auditory brainstem response test. This test is recommended soon after birth to evaluate inner ear function.

PREP Pearls
Neonates with microtia, a malformation of the external ear, should have their hearing evaluated with an auditory brainstem response test soon after birth.
Microtia is usually an isolated congenital anomaly. Goldenhar and Treacher-Collins are the most common associated syndromes.
Treatment of microtia consists of hearing restoration and external structure reconstruction.

116
Q

A 4-week-old infant is seen for a health supervision visit. His parents are concerned about excessive crying. He sleeps well during the day, waking to feed every 3 hours, and generally has a good temperament, but most nights he cries for 3 to 4 hours even after being fed and changed. Afterward, he can be consoled and sleeps well. He is not gassy and does not spit up or arch his back. His urine and stool output is normal in appearance, quantity, and frequency. The infant was born full term via normal vaginal delivery. He has no significant medical history. His vital signs are normal for age, growth parameters are all at the 50th percentile with normal weight gain, and physical examination findings are normal. His parents ask when they can expect his symptoms to improve.

Of the following, the BEST response is that this should occur by age

A.	2 months
B.	4 months
C.	6 months
D.	8 months
A

The infant in the vignette most likely has colic. His parents can be reassured that it should improve by the time he is 4 months of age.

117
Q

A 7-year-old boy is evaluated in the outpatient clinic for a 1-year history of hematochezia and parental concern for a hemorrhoid. He has soft, formed stools daily with bright red blood on the outside of the stool. He is otherwise healthy. Physical examination reveals a well-appearing child with normal weight and height for age. His abdomen is soft and nontender. Rectal examination demonstrates a normal-appearing anus with a 1-cm, palpable, mobile, nontender mass in the rectum.

Colonoscopy is performed and 10 pedunculated polyps (0.5-1 cm in diameter) are noted to be scattered throughout the colon (Item Q174). All visualized polyps are successfully removed and retrieved; each polyp demonstrates pathology consistent with a juvenile polyp.

Of the following, the BEST next step in the management of this child is

A.	administration of a nonsteroidal anti-inflammatory drug
B.	referral for genetic testing
C.	referral to a pediatric colorectal surgeon
D.	repeat colonoscopy only if hematochezia recurs
A

The child in the vignette has clinical features concerning for juvenile polyposis syndrome (JPS), evidenced by the presence of 5 or more juvenile polyps in the colon. Because JPS is associated with an increased risk for colorectal cancer, testing and referral to genetics for counseling is recommended for the child and other potentially affected family members.

PREP Pearls
For a child with fewer than 5 juvenile colonic polyps, endoscopic removal is recommended; repeat colonoscopy is not indicated unless symptoms recur.
Children with 5 or more juvenile polyps should be referred for juvenile polyposis syndrome genetic testing.
Pediatric polyposis syndromes often require monitoring for extraintestinal malignancies; the involvement of genetic counselors or cancer risk teams is essential.

118
Q

An 18-month-old boy is brought to the clinic for a routine health supervision visit. He takes a few steps on his own but cannot walk without holding onto someone or something. He tries to eat with a spoon and scribbles. He points to show the physician something interesting, puts hands out to be washed, looks at a few pages in a book with the reader, and helps with dressing by pushing his arm through a sleeve. The boy tries to say 3 or more words besides “mama” and “dada” and follows 1-step directions without any gestures.

Of the following, this boy most likely has developmental DELAY in

A.	fine motor skills
B.	gross motor skills
C.	language-communication skills
D.	social-emotional skills
A

The boy in the vignette is 18 months old. At this age, expected gross motor skills include walking without holding onto anyone or anything and climbing on and off a couch or chair without help. As the boy can take a few steps but cannot walk without holding onto anyone or anything, he most likely has delayed gross motor skills. His skills in the other developmental domains are appropriate for his age.

119
Q

A 14-year-old adolescent girl is brought to the emergency department for difficulty ambulating. One week ago, she developed leg pain, primarily in her thighs and feet. Her pain progressed to difficulty walking up and down stairs. Over the past 24 hours she has had difficulty standing and walking, and she feels that her arms have become “clumsy.” She denies back pain, trauma, or bowel or bladder changes. Three weeks ago she had a mild upper respiratory infection that resolved with supportive care.

The girl is anxious, alert, and oriented to person, place, and time. Her cranial nerve examination findings are normal. Motor examination shows hypotonia in her bilateral lower extremities. She is able to move her legs across the bed but not lift them against gravity, and she cannot plantar flex or dorsiflex her ankles. Her upper extremities have normal proximal strength, with mild distal weakness. Deep tendon reflexes are 1+ in the upper extremities, and absent in the lower extremities. Her toes are mute to plantar stimulation. Sensation is intact; there is no sensory deficit level. Finger-to-nose testing is normal. She cannot ambulate. The remainder of her physical examination findings are normal.

Magnetic resonance imaging of the spine is normal.
Of the following, the BEST next management step for this adolescent is administration of intravenous

A.	botulism antitoxin
B.	ceftriaxone
C.	immune globulin
D.	methylprednisolone
A

The adolescent in the vignette has ascending flaccid paralysis and areflexia without evidence of encephalopathy or myelopathy. This presentation is consistent with Guillain-Barré syndrome (GBS). The diagnosis of GBS is clinical. While diagnostic testing can be used to exclude alternative etiologies or provide additional supportive data, testing results can be normal, especially early in its clinical course. The foundation of treatment is supportive, with close monitoring for evolution to impending respiratory failure. Of the response choices, administration of intravenous immune globulin (IVIG) is the best next management step for this adolescent because of the severity and ongoing progression of her symptoms. Intravenous immune globulin is an immunomodulating therapy that can reduce hospital length of stay and time to ambulation in individuals with GBS.

PREP Pearls
Guillain-Barré syndrome is an autoimmune, inflammatory, demyelinating polyneuropathy that can have an acute-subacute presentation characterized by ascending symmetric flaccid paralysis and areflexia.
Guillain-Barré syndrome is a clinical diagnosis; testing is not required to initiate treatment. While testing can be used to exclude alternative etiologies or provide additional supportive data, results can be normal, especially early in its course.
Intravenous immune globulin is an immunomodulating therapy that can shorten hospital length of stay and time to ambulation for individuals with Guillain-Barré syndrome.

120
Q

A 5-year-old boy with acute lymphoblastic leukemia is admitted to the hospital after 3 days of diarrhea. He has had 8 small-volume, loose stools with mucus and blood during the past 24 hours. His younger brother has similar symptoms. The family visited a waterpark 2 days before symptom onset. On physical examination, the boy’s temperature is 39°C, heart rate is 120 beats/min, respiratory rate is 28 breaths/min, blood pressure is 95/55 mm Hg, and oxygen saturation is 99% in room air. He appears uncomfortable, and his abdomen is slightly tender.

The boy’s absolute neutrophil count today is 500/μL (0.50 × 109/L). A stool culture from 2 days ago yields a gram-negative rod that is non–lactose fermenting, oxidase negative, and non–hydrogen sulfide producing.

Of the following, the BEST next management step for this child is

A.	intravenous rehydration without antibiotics
B.	intravenous rehydration and ceftriaxone
C.	oral rehydration and amoxicillin
D.	oral rehydration without antibiotics
A

The child in the vignette has symptoms of acute severe gastroenteritis, including fevers, vomiting, and dysentery (diarrhea with blood and mucus). The Gram stain and biochemical test characteristics on stool culture suggest infection from a Shigella species. The boy is immunocompromised, with an absolute neutrophil count of 500/μL (0.50 × 109/L). In this situation, intravenous hydration and empiric treatment with intravenous ceftriaxone are recommended while awaiting culture and sensitivity data. Oral amoxicillin is not well absorbed in cases with these clinical findings; therefore, it is not an appropriate treatment option. Rehydration, preferably oral, is the mainstay of management in an otherwise healthy child with mild to moderate symptoms; antibiotics would not be recommended in such cases.

PREP Pearls
Shigella is the third most common cause of acute bacterial gastroenteritis in the United States after nontyphoidal Salmonella and Campylobacter species.
The mainstay of management in an otherwise healthy child with mild to moderate symptoms of Shigella infection is rehydration. Antibiotic therapy is not indicated.
Antibiotic therapy is indicated for Shigella infection in an immunocompromised child or a child with severe illness (dysentery and bacteremia). Intravenous ceftriaxone is the treatment of choice.

121
Q

An 8-month-old boy is seen in the emergency department after falling from his crib. A cephalohematoma is noted on physical examination. His vital signs are appropriate for age. The remainder of his physical examination findings are normal. His medical history includes a 2-week neonatal intensive care unit admission for prolonged bleeding after circumcision.

Laboratory data are shown:

Laboratory Test

Result

White blood cell count

9,800/µL (9.8 × 109/L)

Hemoglobin

8.5 g/dL (85 g/L)

Platelet count

90 × 103/µL (90 × 109/L)

Mean corpuscular volume

85 fL

Reticulocyte

2.5%

vWF activity

34% (reference range, 54%-152%)

vWF antigen

88% (reference range, (50%-150%)

Factor VIII assay

95% (reference range, (50%-200%)

vWF multimer assay

Abnormal

vWF, von Willebrand factor.

Of the following, the BEST next step in management of this infant is

A.	blood transfusion
B.	desmopressin therapy
C.	factor complex injection
D.	platelet transfusion
A

The boy in the vignette most likely has von Willebrand disease (vWD); this diagnosis is supported by his laboratory findings of an abnormal von Willebrand panel. Item C179 summarizes the different types of vWD. Type 2B is associated with thrombocytopenia, including neonatal thrombocytopenia, and is the most likely diagnosis for the child in the vignette.

Desmopressin causes an increase in factor VIII activity, von Willebrand antigen, and ristocetin cofactor activity. Desmopressin is the most common treatment for individuals with type 1 vWD, although not every affected individual has a response. Treatment with desmopressin may cause hyponatremia and therefore requires fluid restriction when used. The infant in the vignette does not have type 1 vWD and would therefore not benefit from this therapy. In addition, in individuals with type 2B vWD, desmopressin can lead to a transient decrease in platelets, which may worsen bleeding.

PREP Pearls
Von Willebrand factor is a plasma protein that functions as a carrier for factor VIII and helps bind platelets to damaged endothelium.
Von Willebrand disease type 2B may be associated with thrombocytopenia
Desmopressin is not an appropriate treatment for type 2B von Willebrand disease and may worsen thrombocytopenia.

122
Q

A neonate at term gestation was delivered vaginally 2 hours ago. His mother has a history of genital herpes simplex virus (HSV); she was treated with appropriate antiviral suppressive therapy during pregnancy. She had no HSV outbreaks during pregnancy and currently has no prodromal symptoms or active HSV lesions anywhere on her body. The neonate’s Apgar scores were 9 at both 1 and 5 minutes after birth. His physical examination findings are normal.

Of the following, the BEST next step in this neonate’s management is to

A.	avoid breastfeeding
B.	observe for signs of infection
C.	obtain surface cultures for herpes simplex virus
D.	start empiric acyclovir
A

PREP Pearls
Neonates born to mothers who have a history of genital herpes simplex virus infection but do not have active lesions or prodromal genital symptoms (eg, itching or burning) at delivery require only observation for signs of infection.
To reduce the risk of vertical transmission, cesarean section should be performed for mothers who have active herpes simplex virus lesions or prodromal genital symptoms (eg, itching or burning) at the time of delivery.

123
Q

A 6-year-old girl is seen for a routine health supervision visit. Her mother is concerned because the girl has been wetting her bed for the past few weeks; she is dry during the day. She has had no fever, increased urinary frequency or urgency, or burning with urination. The girl has been completely toilet trained since age 4 years. She does not awaken when she wets the bed, occasionally snores while sleeping, and recently seems tired in the morning. Her physical examination findings are normal except for 3+ bilateral tonsillar enlargement.

Of the following, the MOST appropriate next step in management for this girl is

A.	administration of antibiotics
B.	institution of proper sleep hygiene
C.	referral to an otolaryngologist
D.	use of a bedwetting alarm
A

The girl in the vignette has nocturnal enuresis which is likely secondary to sleep-disordered breathing (SDB). Her additional signs and symptoms of SDB include snoring and daytime fatigue. Given her 3+ tonsillar hypertrophy on physical examination, the most appropriate next step in management is referral to an otolaryngologist for consideration of tonsilloadenoidectomy.

PREP Pearls
Tonsillar and adenoid hypertrophy can lead to sleep-disordered breathing and obstructive sleep apnea.
Nocturnal enuresis can be a symptom of sleep-disordered breathing secondary to tonsillar and adenoid hypertrophy.
Tonsilloadenoidectomy is an appropriate first-line treatment in otherwise healthy children with obstructive sleep apnea due to documented tonsillar and adenoid hypertrophy.

124
Q

A 2-month-old infant is seen for a health supervision visit. She has normal growth and age-appropriate developmental milestones. The infant was diagnosed with a metabolic disorder identified through the state newborn screening program. She was evaluated in the metabolic clinic and started on a special formula. The infant’s mother reports that her daughter’s urine and cerumen had a maple syrup odor that resolved after dietary intervention.

Of the following, the metabolic condition identified in this infant is a DISORDER of

A.	amino acid metabolism
B.	fatty acid oxidation
C.	glycogen storage
D.	lysosomal storage
A

PREP Pearls
Inborn errors of metabolism arise from enzyme deficiencies, which lead to accumulation of toxic substrates and inadequate creation of the product(s) of the enzymatic reaction.
Maple syrup urine disease can be identified on newborn screening. Clinical features in an untreated neonate may include maple syrup odor in cerumen and urine, encephalopathy, and central respiratory failure leading to death.
Amino acid disorders associated with an abnormal odor include maple syrup urine disease (maple syrup), phenylketonuria (mousy/musty), and isovaleric acidemia (sweaty feet).

125
Q

A 3-year-old girl is brought to the office for evaluation of burning with urination for 2 days. She has no urinary urgency, frequency, nighttime or daytime accidents, back pain, fever, rash, or joint pains. She has a soft bowel movement every day. She has been toilet-trained since 2 years of age. She has bubble baths 3 times a week and showers on most other days. The girl’s weight is at the 50th percentile, height is at the 75th percentile, and her blood pressure is 90/60 mm Hg. Her physical examination findings, including the genitourinary examination findings, are normal.

Urinalysis with microscopy results are shown:

Test

Result

Appearance

Yellow

Specific gravity

1.015

Leukocyte esterase

Negative

Nitrite

Negative

Blood

Positive

Protein

Negative

Red blood cells

10-20/HPF

White blood cells

<5/HPF

Of the following, the MOST likely diagnosis for this girl is

A.	chemical urethritis
B.	dysfunctional voiding
C.	urethral prolapse
D.	urinary tract infection
A

The girl in the vignette most likely has chemical urethritis caused by frequent bubble baths. This diagnosis is supported by her symptom of dysuria and the urinalysis results showing microscopic hematuria without pyuria.

PREP Pearls
Dysuria is a common symptom seen with urinary tract infection, inflammation, irritation, trauma, obstruction, and several systemic illnesses.
Chemical urethritis can be caused by detergents, soaps, ointments, and bubble baths.

126
Q

An 8-year-old boy is brought to the emergency department with right elbow pain after falling from the monkey bars at school. He appears distressed, is holding his right arm, and is crying with severe pain. On physical examination, he has significant swelling and deformity of his right elbow. Intravenous access is obtained and analgesics are administered. A right upper extremity radiograph is obtained (Item Q185 ).

Of the following, the BEST next step in management for this boy is to

A.	attempt reduction under conscious sedation
B.	consult orthopedic surgery
C.	perform a thorough neurovascular examination
D.	place a long-arm posterior splint
A

The radiograph of the boy in the vignette demonstrates a type III (completely displaced) supracondylar fracture. Any significantly displaced fracture in a child warrants immediate assessment for neurovascular compromise with a thorough neurovascular examination of the affected extremity. If the child displays signs of neurovascular compromise, urgent reduction under conscious sedation or general anesthesia is indicated. If there are no signs of neurovascular compromise, the next management steps include orthopedic surgery consultation and placement in a long-arm posterior splint until the specialist is consulted. However, these steps should not be undertaken until both neurologic and vascular status of the extremity is completely assessed.

There are 3 types of supracondylar humerus fractures:

Type I: Non-displaced or minimally displaced
Type II: Displaced with intact posterior cortex
Type III: Completely displaced with no cortical contact

127
Q

A 7-year-old girl is seen for concerns about rapid growth. She has not had headaches, vision changes, increased thirst, or increased urination. Her family has not noted any breast development, vaginal bleeding, or vaginal discharge. Review of the girl’s growth chart shows that she had been growing along the 25th percentile for height for age, but in the last year, her height increased to the 95th percentile. Her weight is at the 60th percentile and body mass index is at the 15th percentile. On physical examination, her heart rate is 140 beats/min and her blood pressure is 120/50 mm Hg. She appears tall, thin, and “jittery.” Her neck appears full. Sexual maturity rating is 1 for breast development and pubic hair. The remainder of her physical examination findings are normal.

Of the following, the test MOST likely to reveal this girl’s diagnosis is a/an

A.	17-hydroxyprogesterone level
B.	insulin-like growth factor 1 level
C.	luteinizing hormone level
D.	thyroid-stimulating hormone level
A

The girl in the vignette, with linear growth acceleration, tachycardia, hypertension, goiter, and tremulousness, has hyperthyroidism. Her thyroid-stimulating hormone production would likely be completely suppressed due to thyroid hyperfunction, and her free thyroxine level would be elevated. Elevated levels of thyroid hormone increase linear growth through a direct stimulatory effect on the growth plates and enhanced growth hormone (GH) secretion from the pituitary gland. In contrast, poor linear growth is frequently seen with longstanding hypothyroidism.

PREP Pearls
Thyroid disease should be considered in the setting of abnormal growth.
Thyroid hormone has a direct stimulatory effect on the growth plates and enhances growth hormone secretion from the pituitary gland; poor growth is often seen with longstanding hypothyroidism.
The increased growth velocity of hormonally related tall stature can be differentiated from the normal growth velocity of non-hormonal causes of tall stature.

128
Q

A 10-year-old girl is seen for a health supervision visit. Her parents are concerned because their daughter can bend backward at the elbows and knees to a degree that seems unnatural and she is “double-jointed.” They want to make sure that it is safe for her to participate in sports. The girl is active in both soccer and gymnastics without pain or limitations. On physical examination, the girl demonstrates 15 degrees of elbow extension and 15 degrees of knee extension bilaterally. She is able to touch her palms to the floor without bending her knees but does not have signs of increased joint mobility in her hands or wrists. The remainder of her physical examination findings are normal.

Of the following, the BEST next step in this girl’s evaluation and management is to

A.	reassure the family about her physical examination findings
B.	recommend that she discontinue gymnastics due to risk of joint injury
C.	refer her to a geneticist for evaluation for a connective tissue disorder
D.	refer her to a physical therapist for generalized strengthening
A

The girl in the vignette exhibits excess mobility of the elbows, knees, and spine. She does not have any joint pain or limitations in her activities. This type of joint mobility is a normal finding in school-aged children. The best next step is to reassure the family about her physical examination findings. She does not need any additional evaluation, treatment, or restriction of her activities.

PREP Pearls
Joint hypermobility that is not associated with pain or activity limitation is a normal finding in school-aged children.
Ehlers-Danlos syndrome, hypermobility type, describes individuals with joint hypermobility, arthralgias, and/or atraumatic joint dislocations, and other criteria, such as skin hyperextensibility, striae, and atrophic scarring.
Ehlers-Danlos syndrome, hypermobility type, is diagnosed clinically.

129
Q

A 17-year-old adolescent boy is evaluated for several months of generalized abdominal pain, diarrhea, flatulence, and bloating. He has 2 to 3 loose, watery, nonbloody stools daily (most commonly after lunch and dinner), without nocturnal stooling. He is otherwise well. His father has had similar symptoms for many years. For breakfast, the adolescent usually eats toast and peanut butter with fruit. At lunch, he usually eats a ham and cheese sandwich, chips, a salad with ranch dressing and cheese, and chocolate milk. Dinners are often fast food options after sports games, typically large sandwiches and chocolate milk or chicken nuggets, french fries, and a milkshake. He snacks on fruit and nuts during the day. On physical examination, the boy appears healthy with a weight of 64 kg (50th percentile for age), height of 175 cm (50th percentile for age), and body mass index of 21 kg/m2 (50th percentile for age). His abdomen is soft and nontender, but distended and tympanitic. His perianal examination findings are normal.

Additional testing reveals:

Laboratory Test (Stool)

Result

Fecal occult blood test

Negative

Ova and parasite examination

Negative

Lactoferrin

Negative

pH

5.0

Bacterial pathogen testing

Negative

Of the following, the test MOST likely to identify the cause of the adolescent’s symptoms is

A.	colonoscopy
B.	esophageal pH impedance study
C.	lactose breath test
D.	wheat IgE concentration
A

The adolescent male in the vignette has signs and symptoms consistent with carbohydrate malabsorption (watery and acidic stools, bloating, and flatulence). He reports an increase in symptoms after eating lactose-containing foods; thus, lactose intolerance is very likely and can be further investigated with a lactose breath test. Colonoscopy, esophageal pH impedance study, or wheat IgE testing would not be useful to identify lactose intolerance or other causes of carbohydrate malabsorption.

130
Q

A previously healthy, 16-year-old adolescent boy is seen in the emergency department. Two days ago, he fell from his bicycle and sustained a minor injury to his left arm. For 1 day he has had pain, redness, and swelling of his left upper extremity and fever up to 39.4°C. The pain and swelling have increased in severity over the past few hours, with spread of the redness to his shoulder (Item Q191). He has no sick contacts, exposure to pets or animals, or recent travel.

The adolescent appears toxic, with a temperature of 39.1°C, heart rate of 154 beats/min, respiratory rate of 42 breaths/min, and blood pressure of 83/36 mm Hg. Musculoskeletal examination is significant for exquisite tenderness and tense swelling of his left arm with a purplish discoloration of the skin. The remainder of his physical examination findings are unremarkable.

A complete blood count shows a white blood cell count of 23,000/µL (23 × 109/L) with 97% neutrophils, a hemoglobin level of 10.9 g/dL (109 g/L), and a platelet count of 76 × 103/µL (76 × 109/L). After a blood sample is drawn for culture, intravenous vancomycin and ceftazidime are administered.

Of the following, the BEST next step in this adolescent’s management is

A.	administration of clindamycin, intravenously
B.	magnetic resonance imaging of the left arm
C.	plain radiography of the left arm
D.	urgent surgical debridement
A

d

PREP Pearls
Necrotizing fasciitis is a severe, life-threatening pyogenic infection of the subcutaneous tissues and fascia characterized by rapidly progressive spread of infection along the fascial planes, resulting in tissue necrosis and often sepsis.
Recent trauma, burns, surgery, and varicella infection are common risk factors for necrotizing fasciitis in children.
Necrotizing fasciitis is a surgical emergency. Urgent surgical exploration with aggressive debridement of devitalized tissue is required.

Clindamycin is recommended due to its anti-toxin activity and other properties (eg, long postantibiotic effect unaffected by inoculum size compared to penicillin).

131
Q

A 3-month-old infant is brought to the office for concerns about noisy breathing, poor feeding, and slow growth. He has had noisy breathing since 1 month of age that is worse with feeding and while sleeping on his back. He nurses poorly and is not growing as well as expected. The feeding difficulties and noisy breathing have been worsening over the past month. He has not had fever or other signs of illness. His prenatal and birth history are unremarkable; birth weight was 3.02 kg.

On physical examination, the infant weighs 4.2 kg. His temperature is 37°C, heart rate is 120 beats/min, respiratory rate is 30 breaths/min when upright and calm, and oxygen saturation is 97% in room air. When supine, he has variable inspiratory stridor with use of accessory muscles and intercostal retractions. Breath sounds are clear with good aeration throughout. The remainder of his physical examination findings are unremarkable.

Of the following, the BEST description of this infant’s airway compromise is

A.	dynamic bronchial obstruction
B.	dynamic laryngeal obstruction
C.	fixed nasal obstruction
D.	fixed tracheal obstruction
A

The infant in the vignette has laryngomalacia, a dynamic airway obstruction at the level of the larynx. Most infants with laryngomalacia have noisy breathing that is worse when supine and sometimes worse when feeding. This infant has critical laryngomalacia with increased work of breathing that is interfering with feeding and growth. His inspiratory stridor localizes the site of obstruction to above the glottis, and his varying signs and symptoms corroborate a dynamic rather than fixed process.

PREP Pearls
Upper airway obstruction at or above the glottis is associated with inspiratory stridor; obstruction below the glottis is associated with biphasic or expiratory wheezing.
Nasal obstruction in infants (eg, choanal atresia) is life-threatening, as infants are obligate nose breathers. Acute upper airway obstruction at any age is a medical emergency.
Airway obstruction can be fixed or dynamic; either type can be worsened by superimposed infection or inflammation.

132
Q

The initial physical examination of a 12-hour-old neonate born at 39 weeks’ gestation reveals the finding seen in Item Q194.hypospadius The stretched penile length is 3.8 cm, and the scrotum is well developed with bilateral descended testes. The remainder of his physical examination findings are normal.

Of the following, the BEST next step in this neonate’s management is to

A.	defer circumcision
B.	obtain a pediatric endocrinology consultation
C.	obtain pelvic ultrasonography
D.	obtain an urgent pediatric urology consultation
A

The neonate in the vignette has distal shaft hypospadias (Item C194A). As in this case, a dorsal hooded prepuce is often seen. Distal shaft hypospadias, which accounts for approximately 90% of cases, is usually surgically repaired at 6 to 12 months of age. Circumcision, if desired, should be deferred because the foreskin tissue may be used in the hypospadias repair. Parents should be reassured that circumcision can be performed at the time of hypospadias repair. Referral to a pediatric urologist is recommended, but an urgent urology consultation is not indicated. The neonate in the vignette’s stretched penile length is normal for age (mean [SD], 3.9 [0.8] cm in term infants), and both testes are descended in a normal scrotum, making a disorder of sexual development (DSD) unlikely; therefore, pelvic ultrasonography and a pediatric endocrinology consultation are not indicated.

PREP Pearls
Circumcision should be deferred for neonates with hypospadias because the foreskin tissue may be used in the surgical repair.
Ninety percent of hypospadias cases are the distal shaft subtype.
A neonate with hypospadias and micropenis, with or without undescended testes, should have an evaluation for disorders of sexual development.

133
Q

A term newborn male, intubated for respiratory distress, is in the neonatal intensive care unit. Physical examination shows a cleft palate, hooded eyelids, prominent nasal bridge, bulbous nose, and micrognathia. The remainder of his physical examination findings are normal. Significant findings on further evaluation include: tetralogy of Fallot, absent thymus, and a total serum calcium level of 7.8 mg/dL (1.9 mmol/L).

Of the following, this neonate’s MOST likely diagnosis is

A.	22q11.2 deletion syndrome
B.	trisomy 13
C.	trisomy 21
D.	Turner syndrome
A

Clinical features of trisomy 13 include small for gestational age, failure to thrive, microcephaly, cutis aplasia, cleft palate, holoprosencephaly, and postaxial polydactyly. Tetralogy of Fallot can be seen in trisomy 13. However, the neonate in the vignette has hypocalcemia and an absent thymus; his clinical features make 22q11.2 DS a more likely diagnosis.

PREP Pearls
22q11.2 deletion syndrome is characterized by conotruncal heart defects, cleft palate, immune deficiency, dysmorphic facial features, and endocrine abnormalities (eg, hypoparathyroidism leading to hypocalcemia).
The diagnostic test of choice for 22q11.2 deletion syndrome is a chromosomal microarray. Fluorescent in situ hybridization for the 22q11.2 critical region is reserved for a critically ill neonate with classic clinical features in whom a rapid turnaround time is required.

134
Q

A full-term, 2-day-old male neonate in the newborn nursery has a platelet count of 48.0 × 103/µL (48.0 × 109/L). A complete blood cell count was ordered after his parents reported that his older sister had thrombocytopenia at birth. The sister is now doing well without any medical problems. The neonate’s physical examination findings are normal.

Of the following, the BEST next step in this neonate’s management is

A.	head ultrasonography
B.	intravenous immune globulin administration
C.	observation
D.	platelet administration
A

The neonate in the vignette has moderate thrombocytopenia (platelet count <50.0 × 103/µL [50.0 × 109/L]) but is otherwise stable with normal physical examination findings. The family history of thrombocytopenia makes neonatal (or fetal) alloimmune thrombocytopenia (NAIT) a likely diagnosis. Head ultrasonography is indicated at this time due to his level of thrombocytopenia and the family history of thrombocytopenia in a previous pregnancy.

PREP Pearls
Neonatal (fetal) alloimmune thrombocytopenia should be ruled out in any neonate with unexplained, moderate to severe thrombocytopenia.
Head ultrasonography should be ordered for a neonate with moderate to severe thrombocytopenia to evaluate for evidence of intracranial bleeding.
Neonatal (fetal) alloimmune thrombocytopenia can be more severe and occur earlier in future pregnancies; antenatal treatment may be warranted.

135
Q

A neonate is evaluated in the newborn nursery 2 hours after birth. She was born at 36 weeks’ gestation to a 24-year-old woman with a history of gestational diabetes and smoking during pregnancy. The mother’s membranes ruptured 20 hours before delivery. The infant’s Apgar scores were 7 and 9 at 1 and 5 minutes, respectively. The neonate’s weight is 1,860 g (3rd percentile), length is 44 cm (10th percentile), and head circumference is 32 cm (25th percentile). She is alert, active, and pink with no respiratory distress. The remainder of her physical examination findings are normal.

Of the following, the MOST likely cause of this neonate’s low birthweight is

A.	a chromosomal abnormality
B.	fetal hypoxia
C.	low insulin level
D.	sepsis
A

The neonate in the vignette is small for gestational age (SGA), most likely due to maternal smoking during pregnancy. Small for gestational age is defined as a birthweight below the 10th percentile for gestational age. Etiologic factors in SGA include conditions causing placental insufficiency, congenital viral infections, chromosomal and genetic conditions, and toxins such as cigarette smoke, alcohol, and illicit drugs.

136
Q

A 6-month-old boy is brought to the emergency department for excessive crying over the past 4 hours. He began crying abruptly when his mother was walking him in his stroller and has been inconsolable and rubbing his eyes since then. He has had no fevers or recent illness. The boy’s physical examination findings are normal except for mild injection of his left conjunctiva and clear tearing from both eyes. Fluorescein stain is applied to both eyes, and results of a Wood lamp examination of the left eye are shown in Item Q202.

Of the following, the MOST appropriate therapy for this boy is application of

A.	an eye patch for 72 hours
B.	steroid ophthalmic drops
C.	tetracaine ophthalmic drops
D.	topical ophthalmic antibiotic
A

The boy in the vignette has a corneal abrasion (Item C202). The most appropriate treatment is application of topical ophthalmic antibiotic ointment or drops to the affected eye until the abrasion is healed. Corneal abrasions are very common in childhood. Typical signs and symptoms include pain, tearing, redness, blurry vision, photophobia, and foreign body sensation. Children may also have mild eyelid swelling or redness resultant from mechanical irritation of rubbing the eye. In infants and very young children the only symptom of a corneal abrasion may be fussiness or excessive crying.

PREP Pearls
Signs and symptoms of a corneal abrasion include pain, tearing, redness, blurry vision, photophobia, and foreign body sensation; it may present as excessive fussiness or crying in young infants.
The treatment of a corneal abrasion is application of topical ophthalmic antibiotic ointment or drops to the affected eye until the abrasion is healed.
Tetracaine eye drops should not be prescribed for home use; tetracaine overuse can cause corneal cell death and inhibit corneal healing.
Suspected corneal abrasions should be stained with fluorescein, and an examination performed with a cobalt blue light, using a standard ophthalmoscope or a Wood lamp.

137
Q

A 4-hour-old male neonate is admitted to the neonatal intensive care unit for cyanosis. The maternal history is unremarkable, and the mother received good prenatal care. He was born by spontaneous vaginal delivery at 39 weeks’ gestation. He has a heart rate of 160 beats/min, respiratory rate of 50 breaths/min, and blood pressure of 65/35 mmHg. Pulse oximetry readings are 85% on all 4 extremities in room air. His lungs are clear to auscultation, there is no increased work of breathing. A high-pitched systolic ejection murmur is heard over the left upper sternal border, and good pulses are palpated throughout. Physical examination findings are otherwise unremarkable. A prostaglandin infusion is started.

Of the following, this neonate’s MOST likely diagnosis is

A.	aortic valve stenosis
B.	mitral valve stenosis
C.	pulmonary valve stenosis
D.	tricuspid valve stenosis
A

The neonate in the vignette has cyanosis, no respiratory distress, low pulse oximetry readings in all extremities, and a systolic ejection murmur in the pulmonic location. These findings are most consistent with pulmonary valve stenosis. If the neonate had aortic valve stenosis, the murmur would be more prominent at the right upper sternal border (aortic valve position), and the right upper extremity saturation would be higher than the lower extremity saturation. Stenosis of either atrioventricular valve (mitral or tricuspid) would result in a diastolic murmur.

PREP Pearls
The murmur of pulmonary valve stenosis is a middle- to high-pitch systolic ejection murmur best heard at the left upper sternal border.
Neonates with critical pulmonary stenosis will be desaturated due to limited pulmonary blood flow and right-to-left shunting, primarily at the atrial level.
Neonates with critical and severe pulmonary stenosis should undergo balloon valvuloplasty.

138
Q

A 9-month-old infant is evaluated in the clinic for an itchy rash of 3 days’ duration. The infant had a similar rash at ages 2 and 6 months. Each time, the rash resolved after a few days with the use of moisturizers. He is otherwise healthy. His medical history is significant only for physiologic jaundice on day 5 after birth requiring phototherapy and a brief viral illness at 4 months of age. The family history is notable for allergic rhinitis in the father. The infant was exclusively breastfed until 4 months of age and currently is taking breast milk, pureed foods, and infant cereals. On physical examination, an erythematous maculopapular rash with scaling and excoriations is seen on his face (Item Q204) and the extensor aspects of the arms, legs, and wrists.

Of the following, the MOST likely predisposing factor for this infant’s condition is

A.	allergic rhinitis in the father
B.	breastfeeding
C.	jaundice requiring phototherapy
D.	prior viral illness
A

The infant described in the vignette has a history and physical examination findings consistent with infantile atopic dermatitis. Atopic dermatitis (AD) is a chronic skin condition with inflammation presenting as erythema and pruritus that is characterized by exacerbations and remissions. The prevalence of atopic dermatitis is increasing worldwide, especially in better resourced countries; most affected individuals develop the condition during infancy. Atopic dermatitis is grouped under the atopic spectrum of diseases which includes food allergies, allergic rhinitis, and asthma.

PREP Pearls
Atopic dermatitis is a chronic inflammatory skin condition characterized by exacerbations and remissions.
The prevalence of atopic diseases is increasing, especially in better-resourced countries.
Genetic predisposition plays a strong role in the development of atopic dermatitis and other allergic conditions.

139
Q

A 17-year-old adolescent girl is seen in the emergency department. She has a 1-month history of worsening severe early morning headaches, tinnitus, and intermittent blurring of her vision. She was sent for evaluation by her ophthalmologist who noted bilateral papilledema and visual acuity of 20/30 in both eyes on examination that morning. She is otherwise healthy and takes no medications.

In the emergency department, the adolescent’s vital signs are normal. Her weight is 110 kg and body mass index is 41.4 kg/m2. With the exception of bilateral papilledema, her physical examination findings are normal. Brain magnetic resonance imaging with and without contrast and magnetic resonance venography findings are normal. A lumbar puncture is performed and reveals an opening pressure of 36 cm H2O and normal cerebrospinal fluid laboratory results.

Of the following, the BEST next step in this adolescent’s acute management is

A.	administration of acetazolamide
B.	counseling on healthy weight loss
C.	neurosurgery consultation
D.	referral back to ophthalmology
A

The adolescent in the vignette has idiopathic intracranial hypertension (IIH; previously known as pseudotumor cerebri); administration of acetazolamide is the best next step in her initial acute management. Idiopathic intracranial hypertension is a syndrome of unknown etiology; it is a diagnosis of exclusion. The pathophysiology is poorly understood. A proposed mechanism is blockage of cerebrospinal fluid (CSF) absorption at the arachnoid villi secondary to cerebral venous hypertension from transverse venous sinus stenosis.

PREP Pearls
Idiopathic intracranial hypertension is a syndrome of unknown etiology; it is a diagnosis of exclusion. Signs and symptoms of increased intracranial pressure include headache, visual obscurations, pulsatile tinnitus, diplopia, papilledema, cranial nerve VI palsies, and visual field defects.
The management of idiopathic intracranial hypertension has 2 goals: 1) preservation of vision and 2) acute symptom relief. A carbonic anhydrase inhibitor (eg, acetazolamide) is the first-line medical therapy in conjunction with weight loss (for children with obesity); surgical intervention is reserved for fulminant presentations or treatment failure.
Children and adolescents with idiopathic intracranial hypertension require long-term ophthalmological follow-up with serial ophthalmological examinations that evaluate the fundi, visual acuity, and visual fields.

140
Q

A 13-year-old adolescent girl is seen in the office for evaluation of right knee pain and swelling. The day prior, she twisted her knee while rising from a chair. She felt something shift in her knee and had a sudden onset of pain. Within an hour, she developed knee swelling. On physical examination, she is unable to fully bend or straighten her right knee, and there is a large joint effusion. There is tenderness around the patella, and she is very apprehensive with attempts to move her patella. There is no ligamentous laxity with the Lachman test or with valgus or varus stress. The remainder of her physical examination findings are normal.

Of the following, this girl’s MOST likely diagnosis is

A.	anterior cruciate ligament tear
B.	medial meniscus tear
C.	patella dislocation
D.	patella tendon rupture
A

The girl in the vignette felt something shift in her knee and experienced a rapid onset of pain and swelling after a seemingly mild twisting injury. She has a large effusion and apprehension with patellar motion but no anterior cruciate ligamentous laxity. The most likely diagnosis is a patellar dislocation.

PREP Pearls
Patellar dislocations and anterior cruciate ligament tears can both present in adolescents with rapid onset of pain and swelling following a twisting knee injury.
Knee injury prevention warm-up programs have been shown to reduce rates of knee injury in young athletes.
Patella tendon rupture is very uncommon in the pediatric population.

141
Q

A right orchiectomy and left orchidopexy were performed on a 1-day-old boy for in utero right testicular torsion. Postoperatively, he was returned to the neonatal intensive care unit with normal vital signs in room air. Six hours after surgery, he was minimally responsive and hypotonic, with pinpoint pupils. A narcotic overdose was suspected. A review of the medication list revealed that while in the postanesthesia care unit the neonate received 3.6 mg of intravenous morphine for pain instead of 0.36 mg. The resident gave a verbal order, documented in the medical record, for 0.1 mg/kg of morphine. The neonate’s weight is 3.6 kg. Appropriate doses of naloxone were administered twice during a 4-hour period. Five hours after treatment with naloxone, he was awake, breastfeeding, and active. The medical error was disclosed to the parents by the attending physician.

Of the following, the health care team’s BEST next step in response to this event is to

A.	implement a policy of no verbal orders
B.	implement an error prevention campaign
C.	notify the risk management team
D.	provide support and counseling
A

d

PREP Pearls
Medication errors are the most common adverse event in pediatrics.
Disclosure of an adverse event to a patient and family is imperative and should occur at the earliest opportunity.
Support and counseling should be provided to the patient, family, and health care practitioner after an adverse event. The practitioner is often the “second victim,” experiencing emotional distress following a patient adverse event.

142
Q

A 16-year-old adolescent boy is seen for follow-up after hospital admission for acute onset of abdominal pain, nausea, and hematemesis. His evaluation included an upper endoscopy which revealed acute hemorrhagic erosive gastritis. The adolescent was started on acid-suppressing medication and discharged from the hospital after symptom resolution. He has a history of asthma, for which he uses fluticasone and montelukast daily, and a history of headaches, for which frequent use of acetaminophen is reported. In addition, he has a history of alcohol and marijuana use; on the day the symptoms began, he had consumed multiple alcoholic beverages and used marijuana heavily.

Of the following, the substance MOST likely to have caused the adolescent’s endoscopy finding is

A.	acetaminophen
B.	alcohol
C.	marijuana
D.	montelukast
A

The adolescent in the vignette developed acute onset of hematemesis, and endoscopy demonstrated acute hemorrhagic erosive gastritis. Of the substances listed, alcohol is most likely to cause these findings. Use of acetaminophen, montelukast, and/or marijuana are not associated with this endoscopic finding.

PREP Pearls
Symptoms of alcohol-induced gastritis can include nausea, vomiting, abdominal pain, and hematemesis.
Alcohol use should be considered in children or adolescents exhibiting nausea, vomiting, abdominal pain, and/or hematemesis.
Treatment of alcohol-induced gastritis includes acid suppression and avoidance of alcohol.
ABP Content Specifications(s)

143
Q

An 11-month-old boy was admitted to the hospital with fever of 6 days’ duration, conjunctival injection, and rash. Echocardiography showed a small pericardial effusion, normal coronary vessels, and normal ventricular function. He was diagnosed with Kawasaki disease and treated with intravenous immunoglobulin and high-dose aspirin. He is currently afebrile, doing well, and ready for discharge to home on low-dose aspirin.

Of the following, the BEST next step in this boy’s management is to

A.	administer inactivated influenza vaccine prior to discharge
B.	administer the measles-mumps-rubella and varicella vaccines at age 12 months
C.	advise his parents to monitor the number of urine-filled diapers each day
D.	restrict his formula intake to no more than 16 oz/day
A

The boy in the vignette is being treated with low-dose aspirin for Kawasaki disease. Of the response choices, the best next management step is to administer an inactivated influenza vaccine prior to hospital discharge. Children taking aspirin are at risk for developing Reye syndrome if they contract influenza or varicella infection. Although the risk is minimal, there have been case reports of Reye syndrome associated with Kawasaki disease and aspirin use. The American Heart Association guidelines recommend that infants and children (aged 6 months and older) who are prescribed aspirin receive an inactivated influenza vaccine prior to hospital discharge, if not already vaccinated for the influenza season. Children on long-term aspirin therapy should receive the influenza vaccine annually. Parents of children taking aspirin should be counseled regarding the symptoms of Reye syndrome and advised to seek immediate medical attention if these symptoms develop.

PREP Pearls
Children taking aspirin are at risk of developing Reye syndrome if they contract influenza or varicella infection.
Reye syndrome is characterized by altered mental status associated with significant elevation of liver transaminases and ammonia level.
Children with Kawasaki disease who are prescribed aspirin therapy should receive an inactivated influenza vaccine prior to hospital discharge.

144
Q

A 4-week-old male infant is evaluated for rapid breathing which has been present since birth. He had a respiratory rate of 50 breaths/min at his 2-week visit. He is otherwise healthy, nursing well, and growing appropriately. He does not have cyanosis or color change with feeding or sleeping. There is no family history of metabolic disease, and his newborn screening results are normal.

On physical examination, the infant is alert and comfortable. His temperature is 36.5°C, heart rate is 120 beats/min, respiratory rate is 55 breaths/min (counted over a full minute), and oxygen saturation by pulse oximetry is 99% in room air. His breath sounds are clear with good air exchange. There is no retraction, nasal flaring, or use of accessory muscles of respiration. The remainder of his physical examination findings are normal.

Of the following, the BEST next step is to

A.	begin inhaled albuterol treatments
B.	monitor with home pulse oximetry for 1 month
C.	order echocardiography and chest computed tomography D.	reassure the infant’s mother and reevaluate in 1 month
A

The infant in the vignette has a respiratory rate that is at the upper end of the normal range for his age; there are no findings to suggest underlying pathology. The best next step is to reassure the mother that he is not in danger, and schedule follow-up for reevaluation. This scenario is not uncommon and often precipitates an unnecessary referral to a specialist.

PREP Pearls
There is a broad range of normal respiratory rates with the widest variation in the first 2 years after birth.
Multiple factors affect respiratory rate including body temperature, anxiety, metabolic dysfunction, and central neurologic dysfunction.

145
Q

A 2-month-old boy is brought to the office for a health supervision visit. He was born to a 30-year-old mother by an uncomplicated vaginal delivery. Prenatal ultrasonography revealed multiple noncommunicating cysts on the left kidney; the right kidney appears normal. The family history is negative for cystic kidney disease. He is exclusively breastfed and has 6 to 8 wet diapers every day. The boy’s weight is at 50th percentile and height is at the 50th to 75th percentile. His vital signs are normal for age, and the remainder of his physical examination findings are normal.

Renal ultrasonography after birth revealed multiple noncommunicating cysts of various sizes occupying the left kidney without normal intervening renal parenchyma. The right kidney appears normal.

Of the following, the BEST next step in this infant’s management is to

A.	order a diuretic renal scan
B.	perform serial renal ultrasonography
C.	refer for surgical nephrectomy
D.	start trimethoprim-sulfamethoxazole prophylaxis
A

The infant in the vignette has a left multicystic dysplastic kidney (MCDK) and a normal-functioning right kidney. The best next step in this infant’s management is serial renal ultrasonography to monitor for involution of the MCDK and compensatory hypertrophy of the normal-functioning kidney.

PREP Pearls
Multicystic dysplastic kidney may be diagnosed on prenatal ultrasonography or incidentally by imaging performed for unrelated conditions or may present as an abdominal mass in the newborn.
Renal ultrasonography of a multicystic dysplastic kidney characteristically reveals multiple noncommunicating cysts of different sizes with dysplastic renal parenchyma and an atretic ureter.
Serial renal ultrasonography is indicated to monitor for involution of a multicystic dysplastic kidney and compensatory hypertrophy or any abnormalities of the contralateral kidney.

146
Q

A 16-year-old adolescent girl with systemic lupus erythematosus is seen in the emergency department for shortness of breath and light-headedness. Two weeks ago she had a cough, congestion, low-grade fever, and rhinorrhea that resolved. She appears ill with a temperature of 36.5°C, heart rate of 110 beats/min, and respiratory rate of 25 breaths/min. Lung examination reveals clear breath sounds without rales or wheezing; aeration is better on the right compared to the left. She has mild abdominal tenderness, and a fluid wave is noted. Her pulses are weak, and she has 1+ distal edema. Chest radiography results are shown in Item Q213.

Of the following, the MOST likely additional finding for this girl is (a)

A.	loud ejection murmur over the right sternal border
B.	low central venous pressure
C.	quiet precordium with distant heart sounds
D.	systolic hypertension in the upright position
A

The girl in the vignette is displaying signs and symptoms that cause concern for pericardial tamponade. She has shortness of breath, nonfocal lung examination findings, an abdominal fluid wave, diminished pulses, and distal edema. Chest radiography shows cardiomegaly and a pleural effusion. With her medical history of systemic lupus erythematosus (SLE), and physical examination and imaging findings concerning for serositis (abdominal fluid wave and pleural effusion), pericardial tamponade and pericardial effusion are high on the differential diagnosis list. Of the response choices, the most likely additional finding is a quiet precordium with distant heart sounds.

PREP Pearls
A high index of suspicion is required to diagnose pericardial tamponade in children. Bedside ultrasonography or echocardiography are useful in detecting pericardial fluid and assessing for tamponade.
Risk factors for pericardial tamponade include chest trauma, central venous catheters in or near the heart, recent cardiac surgery, malignancy, collagen vascular disease, severe renal disease, systemic infection with capillary leak, and hypothyroidism.
Initial stabilization of individuals with pericardial tamponade requires resuscitation with parenteral boluses of isotonic fluid to maintain cardiac output.

147
Q

A 4-month-old, full-term male infant with myelomeningocele, who underwent repair on the second day after birth, is seen for a routine health supervision visit. He has been doing well overall and is followed in the multidisciplinary myelomeningocele clinic. He can coo, laugh, grasp at toys, and bring his hands to his mouth. He is not yet rolling but can push himself up on his forearms when placed prone. He has started vomiting with feedings over the past week. He is not choking or gagging on feedings. The infant’s head circumference growth chart is shown in Item Q215.

Of the following, the MOST likely condition causing this infant’s symptoms and signs is

A.	Chiari II malformation
B.	dysgenesis of the corpus callosum
C.	hydrocephalus
D.	polymicrogyria
A

The infant in the vignette has an increasing head circumference and intermittent vomiting, which should raise concern for evolving increased intracranial pressure. With his history of myelomeningocele, these symptoms and signs are concerning for the development of hydrocephalus, a common comorbid condition. Hydrocephalus affects 60% to 95% of children with myelomeningocele, especially with higher-level lesions; ongoing surveillance for the development of signs and symptoms of hydrocephalus is important for prompt recognition and treatment.

The most common brain malformation accompanying myelomeningocele is a Chiari II malformation, in which there is downward displacement of part of the brainstem and cerebellum. Symptoms may include dysphagia, choking, hoarseness, stridor, central apnea, disordered breathing, opisthotonus, and in rare cases, sudden death. Many children with this condition are asymptomatic.

PREP Pearls
Myelomeningocele is a neural tube defect in which a portion of the spinal cord and nerves within the meningeal sac protrude through a defect in the vertebral column. Affected children experience motor and sensory dysfunction below the affected level of the spinal cord, the severity of which varies with the lesion level.
Malformations of the brain, spinal cord, soft tissues, and vertebral column can accompany myelomeningocele; Chiari II malformations, hydrocephalus, and malformations of the brain are common.
Hydrocephalus is a life-threatening condition which can present with a bulging or tense fontanelle, rapidly accelerating head circumference, lethargy, paralysis of upward gaze (sunsetting eyes), cranial nerve dysfunction, vomiting, headache, and irritability.

148
Q

A full-term neonate is examined in the newborn nursery. He was delivered via cesarean section with a birth weight of 7 lb, 10 oz. On physical examination, he is in no distress and has macrocephaly, posteriorly rotated ears, a symmetric defect of both upper extremities with intact thumbs, and a 3/6 mid-diastolic murmur best heard at the lower left sternal border. The remainder of his physical examination findings are unremarkable.

Results of a complete blood cell count are shown:

Laboratory Test

Result

White blood cell count

7,400/µL (7.4 × 109/L)

Neutrophils

59%

Lymphocytes

33%

Monocytes

7%

Eosinophils

1%

Hemoglobin

14.4 g/dL (144 g/L)

Platelet count

30 × 103/µL (30 × 109/L)

Mean corpuscular volume

100 fL

Of the following, the BEST treatment for this neonate’s condition is

A.	administration of corticosteroids
B.	administration of intravenous immunoglobulin
C.	bone marrow transplantation
D.	platelet transfusion
A

The neonate in the vignette has thrombocytopenia absent radius syndrome (TAR). This condition is characterized by severe thrombocytopenia and physical examination findings, including skeletal abnormalities of the upper (Item C216) or lower extremities, short stature, cardiac abnormalities (most commonly septal defects), and characteristic craniofacial features (posteriorly rotated ears, micrognathia, and macrocephaly). Thrombocytopenia absent radius syndrome is a congenital thrombocytopenia that is usually inherited in an autosomal recessive manner. It presents in infancy with thrombocytopenia, which may result in bleeding if not identified and treated promptly.

The mainstay of treatment for TAR syndrome is supportive care with platelet transfusions. Irradiated, single-donor platelets should be administered to prevent adverse effects. Because the thrombocytopenia associated with TAR syndrome usually resolves after 1 year of age, definitive treatment with bone marrow transplantation is not warranted. Bone marrow transplantation is the recommended treatment for some congenital causes of thrombocytopenia (eg, congenital amegakaryocytic thrombocytopenia and Wiskott-Aldrich syndrome).

149
Q

Painful vesicles on the tip of the nose and a vesicular rash over the forehead meet the criteria for Hutchinson sign, defined as varicella-zoster virus (VZV) infection involving the nasociliary nerve (a branch of the ophthalmic nerve, cranial nerve V, innervating the nose, eyelid, and conjunctiva). In cases of herpes zoster ophthalmicus, individuals with Hutchinson sign have a 3.4-fold higher incidence of ocular inflammation and 4-fold higher incidence of corneal denervation. Prompt ophthalmologic consultation is recommended to evaluate for ocular involvement and assess the extent of infection. Herpes zoster ophthalmicus, VZV involvement of the ophthalmic division of the fifth cranial nerve, is a potentially sight-threatening condition. Otolaryngologists, infectious disease specialists, and immunologists may also be involved in the care of affected children.

if pt lives with 7 yo sister who just got a BMT, what do you treat her with?

A

The adolescent’s 7-year-old sister is considered severely immunocompromised, because she is a recent hematopoietic stem cell transplant recipient. As she is a member of the adolescent’s household and the exposure to VZV is within 10 days, administration of varicella-zoster immune globulin is recommended at a dose of 125 units per 10 kg, intramuscularly (preferably within 96 hours). If varicella-zoster immune globulin is not available, oral valacyclovir can be administered for 22 days from the time of exposure.

150
Q

A 14-year-old adolescent boy is seen for a health supervision visit. He has a history of exercise-induced asthma and had an elevated blood pressure at a visit 6 months ago. He participates in a community baseball league. Today, his physical examination findings are significant for a blood pressure of 125/79 mm Hg (measured by manual sphygmomanometer), a body mass index of 35.9 kg/m2, (138% of the 95th percentile) and acanthosis nigricans around his neck. The remainder of his physical examination findings are unremarkable.

Of the following, the BEST next step in this adolescent’s management is to

A.	order a fasting glucose level, basic metabolic panel, lipid panel, liver transaminases, and urinalysis
B.	provide nutrition counseling and reassurance that sports participation will lead to weight loss
C.	recommend that he discontinue sports participation pending evaluation of his hypertension
D.	refer him for ambulatory blood pressure monitoring
A

The adolescent in the vignette meets criteria for the diagnosis of obesity with a body mass index (BMI) ≥95th percentile for age and sex. He has evidence of comorbidities, including insulin resistance (acanthosis nigricans) and hypertension. The best next step in his management is to order a fasting glucose level, basic metabolic panel, lipid panel, liver transaminases, and urinalysis to screen for additional comorbidities associated with obesity and hypertension. Current clinical guidelines recommend screening children and adolescents with a BMI ≥95th percentile for diabetes (fasting glucose level, hemoglobin A1c, or 2-hour oral glucose tolerance test), hyperlipidemia (lipid panel), and fatty liver disease (alanine aminotransferase and aspartate aminotransferase levels). A basic metabolic panel and urinalysis is indicated for all adolescents with hypertension.

151
Q

A pediatrician is urgently called to the delivery room to examine a newborn who had been delivered precipitously by a mother who did not receive any prenatal care. The infant is breathing comfortably in room air. Physical examination reveals a term-appearing infant with the abdominal findings shown (Item Q221). gastroschesis

Of the following the MOST appropriate next management step for this neonate is to

A.	call a surgical consult immediately
B.	cover the exposed bowel with warm, saline-soaked gauze
C.	perform endotracheal intubation and administer positive-pressure ventilation
D.	wrap the exposed bowel with a sterile compression bandage
A

The neonate in the vignette has gastroschisis, an abdominal wall defect lateral to the umbilicus with exposed bowel. In most cases, this condition will be identified prenatally, and the delivery room staff will be prepared ahead of time to manage the care of the neonate. In addition to providing regular delivery room care, it is critically important to protect the bowel from injury due to fluid loss by keeping it covered with warm, soaked gauze and putting the lower half of the body (from the upper abdomen down) in a bowel bag.
Wrapping the bowel in a compression bandage will cause further injury to the bowel.
PREP Pearls
In a neonate with gastroschisis, it is critically important in the delivery room to protect the exposed bowel from further injury due to fluid loss.
Surgical treatment of gastroschisis consists of either primary closure, or a staged approach with silo placement to protect the bowel while it is slowly introduced into the underdeveloped abdomen, and subsequent closure.
Gastroschisis may be associated with intestinal atresia.

152
Q

An 18-year-old young woman is brought to the emergency department by ambulance after being “found down” at her grandparents’ home while visiting them during summer break. She was found lying next to several empty liquor bottles. Since she was found, she has had no vomiting and received no medications. On physical examination, the adolescent is minimally responsive to painful stimuli. Vital signs are a temperature of 36.2°C, heart rate of 61 beats/min, respiratory rate of 12 breaths/min, blood pressure of 110/60 mm Hg, and oxygen saturation of 98% in room air. Her weight is 60 kg. Her pupils are equal, 3 mm, and reactive. The remainder of her physical examination findings are unremarkable.

Of the following, the BEST next step in this adolescent’s management is

A.	administration of charcoal by nasogastric tube
B.	administration of naloxone intramuscularly
C.	intubation for airway protection before performing urgent head computed tomography
D.	obtaining rapid serum glucose testing and ethanol, acetaminophen, and salicylate levels
A

Of the response choices, the best next step for the adolescent in the vignette is obtaining rapid serum glucose testing and ethanol, acetaminophen, and salicylate levels. The most common laboratory abnormality found in ethanol ingestion, and one requiring urgent treatment, is hypoglycemia. It is important to obtain ethanol and possible coingestant levels rapidly to direct further management.

PREP Pearls
Alcohol or ethanol intoxication is managed with supportive care and includes rapid evaluation for hypoglycemia, electrolyte derangements, and coingestant levels.
Adolescent alcohol use increases the risk of alcohol use disorders as adults, and thus requires early intervention and possible referral to treatment.

153
Q

PerformanceConfidenceAssessment History

Search
All Questions
Question View:
All
223 of 269Print Add Bookmark
ASSESSMENT PROGRESS:Total Questions: 269 Questions Answered: 222 Correct Answers: 146
Question 223
A previously healthy, 14-year-old adolescent girl is brought to the office for evaluation. She has a 4-day history of worsening right ankle pain and a 2-day history of fever and limp. She is now unable to bear weight. One week ago, she fell off her bicycle and sustained a minor bruise to her right ankle. Her immunizations are up to date.

On physical examination, the adolescent is alert and ill appearing, with a temperature of 38.2°C, heart rate of 106 beats/min, respiratory rate of 18 breaths/min, and blood pressure of 116/66 mm Hg. There is tenderness to palpation over the superior and medial regions of her right ankle, but no warmth, redness, or swelling. The remainder of her physical examination findings are normal.

Laboratory data are shown:

Laboratory Test

Result

White blood cell count

19,000/μL (19.0 × 109/L)

Segmented neutrophils

84%

Lymphocytes

13%

Monocytes

3%

C-reactive protein

7.9 mg/dL (79 mg/L)

A radiograph of her right ankle is normal.

Of the following, the BEST next step in this adolescent’s evaluation is to order

A.	a blood culture
B.	computed tomography of the right ankle and distal tibia
C.	serum procalcitonin level
D.	ultrasonography of the right ankle
A

The adolescent in the vignette has acute onset of fever, limp, right ankle bony tenderness, and an elevated C-reactive protein (CRP) level after an ankle injury. These signs and symptoms are consistent with acute hematogenous osteomyelitis (AHO).

PREP Pearls
The initial laboratory evaluation of children with suspected acute hematogenous osteomyelitis should include a C-reactive protein level and blood culture. Magnetic resonance imaging is the imaging modality of choice to confirm the diagnosis.
The pathogen most commonly associated with acute hematogenous osteomyelitis in every age group is Staphylococcus aureus (including methicillin-resistant S aureus); group B Streptococcus and Kingella kingae are important pathogens in neonates and young children, respectively.
Transition to oral antibiotic therapy is recommended for hospitalized children with suspected or proven acute hematogenous osteomyelitis who have demonstrated a good clinical and laboratory response to initial intravenous antibiotic therapy.

154
Q

A 2-day-old neonate born at 35 weeks’ gestation is seen in the newborn nursery for a 12-hour history of persistent yellow-stained emesis, intermittent irritability, and abdominal distention. The neonate has hyperbilirubinemia, currently requiring phototherapy, and is exclusively breastfed. There have been 2 meconium smears since birth. On physical examination, the neonate is not in any acute distress and has normal vital signs for age. The abdomen is soft and distended with a palpable upper abdominal loop. Digital rectal examination findings are normal and results in passage of dark meconium.

Abdominal radiography (Item Q225A) reveals multiple dilated loops of bowel. A contrast enema study (Item Q225B) reveals a transition to dilated colon at the splenic flexure with multiple filling defects consistent with the presence of meconium.

Of the following, the test MOST likely to reveal the cause of this neonate’s vomiting is

A.	abdominal ultrasonography
B.	lumbar puncture
C.	rectal suction biopsy
D.	upper gastrointestinal series
A

The neonate in the vignette has persistent vomiting, abdominal distention with a palpable bowel loop, and multiple dilated loops of bowel noted on radiography. This presentation is concerning for an intestinal obstruction, even if the emesis is not bilious (green). The contrast enema findings are consistent with Hirschsprung disease (HD); thus, a rectal suction biopsy is indicated as the next diagnostic step, and of the response choices is the test most likely to reveal the diagnosis. For this neonate, the biopsy demonstrated an absence of ganglion cells and confirmed the diagnosis of HD.

PREP Pearls
Intestinal obstruction should be suspected in neonates with bilious (green) emesis, failure to pass meconium by 24 to 48 hours after birth, abdominal distention, and/or history of maternal polyhydramnios. The absence of 1 or more of these signs and symptoms does not exclude neonatal intestinal obstruction.
Initial management steps when a neonatal bowel obstruction is suspected include a thorough physical examination, fluid resuscitation as needed, and urgent pediatric surgical consultation.
When intestinal obstruction is suspected, clinically unstable neonates may require urgent operative management without definitive imaging results.

155
Q

4-year-old girl is evaluated for a 2-day history of left eye redness and eyelid matting upon awakening. She has occasional left eye itching. She has been otherwise healthy without fever or rhinorrhea. The girl attends preschool and cannot return until cleared by a doctor.

On physical examination, there is left bulbar and tarsal conjunctival erythema with a small amount of green discharge at the corner of the left eye. There is purulent fluid visible behind her left tympanic membrane. The remainder of her physical examination findings are unremarkable.

Of the following, the MOST likely etiology of this girl’s findings is

A.	adenovirus type 7
B.	coxsackie virus A24
C.	Haemophilus influenzae non-typeable
D.	Moraxella catarrhalis
A

The girl in the vignette has conjunctivitis-otitis syndrome. Non-typeable Haemophilus influenzae is the most common pathogen associated with this syndrome and is therefore the most likely etiology of this girl’s findings. The Haemophilus influenzae childhood vaccine protects against Haemophilus influenzae type b; it does not provide protection against non-typeable Haemophilus influenzae. Oral amoxicillin-clavulanate is the first-line treatment for conjunctivitis-otitis syndrome.

PREP Pearls
Non-typeable Haemophilus influenzae is the most common etiology of conjunctivitis-otitis syndrome.
There is conflicting evidence as to whether children are more likely to have bacterial or viral causes of conjunctivitis; many sources report that bacteria may cause up to 80% of infectious conjunctivitis in children.
Both bacterial and viral conjunctivitis are self-limiting conditions. Good hand hygiene is the most effective way to prevent spread.

156
Q

A 3-year-old girl is being evaluated for an abnormal speech pattern. Her speech is hypernasal, and air escapes from her nose when she talks. She has a history of frequent ear infections. On physical examination, the girl has a long face, widely spaced eyes, and a bifid uvula with a ridge at the end of the hard palate. The remainder of the girl’s physical examination findings are normal.

Of the following, the BEST next step is to evaluate this girl for

A.	cardiac abnormalities
B.	gastroesophageal reflux
C.	hearing loss
D.	immunodeficiency
A

The girl in the vignette has a bifid uvula and clinical findings suggestive of a submucosal cleft palate (SMCP), including hyper-nasal speech, air escape from her nose, and a palatal ridge. The additional findings of hypertelorism and long face suggest an underlying genetic syndrome such as Loeys-Dietz or DiGeorge syndrome. DiGeorge syndrome is less likely because there is no history of neonatal hypocalcemia or infections due to T-cell defects.

The girl in the vignette most likely has Loeys-Dietz syndrome (LDS).

PREP Pearls
Bifid uvula may be associated with a submucosal cleft palate.
Bifid uvula is associated with several genetic syndromes; all children with a bifid uvula should be evaluated for cardiac anomalies.
Chronic recurrent ear infections, nasal regurgitation, and velopharyngeal insufficiency are commonly seen in children with submucosal cleft palate.

157
Q

A 17-year-old, previously healthy adolescent boy is brought to the emergency department with lower extremity weakness. He was walking into his house after checking the mailbox when he experienced the onset of bilateral lower extremity weakness, which he describes as a heaviness. His weakness progressively worsened over several hours to the point that he could not stand up from the dinner table. The boy recalls a self-limited episode of blurred vision 6 months ago, but no other episodes of focal neurological deficits.

On physical examination, the adolescent’s vital signs are normal. His mental status and cranial nerve examination findings are normal. Motor examination reveals normal upper extremity strength, tone, and reflexes; lower extremities are flaccid with 0/5 strength, absent reflexes, and hypotonia. His toes are upgoing to plantar stimulation. He has a sensory level at his mid-thorax, below which he is unable to reliably feel light touch, pin prick, temperature, or vibration. He has normal finger to nose testing but cannot perform heel-to-shin testing due to weakness. He is unable to ambulate. The remainder of his physical examination findings are normal.

Of the following, the BEST next step in this adolescent’s management is

A.	electromyography and nerve conduction studies
B.	lumbar puncture with cerebrospinal fluid analysis
C.	spine computed tomography
D.	spine magnetic resonance imaging
A

The adolescent in the vignette is experiencing an acute myelopathy, a neurological emergency requiring urgent neuroimaging to evaluate for spinal cord compression or inflammation. Magnetic resonance imaging (MRI) of the spine with and without contrast is the best test to evaluate the spinal cord and associated structures. The presence of a compressive lesion requires neurosurgical intervention.

PREP Pearls
Acute spinal cord dysfunction is a neurological emergency requiring careful history, physical examination, and urgent neuroimaging to evaluate for cord compression. Magnetic resonance imaging of the spine with and without contrast is the best test to evaluate the spinal cord and associated structures.
Common symptoms and signs of an acute myelopathy include: bilateral motor and/or sensory symptoms, bladder and/or bowel dysfunction, and neck/back pain.
In the acute phase, children and adolescents with myelopathy can present with a flaccid paralysis and areflexia below the level of the lesion; this may lead to diagnostic confusion due to the absence of upper motor neuron signs.

158
Q

A 12-year-old boy is brought to the emergency department after being bitten by a snake while hunting in Florida. Photos show a dark brown snake with diamond-shaped marks on its back (Item Q229). The boy is ill-appearing and pale. His vital signs are a temperature of 36.7°C, heart rate of 140 beats/min, respiratory rate of 18 breaths/min, and blood pressure of 102/65 mm Hg. Physical examination reveals 2 small puncture wounds on his lower calf, which is swollen, red, ecchymotic, and extremely tender to palpation. The erythema is extending up his calf toward his knee. Shortly after arrival, the boy begins to complain of nausea and has 1 episode of emesis.

Of the following, the BEST next step in the management of this boy is to

A.	administer crotalidae polyvalent immune fab
B.	apply ice to the puncture wounds
C.	apply a tourniquet to the upper thigh to prevent the spread of venom
D.	place the leg in a splint and elevate the leg above the level of the patient’s heart
A

PREP Pearls
Crotalidae polyvalent immune fab is the standard of treatment for children with pit viper snake bite displaying severe local reactions, systemic symptoms, or rapid progression of symptoms. Initial dosing is 4 to 6 vials.
The 2 families of poisonous snakes in the United States are Crotalidae (pit vipers) and Elapidae (eg, coral snakes and cobras).
Crotalidae envenomations cause severe local tissue damage and can progress to systemic hematotoxicity (eg, nausea, vomiting, hypotension, coagulation abnormalities); Elapidae envenomations are neurotoxic with little to no local tissue damage.

159
Q

A 10-year-old boy is seen in the office for 2 days of left ear pain. He has a history of mild intermittent asthma but has not needed albuterol in over 1 year. He has no upper airway symptoms, cough, or fever. The boy was camping last week near a lake where he engaged in swimming and water sports. On physical examination, the boy has pain with traction or movement of his left outer ear. Findings on otoscopic examination of the left ear are shown in Item Q230 yellow ear canal. The remainder of his physical examination findings are normal.

Of the following, the BEST treatment for this boy is

A.	oral analgesics and ear canal barrier protection
B.	topical acetic acid 2% solution
C.	topical antifungal drops
D.	topical combined antibiotic and steroid drops
A

d

The boy in the vignette has no symptoms beyond ear pain, had a recent experience with swimming and water sports, and has otoscopic examination findings (Item C230) typical for otitis externa (external canal edema and debris); this scenario is consistent with a diagnosis of otitis externa. Swimmers have a 5 times increased incidence of otitis externa. The differential diagnosis of an ear canal with this appearance includes otitis media with tympanic membrane rupture, skin conditions such as atopic dermatitis or psoriasis, and a missed foreign body in the canal causing inflammation.

PREP Pearls
Otitis externa often presents with isolated ear pain after a recent exposure to water (eg, swimming, water sports).
Topical combined antibiotic and steroid drops is the first-line treatment of otitis externa.
Ototoxic preparations (eg, aminoglycosides) should not be instilled in the ear canal without first visualizing an intact tympanic membrane.

160
Q

A 3-year-old boy recently adopted from India is seen for profuse watery diarrhea. The boy has HIV infection. His recent laboratory testing showed an HIV viral load of 250,000 copies/mL and an absolute CD4 cell count of 200/μL. He is not receiving antiretroviral therapy. The diarrhea started 3 days after the family visited a recreational waterpark. On physical examination, the boy’s temperature is 38.5°C, heart rate is 90 beats/min, respiratory rate is 22 breaths/min, blood pressure is 100/65 mm Hg, and oxygen saturation is 99% in room air. Stool microscopy reveals acid-fast oocysts.

Of the following, the BEST next step in this boy’s management is to administer

A.	albendazole
B.	antiretroviral therapy
C.	metronidazole
D.	nitazoxanide
A

The child in the vignette has a low CD4 cell count because of untreated HIV infection. The presence of acid-fast oocysts in the stool is consistent with cryptosporidiosis. Diarrheal illness from Cryptosporidium species is associated with recreational water parks and contaminated drinking water. In a child with HIV infection, prompt initiation of antiretroviral therapy (eg, dolutegravir, emtricitabine, and tenofovir combination) to improve the CD4 cell count results in resolution of diarrheal symptoms. Treatment is generally not necessary in otherwise healthy children; however, treatment with oral nitazoxanide is considered in children without HIV infection for severe symptoms (>10 loose stools per 24 hours) or persistent symptoms (>2 weeks).

PREP Pearls
Cryptosporidium is an intracellular parasite responsible for diarrheal illness.
Most Cryptosporidium infections are asymptomatic and self-limiting in immunocompetent children. Severe or protracted illness is seen in children with untreated HIV, solid or stem cell transplantation, or immunodeficiency.
In a child with HIV infection, prompt initiation of antiretroviral therapy can lead to resolution of diarrhea due to Cryptosporidium.

161
Q

A term female neonate is brought to the clinic for a health supervision visit. Her parents report that their 4-month-old son died from sudden unexplained infant death syndrome 2 years ago. They ask how best to prevent a similar event happening in their daughter.

Of the following, the BEST recommendation for this neonate’s parents is to

A.	avoid feeding her with a bottle
B.	avoid smoke exposure
C.	place her to sleep in the side position
D.	use home apnea and bradycardia monitoring
A
162
Q

A 15-year-old adolescent girl is seen in the clinic for follow-up of anemia. She was initially evaluated 1 month ago for a 6-month history of fatigue. At that visit, her hemoglobin level was 9.0 g/dL (90 g/L), and she was prescribed a daily iron supplement. The adolescent often skips breakfast and goes with her friends to the corner store for “lunch,” which usually consists of chips and iced tea. She eats a home-cooked dinner with her parents every evening. She has regular menstrual periods which she describes as moderately heavy, requiring more than 4 pads during the day and lasting 7 days. The adolescent’s physical examination findings are normal except for pallor of her palpebral conjunctiva. A repeat hemoglobin level today is 9.2 g/dL (92 g/L).

Of the following, the BEST next step in this adolescent’s management is to

A.	assist her with strategies to improve medication adherence
B.	obtain laboratory data to evaluate her iron levels C.	provide her with a detailed list of iron-containing foods
D.	start an oral contraceptive to prevent excessive menstrual blood loss
A

b

The adolescent in the vignette has anemia and multiple risk factors for iron deficiency including a diet deficient in iron-containing foods and heavy monthly menses. However, her hemoglobin level did not improve 1 month after starting an iron supplement. The best next management step is to obtain laboratory testing to evaluate her iron levels to confirm the diagnosis of iron-deficiency anemia.

Most likely, the lack of improvement in the adolescent’s hemoglobin level is due to non-adherence with her iron supplement. However, it is important to ensure that there is not an alternative etiology of her anemia before focusing solely on methods to increase her iron intake (eg, a detailed list of iron-containing foods, strategies to improve medication adherence). Decreasing her menstrual flow with an oral contraceptive is unlikely to correct the current iron deficiency when there is poor dietary iron intake.

163
Q

A 17-year-old adolescent girl is evaluated for a 3-week history of foul-smelling vaginal discharge and mild vaginal itching. She has had 1 lifetime sexual partner and uses condoms inconsistently. She was evaluated at the local health department 1 week ago where testing results for sexually transmitted infections, including chlamydia, gonorrhea, trichomonas, and HIV, were negative. On speculum examination, there is a homogeneous, white vaginal discharge with a fish-like odor, and a normal cervix. There is no tenderness on the bimanual examination. The remainder of her physical examination findings are normal. The vaginal discharge is further evaluated.

Of the following, the MOST likely laboratory finding for this adolescent is

A.	>10 white blood cells/HPF on a wet preparation
B.	budding hyphae on a 10% potassium hydroxide preparation
C.	multiple clue cells on a wet preparation
D.	vaginal pH <4.5 on pH paper
A

The adolescent in the vignette has signs and symptoms consistent with bacterial vaginosis (BV). The gold standard test for BV is the Nugent score of a vaginal fluid Gram stain. In clinical practice, BV can be diagnosed by the presence of at least 3 of the following Amsel criteria:

Homogeneous, thin vaginal discharge that smoothly coats the vaginal walls
Clue cells (vaginal epithelial cells covered with adherent bacteria) on microscopic examination (Item C234A)
pH of vaginal fluid >4.5 (normal vaginal pH is 3.8 to 4.3)
Fishy odor of vaginal discharge before or after addition of 10% potassium hydroxide (KOH; ie, the whiff test)

First-line treatment for bacterial vaginosis is metronidazole orally or intravaginally.

164
Q

A 7-year-old girl is seen in the clinic to establish care. Review of her vaccination records reveals that she received the diphtheria, tetanus, pertussis (DTaP) vaccine at 2, 4, and 6 months but did not receive subsequent boosters. In order to enter school, her tetanus, diphtheria, and pertussis vaccination status must be up to date.

Of the following, the MOST appropriate vaccine to administer today is

A.	diphtheria and tetanus (DT)
B.	diphtheria, tetanus, pertussis (DTaP)
C.	tetanus and diphtheria (Td)
D.	tetanus, diphtheria, pertussis (Tdap)
A

d

PREP Pearls
During childhood, a total of 6 doses of a pertussis-containing vaccine are recommended: diphtheria, tetanus, pertussis (DTaP) in a 5-dose series, typically at ages 2, 4, 6, and 15 to 18 months, and 4 to 6 years; and 1 dose of tetanus, diphtheria, pertussis (Tdap) at age 11 to 12 years.
For children and adolescents aged 7 to 18 years who are not fully vaccinated against tetanus, diphtheria, and pertussis, the preferred first vaccine in the catch-up series is tetanus, diphtheria, pertussis (Tdap).

165
Q

A term neonate had Apgar scores of 8 and 9 at 1 and 5 minutes, respectively, after birth. After delivery, he was active and crying, with room air oxygen saturations ranging from 93% to 97%. Approximately 10 minutes after birth, when he was quiet and calm, the neonate became cyanotic, his oxygenation saturation level decreased to 65% to 70%, and he developed significant respiratory distress with subcostal retractions and nasal flaring. He was treated with bag/mask positive-pressure ventilation using inspired fractionated oxygen of 100% without significant improvement in clinical status. He was, therefore, intubated, and positive-pressure ventilation was continued. The neonate’s clinical status improved almost immediately after intubation. His oxygen saturation increased to 100%; oxygen supplementation was quickly weaned and discontinued. He was placed on continuous positive airway pressure of 5 cm H2O. He is now pink, well perfused, and breathing comfortably in room air, with oxygen saturations of 95% to 98%.

Of the following, the BEST next step in this neonate’s management is to

A.	obtain an arterial blood gas
B.	obtain echocardiography
C.	pass a nasogastric tube through both nares
D.	place an umbilical venous catheter
A

The neonate in the vignette most likely has bilateral choanal atresia. After delivery, he had no respiratory distress and had normal oxygen saturation levels. He quickly developed oxygen desaturation and respiratory distress when he became quiet and calm. The best next step in his management would be to pass a nasogastric tube through both nares. In the case of bilateral choanal atresia, the physician would be unable to pass the tube through either naris.

PREP Pearls
Bilateral choanal atresia in a neonate is a medical emergency because neonates are obligate nose breathers.
Passage of a nasogastric tube through both nares should be attempted in a neonate with suspected choanal atresia.
A neonate with choanal atresia or stenosis should undergo evaluation for associated anomalies, such as those seen in CHARGE syndrome (Coloboma, Heart defects, Atresia or stenosis of the choanae, Retardation of growth and development, Genitourinary anomalies, and Ear anomalies [including hearing loss]).

166
Q

A 15-year-old adolescent girl is evaluated in the clinic for a 4-week history of worsening back pain. She has competed in gymnastics for the past 5 years but is currently limited by pain, especially with back extension. There is no history of trauma. She does not have numbness, tingling, or changes in her bowel or bladder function. On physical examination, the adolescent has pain with back extension. Back flexion and rotation are normal and pain free. She has normal strength in her arms and legs. The remainder of the adolescent’s physical examination findings are unremarkable.

Of the following, the BEST imaging modality to evaluate this adolescent’s findings is

A.	a bone scan
B.	computed tomography
C.	magnetic resonance imaging
D.	plain radiography
A

The adolescent in the vignette is a gymnast and has pain with back extension. Spondylolisthesis is the most likely diagnosis. This condition can usually be identified on plain radiography. If plain radiographs are normal and there is a high degree of suspicion, magnetic resonance imaging (MRI) is the next best option. Computed tomography (CT) and a bone scan may be indicated in the evaluation of back pain due to other etiologies (eg, primary or metastatic bone tumor and injury) but would not be the best imaging options to evaluate the adolescent in the vignette.

167
Q

A 6-year-old girl is seen in the office for evaluation of recurrent urinary tract infections (UTIs). She has had 4 UTIs in the past 6 months. With each infection, she was seen with dysuria, urinary urgency, and suprapubic pain. She has not had fever, nausea, vomiting, back pain, diarrhea, or hematuria. During each episode, a urinalysis was significant for 2+ leukocyte esterase and positive for nitrites, and each urine culture yielded more than 100,000 CFU of Escherichia coli. After appropriate treatment of each infection, her symptoms resolved, posttreatment urinalysis results were normal, and a urine culture result was negative. Renal ultrasonography and a voiding cystourethrography had normal results. The girl is developmentally appropriate. She has a bowel movement once a day that is usually hard and painful. Her review of systems is otherwise negative. The girl’s vital signs, growth parameters, and physical examination findings are normal.

Of the following, the BEST next step in this girl’s management is to

A.	administer prophylactic antibiotics
B.	optimize bowel regimen
C.	provide reassurance
D.	refer to a urologist
A

The girl in the vignette has recurrent urinary tract infections (UTIs) most likely secondary to bowel bladder dysfunction; therefore, the best next management step is to optimize her bowel regimen. Prophylactic antibiotics will not address her underlying condition (chronic constipation), are unlikely to prevent her UTIs in the long term, and may contribute to antibiotic resistance. Referral to a urologist may be necessary if she continues to have recurrent UTIs after optimization of her bowel regimen. Reassurance alone is inadequate because she is likely to have recurrent UTIs until her chronic constipation is successfully treated.

PREP Pearls
Bowel and bladder dysfunction resulting in inadequate bladder emptying is a common but underrecognized cause of recurrent urinary tract infections.
Renal scarring may occur after recurrent urinary tract infections but does not usually lead to long-term sequelae in the absence of anatomical urinary system abnormalities.
If bacteria enter the bloodstream from the urinary tract, urosepsis and meningitis may occur; neonates and immunocompromised children are at higher risk for systemic infection.

168
Q

A term female newborn is seen in the neonatal intensive care unit. She undergoes intubation shortly after delivery for respiratory distress. She is small for gestational age with a birthweight of 2,000 g. Physical examination findings include cleft lip and palate, a scalp defect, microcephaly, and polydactyly of the hands bilaterally. Two-dimensional echocardiography shows an atrial septal defect, and brain magnetic resonance imaging shows holoprosencephaly.

Of the following, the results of this neonate’s karyotype would MOST likely be

A.	45,X
B.	47,XX, +13
C.	47,XX, +18
D.	47,XX, +21
A

The neonate in the vignette has findings consistent with trisomy 13; testing would most likely result in a karyotype of 47,XX, +13. Neonates with trisomy 13 exhibit intrauterine growth restriction and are small for gestational age. Clinical features may include:

Midline defects: Cleft lip and/or palate, cyclopia
Brain anomalies: Holoprosencephaly, microcephaly
Scalp defect: Cutis aplasia
Cardiac defects: Atrial septal defect, ventricular septal defect, tetralogy of Fallot, double outlet right ventricle
Eye anomalies: Micro-ophthalmia, anophthalmia
Hernia: Inguinal, umbilical
Renal: Polycystic kidney, hydronephrosis, horseshoe kidney
Musculoskeletal: Postaxial polydactyly, congenital talipes equinovarus

169
Q

A 16-year-old adolescent boy is brought to the emergency department for evaluation of right shoulder pain. He was playing baseball and collided with another player while sliding into a base. He felt immediate pain in his right shoulder and had limited movement of his right arm due to the pain. On physical examination, there is tenderness to palpation over his right clavicle with crepitus and swelling of the mid-clavicular area. Adduction and flexion of the right shoulder are limited due to pain. There is no skin tenting. His right arm is neurovascularly intact. The remainder of the adolescent’s physical examination findings are unremarkable. Appropriate analgesia is administered, and right shoulder and clavicle radiographs are obtained (Item Q241A and Item Q241B). nondisplaced clavicle fracture

Of the following, the BEST next step in this adolescent’s management is

A.	closed reduction, analgesia, immobilization, and follow-up with an orthopedic surgeon in 4 weeks
B.	rest, ice, compression bandage, analgesia, and follow-up with an orthopedic surgeon in 2 weeks
C.	urgent referral to an orthopedic surgeon for open reduction and internal fixation
D.	use of arm sling, analgesia, and follow-up with the pediatrician in 2 weeks
A

PREP Pearls
The middle third of the shaft is the most common location of a clavicle fracture; it can be treated conservatively with use of an arm sling, analgesia, and follow-up with a pediatrician.
Urgent orthopedic surgery consultation is necessary for an open clavicle fracture, sternoclavicular dislocation, displaced fractures of the distal third of the clavicle, tenting of the skin over the clavicle, or evidence of neurovascular compromise.
Fractures of the proximal third of the shaft are the least common type of clavicular fracture and are often associated with serious chest, lung, neck, and vascular injury.

170
Q

A previously healthy, 3-year-old boy is brought to an urgent care center with a 2-day history of fever, right leg and hip pain, and a limp. Today, he is unable to bear weight on the right leg. One week ago, the boy and several family members had a dry cough and nasal congestion. There is no history of trauma. His immunizations are up to date.

The boy is irritable with a temperature of 39.4°C, heart rate of 164 beats/min, respiratory rate of 32 breaths/min, and blood pressure of 110/56 mm Hg. He is lying on the examination table with his right lower extremity abducted and externally rotated. He cries when his right hip is internally or externally rotated. There is no swelling, redness, or warmth over the right hip. The remainder of the boy’s physical examination findings are normal.

Laboratory data are shown:

Laboratory Test

Result

White blood cell count

21,000/μL (21.0 × 109/L)

Segmented neutrophils

76%

Lymphocytes

20%

Monocytes

4%

C-reactive protein

11.0 mg/dL (110 mg/L)

Erythrocyte sedimentation rate

50 mm/h

A radiograph of the right hip is normal.

Of the following, the BEST next step in this boy’s evaluation is

A.	a blood culture
B.	magnetic resonance imaging
C.	a respiratory virus panel
D.	ultrasonography
A

In the vignette, the boy’s signs and symptoms of fever, limp, pain with internal and external rotation of the right hip, leukocytosis, and elevated C-reactive protein (CRP) level support a diagnosis of pyogenic arthritis of the hip joint. The best next step in his evaluation is ultrasonography of the right hip to evaluate for an effusion. If an effusion is detected, the joint should be aspirated as soon as possible to confirm the diagnosis and obtain fluid to send for cell count and culture. A synovial fluid white blood cell count (WBC) of ≥50,000/μL (≥50.0 × 109/L) with neutrophilic predominance is highly suggestive of pyogenic arthritis. Urgent orthopedic consultation, surgical drainage, and irrigation of the hip joint space is required given the risk of aseptic necrosis of the femoral head from increased intra-articular pressure and compromised blood flow. Joint fluid cultures are positive for a bacterial pathogen in 60% of cases. Polymerase chain reaction of the synovial fluid may be positive for fastidious organisms such as Kingella kingae.

A blood culture is positive for a bacterial pathogen in 40% of cases of pyogenic arthritis. While results of a blood culture may be helpful in guiding antibiotic treatment, making the diagnosis and surgical draining of a septic joint are the first priority.

171
Q

A 14-year-old adolescent girl is seen in the emergency department for evaluation of worsening epigastric abdominal pain of 6 hours’ duration. She has had several episodes of nonbilious, nonbloody emesis and states that deep inspiration is difficult due to pain. She denies dysphagia, diarrhea, and constipation. She has had recurrent episodes of similar, but less severe, epigastric pain over the past month. The adolescent has had a history of dysmenorrhea since her onset of menses at age 13 years, for which she takes ibuprofen a few times monthly. She denies the use of any other medication. She had an appendectomy for acute appendicitis at age 8 years.

The girl is afebrile with a heart rate of 100 beats/min, respiratory rate of 18 breaths/min, weight of 80 kg, height of 155 cm, and a body mass index of 32 kg/m2. She appears uncomfortable but not in acute distress. There is scleral icterus. Her abdomen is soft, nondistended, and without hepatosplenomegaly. There is tenderness to palpation in the epigastrium, and palpation of the right upper quadrant during inspiration results in worsening pain. Her extremities are warm, well perfused, and without edema.

Of the following, the factor MOST likely contributing to this girl’s underlying condition is

A.	age of menarche
B.	nonsteroidal anti-inflammatory drug use
C.	obesity
D.	previous appendectomy
A

The adolescent in the vignette has abdominal pain, a positive Murphy sign (pain associated with palpation of the right upper quadrant during inspiration), and scleral icterus; this clinical scenario is suggestive of choledocholithiasis. Of the response options, obesity is the factor that places the girl at risk for the development of cholelithiasis. Age of menarche, use of nonsteroidal anti-inflammatory drugs, and history of previous appendectomy are not risk factors for the development of cholelithiasis.

172
Q

A 7-year-old girl is brought to the office by her foster mother to establish care. The girl requires constant supervision and assistance with most activities of daily living. Her foster mother reports, “She acts like my 4-year-old. I have to help dress her and she never sits still.” She is struggling academically; a full educational evaluation is scheduled.

During the visit, the girl is very active and has difficulty remaining seated. She is able to provide her full name and age and speaks in 3- to 4-word sentences. Her height and weight are below the 10th percentile for age. The girl’s physical examination findings are remarkable for short palpebral fissures, a thin upper lip, and a smooth philtrum. The remainder of her physical examination findings are normal.

Of the following, the MOST likely historical finding for this girl is maternal

A.	alcohol use
B.	anticonvulsant medication use
C.	cytomegalovirus infection
D.	exposure to cat feces
A

The girl in the vignette has impairments in cognitive and adaptive functioning consistent with an intellectual disability and facial features characteristic of fetal alcohol syndrome. The following 3 criteria must be met to diagnose fetal alcohol syndrome:

Prenatal and/or postnatal growth deficiency
Characteristic facial features including reduced palpebral fissure length, smooth philtrum, and thin upper lip border
Neurological or neurobehavioral deficits

173
Q

A 16-year-old adolescent boy is seen for evaluation of testicular swelling. He was born at 28 weeks’ gestation and has cerebral palsy with spastic quadriplegia and a seizure disorder. At 9 months of age he underwent a bilateral inguinal hernia repair with right orchiopexy. His mother initially noticed right testicular swelling 2 months ago during a diaper change, and the testicle has continued to increase in size. He does not have pain, fever, vomiting, diarrhea, hematuria, or urinary symptoms. Physical examination findings are significant for spasticity of his extremities, nontender right scrotal swelling, and a 4-cm, firm, nontender mass in the right testicle. The remainder of his physical examination findings are unremarkable.

Of the following, the risk factor MOST likely to be associated with this boy’s findings is

A.	cerebral palsy
B.	cryptorchidism
C.	seizure disorder
D.	urinary incontinence
A

The adolescent in the vignette has a painless testicular mass that is increasing in size. Testicular cancer is the most likely diagnosis. Cryptorchidism is a major risk factor for the development of testicular cancer. Infants with cryptorchidism, such as the boy in the vignette who had an orchiopexy at 9 months of age, are up to 8 times more likely to develop testicular cancer even after the testis is placed securely in the scrotum. Cryptorchidism is the risk factor most likely associated with this adolescent’s testicular cancer. It is important to note that premature infants have a high incidence of cryptorchidism. Cerebral palsy, seizure disorder, and urinary incontinence are not associated with an increased risk of developing testicular cancer.

174
Q

A 4-month-old infant is brought to the emergency department after 2 episodes in the last 24 hours of color change, limpness, and cessation of breathing. His mother gave rescue breaths during the last episode and reports that he was limp for more than a minute. There was a similar but less frightening episode 1 week ago that resolved when she blew in his face. He has been more fussy than usual in the last few days but has not shown any other signs of illness.

The infant was born at term after an uncomplicated pregnancy. His newborn screening results were normal. At each of the recommended routine health supervision visits he was noted to be growing and developing well. The infant breastfeeds or takes expressed breast milk. He is not in daycare and stays with his mother’s boyfriend while she works an evening shift.

On physical examination, the infant is alert and breathing normally. His temperature is 37°C, heart rate is 140 beats/min, respiratory rate is 25 breaths/min, and oxygen saturation is 99% in room air by pulse oximetry. There is a bruise on his right upper arm that his mother attributes to a bite from an older sibling. The remainder of his physical examination findings are normal.

Of the following, the BEST next step in this infant’s management is to

A.	obtain computed tomography scan of the head
B.	obtain a nasal swab for respiratory syncytial virus
C.	order home apnea monitoring for 30 days
D.	reassure the mother and schedule follow-up for the next day
A

The infant in the vignette has had 3 episodes that meet criteria for a brief resolved unexplained event (BRUE). Because of the recurrence, duration of >1 minute, and concerning history and physical examination findings, he does not meet criteria for lower-risk BRUE and further evaluation is indicated. Reassurance and follow-up is not appropriate. Potential life-threatening etiologies for his presentation include central nervous system (CNS) abnormalities and abusive head trauma. Thus, the best next step in his management is to obtain computed tomography of the head. A high index of suspicion for nonaccidental trauma should be maintained. Concerning findings include the infant’s fussiness and the bruise on his arm in the context of an otherwise unexplained recurrent potentially life-threatening event.

PREP Pearls
For infants with new or recurrent episodes of apnea with color change and/or loss of tone, without evidence of an underlying condition, a high index of suspicion must be maintained for non-accidental head trauma.
The most common cause of obstructive sleep apnea in children is tonsillar and/or adenoidal hypertrophy.
Children with neuromuscular disorders, skeletal dysplasias, and craniofacial anomalies are at high risk for complicated obstructive sleep apnea.

175
Q

A 4-month-old infant who is new to the practice is seen for a health supervision visit. This visit is his first follow-up since leaving the newborn nursery. The infant was born at term without complications. His physical examination findings are significant only for the extremity abnormality shown in Item Q248. clubfoot

Of the following, the BEST next management step for this infant is to

A.	have him return for reevaluation in 1 month
B.	obtain bilateral foot and ankle radiographs
C.	refer him to a pediatric orthopedic surgeon
D.	refer him to physical therapy
A

The infant in the vignette has congenital idiopathic talipes equinovarus, commonly called clubfoot (Item C248A). The best next step in management is to refer him to a pediatric orthopedic surgeon for treatment. This condition will not improve without treatment, which should begin as soon as possible, so having him return for reevaluation in 1 month is not the preferred response. Imaging is not routinely recommended for this condition, thus foot and ankle radiographs would not be the appropriate next step. Physical therapy alone will not correct the deformity; timely treatment by a pediatric orthopedic surgeon is required.

PREP Pearls
Clubfoot is an inward rotational anomaly of the foot at the ankle with an associated increase in the midfoot crease and a shortened calf muscle on the affected side.
The mnemonic CAVE is used to describe the features of clubfoot: cavus (high arch), adducted position of the forefoot, hindfoot varus (heel curves in towards the midline), and equinus (ankle stuck in a plantarflexed position).
It is important to promptly refer infants with clubfoot to a pediatric orthopedic surgeon.

176
Q

A 14-year-old adolescent girl is brought to the emergency department after she sustained a head injury when she collided with another player while playing soccer. She had a brief loss of consciousness at the time of the event and later developed somnolence, headache, and vomiting. In the emergency department, other than a tired appearance, the adolescent’s physical examination findings are normal. Head computed tomography (CT) scan without contrast demonstrates a hyperdensity in the right frontal lobe with no mass effect. Brain magnetic resonance imaging and angiography, performed to further evaluate the frontal lobe CT finding, demonstrate a tightly packed tangle of blood vessels, with clearly visualized flow voids, within the right frontal lobe without evidence of hemorrhage.

Of the following, the MOST likely diagnosis for this adolescent is

A.	arachnoid cyst
B.	arterio-venous malformation
C.	cavernous malformation
D.	meningioma
A

The adolescent in the vignette has imaging findings consistent with a cerebral arterio-venous malformation (AVM). Arterio-venous malformations are congenital high-flow vascular malformations in which there is a direct connection between the arteries and veins and no intervening capillary bed. The majority of AVMs are caused by sporadic pathologic genetic variants resulting in dysregulated vasculogenesis. However, there are genetic syndromes associated with development of AVMs, most commonly hereditary hemorrhagic telangiectasias.

Cavernous malformations (CM) are slow-flow vascular malformations consisting of a tangled, enlarged, abnormal capillary bed without intervening brain parenchyma. Cavernous malformations may be familial or sporadic. Hemorrhage may occur. Common symptoms of CM include headache, seizures, vision changes, and balance problems. On MRI, cavernous malformations have a “popcorn” or “mulberry-like” cluster of tiny blood vessels with surrounding hemosiderin deposits. Unlike AVMs, there is no intervening brain parenchyma. The imaging appearance of the child in the vignette’s vascular lesion is not consistent with a CM.

PREP Pearls
Arterio-venous malformations are congenital, high-flow, vascular malformations in which there is a direct connection between the arteries and veins and no intervening capillary bed.
The majority of arterio-venous malformations are diagnosed incidentally, during a diagnostic evaluation for seizures, headaches, or progressive focal neurological deficits, or acutely, in the setting of intracranial hemorrhage.
Manifestations of symptomatic arteriovenous malformations vary based on age, location, size, and the features of the lesion itself.

177
Q

An 11-year-old boy is seen for a health supervision visit. His family history is significant for 2 relatives who suffered myocardial infarctions in their 50s and 2 grandparents with type 2 diabetes mellitus. On physical examination, the boy’s blood pressure is 104/62 mm Hg and body mass index is 23.8 kg/m2 (96th percentile). He has acanthosis nigricans over the nape of his neck. His sexual maturity rating is 2 for pubic hair and genital development. The remainder of his physical examination findings are normal.

A fasting laboratory evaluation reveals the following:

Laboratory Test

Result

Total cholesterol

220 mg/dL (5.7 mmol/L)

Triglycerides

120 mg/dL (1.4 mmol/L)

High-density lipoprotein (HDL) cholesterol

22 mg/dL (0.6 mmol/L)

Low-density lipoprotein (LDL) cholesterol

150 mg/dL (3.9 mmol/L)

Glucose

92 mg/dL (5.1 mmol/L)

Aspartate aminotransferase

22 U/L

Alanine aminotransferase

24 U/L

Of the following, the BEST next step in this boy’s management is to initiate

A.	atorvastatin
B.	lifestyle modification
C.	metformin
D.	orlistat
A

b

Statin therapy (eg, atorvastatin) is recommended for those aged 10 years and older whose LDL cholesterol level remains above goal after lifestyle modification. For those with a family history of cardiovascular disease, 1 high-level risk factor, or 2 or more moderate-level risk factors, the threshold for starting statin therapy is an LDL cholesterol level ≥160 mg/dL (4.1 mmol/L). The treatment goal is an LDL level <130 mg/dL (3.4 mmol/L). For those at higher risk, the threshold for statin treatment is an LDL level ≥130 mg/dL (3.4 mmol/L). The treatment goal is an LDL level <100 mg/dL (2.6 mmol/L). Statins inhibit hydroxymethylglutaryl-coenzyme A (HMG-CoA) reductase, the rate-limiting enzyme in cholesterol synthesis, resulting in decreased endogenous production of cholesterol, upregulation of LDL receptors in the liver, and increased clearance of LDL cholesterol.

178
Q

A 14-year-old adolescent girl is seen in the emergency department for evaluation of lower abdominal pain. The pain radiates to her lower back and is associated with nausea. She had an episode of vomiting immediately before coming to the emergency department. Menarche occurred 6 months ago, and she has had monthly cycles with bleeding lasting 5 to 7 days. Her last menstrual period began yesterday. She has never had sexual intercourse and denies dysuria and vaginal discharge. She takes no medications. The girl’s vital signs are normal for age. Her physical examination findings are significant only for mild tenderness to palpation of the lower abdomen without rebound or guarding. The result of a urine pregnancy test is negative.

Of the following, the BEST next step in this adolescent’s management is to

A.	perform a speculum examination with a chaperone
B.	perform transabdominal pelvic ultrasonography
C.	prescribe a combined oral contraceptive pill
D.	prescribe a nonsteroidal anti-inflammatory medication
A

The adolescent in the vignette is experiencing lower abdominal pain during menstruation consistent with dysmenorrhea. Primary dysmenorrhea, menstrual pain in the absence of any underlying pathologic process, is the most common type of dysmenorrhea in adolescents. The first-line treatment of primary dysmenorrhea is a nonsteroidal anti-inflammatory medication (NSAID) such as ibuprofen or naproxen sodium.

Pelvic examination with a speculum (with a chaperone present) and testing for sexually transmitted infections should be performed in sexually active adolescents with severe dysmenorrhea.

179
Q

A 16-year-old adolescent with a 7-year history of type 1 diabetes is seen for concerns about more frequent hypoglycemia detected on their continuous glucose monitor for the past 2 weeks. Their hemoglobin A1c level was 7.8% 2 months ago, and there have not been any recent changes in their insulin regimen or diet. They have participated in dance for the past 10 years, but missed multiple classes recently because of easy fatigability and low blood glucose values. They deny recent illness or depression.

On physical examination, the adolescent’s heart rate is 102 beats/min, blood pressure is 96/58 mm Hg, and oxygen saturation is 99% in room air. Weight is 2.3 kg less than at a visit 2 months ago. There is hyperpigmentation in the creases of the hands. The remainder of the physical examination findings are unremarkable.

Of the following, this adolescent’s MOST likely diagnosis is

A.	autoimmune adrenalitis
B.	celiac disease
C.	an eating disorder
D.	hypothyroidism
A

The adolescent in the vignette has autoimmune adrenalitis, or Addison disease. The history of type 1 diabetes (autoimmune risk factor), new hypoglycemia, fatigue, tachycardia, low blood pressure, and hyperpigmentation in the hand creases are all suggestive of primary adrenal insufficiency. The diagnosis would need to be confirmed by laboratory evaluation.

PREP Pearls
Additional autoimmune conditions, including autoimmune hypothyroidism and celiac disease, commonly occur in individuals with type 1 diabetes.
New onset of celiac disease and Addison disease can cause unexpected hypoglycemia in children and adolescents with type 1 diabetes.
Hypoglycemia in children and adolescents with type 1 diabetes is commonly caused by too high of an insulin dose for the basal requirements, carbohydrate intake, and/or activity level.

180
Q

A 7-year-old child is brought to the office for a left-sided neck mass present for the past 7 weeks. The neck mass was evaluated 4 weeks ago and was treated with a 7-day course of amoxicillin. Vital signs are normal for age. There is a mildly tender, non-fluctuant, left-sided cervical neck mass. The size of the mass is unchanged from 4 weeks ago. The remainder of the physical examination findings are unremarkable.

Of the following, the BEST next step in this child’s management is

A.	complete blood count and viral serology testing
B.	placement of a purified protein derivative test
C.	referral to a surgeon for biopsy of the mass

D. ultrasonography of the mass

A

The child in the vignette has chronic cervical lymphadenopathy. Lymphadenopathy persisting for >6 weeks is considered chronic, 2 to 6 weeks is subacute, and <2 weeks is acute. The best next step in the evaluation of this child’s chronic cervical lymphadenopathy is to obtain a complete blood count, inflammatory markers, and Epstein–Barr virus (EBV), cytomegalovirus (CMV), and Bartonella henselae serology.

Ultrasonography of an enlarged lymph node may assist in identifying an abscess, defining the anatomy of the mass, differentiating a malignant versus benign process, and developing strategies for biopsy. Ultrasonography will not determine the specific etiology of lymphadenopathy and is therefore not the best next step in management for the child in the vignette.

PREP Pearls
Reactive cervical lymphadenopathy from a viral infection is the most common cause of lymphadenopathy in children.
Epstein–Barr virus, cytomegalovirus, and Bartonella henselae (cat scratch disease) are common causes of chronic cervical lymphadenopathy in children.
Neoplasms are rare in children, but must be considered in the differential diagnosis of chronic lymphadenopathy. Non-Hodgkin lymphoma is more common in children younger than 6 years, and Hodgkin lymphoma is more common in children older than 6 years.

181
Q

A 14-day-old neonate born at 26 weeks’ gestation is in the neonatal intensive care unit. He remains intubated, is receiving mechanical ventilatory support, and has a fractional inspired oxygen concentration of 80% to 90%. He is receiving nothing by mouth because of gastric feeding intolerance. On physical examination, he is tachycardic with a continuous machinelike murmur audible throughout the precordium. Chest radiography shows increased pulmonary vascular markings bilaterally. His creatinine level has been slowly increasing during the last 2 days. The decision is made to treat the neonate’s condition pharmacologically.

Of the following, the BEST treatment for this neonate is

A.	acetaminophen, by activating calcium channels and vasoconstricting
B.	acetylsalicylic acid, by blocking calcium channels and vasodilating
C.	ibuprofen, by activating cyclooxygenase-1 and increasing prostaglandin synthesis
D.	indomethacin, by inhibiting cyclooxygenase-1 and decreasing prostaglandin synthesis
A

The premature neonate in the vignette has signs of pulmonary overcirculation (prolonged need for mechanical ventilatory support and increased pulmonary vascular markings on chest radiograph) and decreased systemic perfusion (tachycardia, increasing creatinine levels, and feeding intolerance) caused by a patent ductus arteriosus (PDA).

PREP Pearls
The ductus arteriosus is a blood vessel that anatomically connects the pulmonary artery to the aorta to allow blood to bypass the lungs in fetal life.
Nonsteroidal anti-inflammatory drugs (indomethacin and ibuprofen) and acetaminophen decrease synthesis of prostaglandins, leading to vasoconstriction and subsequent closure of a patent ductus arteriosus.
There is significant practice variation regarding treatment of a hemodynamically significant persistently patent ductus arteriosus.

182
Q

An 18-month-old boy is seen for a health supervision visit. His parents report that he sat at 12 months of age and crawled 15 months of age. The boy started to exhibit seizures at 10 months of age, which have become more frequent. He recently stopped crawling. Physical examination findings are significant for hypotonia and an exaggerated startle response. A dilated funduscopic examination shows a cherry-red spot on the retina.

Of the following the diagnosis MOST consistent with the boy’s clinical features is

A.	Fabry disease
B.	mucopolysaccharidosis type I
C.	Pompe disease
A

The boy in the vignette, with gross motor delay and regression, seizures, hypotonia, exaggerated startle response, and cherry-red spot on his retina, has Tay-Sachs disease (TSD). Tay-Sachs disease is part of a continuum of disorders caused by deficiency of the enzyme β-hexosaminidase A (HEXA), which is needed to break down GM2 ganglioside. In the absence of this enzyme, GM2 ganglioside is aggregated in the lysosomes of brain and nerve cells, leading to the clinical features of the disease. The spectrum of disorders caused by HEXA deficiency includes acute infantile, subacute juvenile, and late-onset TSD. Clinical features expressed depend on the amount of enzyme present. The child in the vignette is seen with acute infantile TSD.

PREP Pearls
Tay-Sachs disease is an autosomal recessive disorder characterized by gross motor delay, seizures, developmental regression, increased startle response, and a cherry-red spot on the retina.
Management of Tay-Sachs disease is supportive with interventional therapies, control of seizures, and nutritional support.

183
Q

A 5-year-old boy is brought to the emergency department with a nosebleed. On arrival, he has active bleeding from the right naris. Pressure is applied for 10 minutes, and the bleeding resolves. The boy’s mother reports that he has frequent nosebleeds. They occur at least once per month and usually resolve with applied pressure. He does pick his nose frequently, and his mother is concerned about the number of nosebleeds he is experiencing. There is no family history of bleeding disorders. The boy has a temperature of 37.4°C, heart rate of 84 beats/min, respiratory rate of 20 breaths/min, blood pressure of 108/70 mm Hg, and oxygen saturation of 99% in room air. His physical examination findings are normal except for dried blood in the anterior portion of the right naris.

Of the following, the BEST next step in this boy’s management is to

A.	obtain a complete blood count and coagulation studies
B.	perform nasal cautery
C.	provide reassurance and recommend conservative measures
D.	refer to an otolaryngologist
A

The boy in the vignette experienced a nosebleed that resolved after a brief period of applying pressure. The best next management step is to provide reassurance and discuss conservative measures that include applying pressure in the appropriate location for 5 to 10 minutes without interruption, frequent moisturizing of the nasal mucosa with nasal saline or ointment, humidifying the air in the home, and discouraging nose picking.

PREP Pearls
Nosebleeds are common in children and most can be treated with conservative management such as applying pressure in the appropriate location for 5 to 10 minutes without interruption, moisturizing the nasal mucosa with nasal saline or ointment, providing humidified air in the home, and discouraging nose picking.
The differential diagnosis of epistaxis includes trauma, inflammation, dry air, neoplasms, intranasal drugs, structural abnormalities, and bleeding disorders.
Nosebleeds are divided into 2 categories, anterior and posterior, based on the location of the bleeding. Anterior nosebleeds are the most common type and are much easier to visualize and control than posterior nasal bleeds.

184
Q

A previously healthy, 16-month-old girl is brought to the emergency department by ambulance for burns on her face and mouth sustained from drinking a lye solution that her mother uses to make soap. The girl’s mother found her crying in great pain with redness and swelling around her mouth. She rinsed the affected area with water and called 911. In the emergency department the girl’s vital signs include a temperature of 37.5°C, heart rate of 165 beats/min, respiratory rate of 38 breaths/min, and oxygen saturation of 98% in room air. On physical examination she has significant facial irritation and lip edema (Item Q257), vocal hoarseness, drooling, nasal flaring, and audible stridor. The remainder of her physical examination findings are unremarkable.

Of the following, the BEST next step in this girl’s management is

A.	chest radiography
B.	endotracheal intubation
C.	esophagogastroduodenoscopy
D.	nasogastric tube placement
A

The girl in the vignette ingested lye, a caustic alkali, and is displaying signs and symptoms of upper airway edema (mouth and lip swelling, vocal hoarseness) and impending airway compromise (drooling, stridor). The best next management step is endotracheal intubation to secure her airway.

PREP Pearls
Alkali ingestions cause tissue liquefaction necrosis resulting in greater injury compared to acid ingestions.
Symptoms of airway injury due to alkali ingestion, including drooling, mouth pain, lip swelling, and stridor, should raise concern for impending airway compromise; a secure airway should be ensured.

Symptomatic, clinically stable children should have esophagogastroduodenoscopy performed within 48 hours of alkali ingestion; current guidelines support a watchful observation period for asymptomatic or minimally symptomatic children.

185
Q

An 11-year-old, previously healthy boy is brought to the office for evaluation of fever, malaise, rhinorrhea, and a cough for 7 days. Two days ago, he developed painful blisters on his lips and oral mucous membranes. He is not taking any medications. The boy’s temperature is 38.4°C, heart rate is 96 beats/min, respiratory rate is 42 breaths/min, and oxygen saturation is 92% in room air. He has ulcerative mucosal lesions involving the lips and oral mucous membranes (Item Q258), bilateral non-purulent conjunctivitis, pharyngeal erythema without exudates, and bilateral diffuse crackles on lung auscultation. Skin examination is notable for a few target lesions on the lower extremities. The remainder of his physical examination findings are unremarkable.

Of the following, the MOST likely cause of this boy’s illness is

A.	adenovirus
B.	coxsackievirus
C.	herpes simplex virus
D.	Mycoplasma pneumoniae
A

The boy in the vignette has a respiratory tract infection associated with conjunctivitis, oral mucositis, and rash. This presentation is consistent with Mycoplasma pneumoniae–induced rash and mucositis (MIRM).

PREP Pearls
Approximately 10% of individuals infected with Mycoplasma pneumoniae develop a nonspecific maculopapular rash.
Mycoplasma pneumoniae–induced rash and mucositis (MIRM) is an extrapulmonary manifestation characterized by painful erosive lesions involving 2 or more mucosal sites; skin lesions (eg, target lesions, bullae) may be absent or sparse.
Treatment with azithromycin and steroid therapy may shorten the clinical course of Mycoplasma pneumoniae–induced rash and mucositis.

186
Q

A 16-year-old adolescent boy is evaluated in the office for exercise-induced cough, upper chest tightness, and difficulty catching his breath. When he was in preschool, the boy was treated with an albuterol inhaler for wheezing with viral respiratory infections. He had no further breathing difficulties until recently, since he began training for cross country running. He was previously moderately active, but cross country running is a higher intensity exercise for him. His symptoms occur shortly after beginning to run and worsen with the intensity of the workout. His symptoms are affecting his ability to compete. The boy tried a friend’s albuterol inhaler in an attempt to treat his symptoms, without relief. He also tried using the inhaler before running, but it did not prevent his symptoms. On physical examination, the boy looks fit and well. He has normal vital signs for age, and normal lung and cardiovascular findings. Office spirometry results are normal.

Of the following, the BEST next step in this boy’s management is to

A.	begin daily inhaled corticosteroids
B.	counsel him regarding albuterol inhaler technique
C.	recommend that he choose another sport
D.	schedule a treadmill exercise provocation test
A

The boy in the vignette has exercise-related respiratory symptoms that are neither relieved nor prevented by the use of an albuterol inhaler. The boy’s symptoms are most consistent with exercise-induced laryngeal obstruction (EILO). The best next step in this boy’s management is to schedule a treadmill exercise provocation test, the best way to reproduce the situation in which he has symptoms and objectively monitor the effect of submaximal exercise on his lung function. If the boy has exercise-induced asthma (EIA), he should have at least partial relief of symptoms or some degree of symptom prevention with the use of an albuterol inhaler, even if his technique is not perfect. Although assessing his inhaler technique would be beneficial, this evaluation would likely delay his diagnosis and appropriate management.

PREP Pearls
Exercise-induced asthma is characterized by onset several minutes after completion of submaximal exercise and is relieved or prevented by bronchodilator inhalation.
A treadmill exercise provocation test is the most appropriate way to document the presence and severity of exercise-induced asthma.
Exercise-induced laryngeal obstruction is associated with neck or upper chest tightness and inspiratory stridor or stertor that may be incorrectly interpreted as wheezing. There is a direct relationship between exercise intensity and symptoms, which are not relieved by albuterol.

187
Q

A 5-year-old boy taking valproic acid for a known seizure disorder is being seen for his routine health supervision visit. He has been healthy overall and is thriving, with no recent seizure activity. He had a viral upper respiratory illness 6 months ago. There are no pertinent findings on the boy’s physical examination. His vital signs are normal for age. There is no palpable lymphadenopathy or hepatosplenomegaly as well as no rash. A screening complete blood cell count is obtained.

Laboratory Test

Result

White blood cell count

4,800/µL (4.8 × 109/L)

Neutrophils

21.2%

Lymphocytes

63.5%

Monocytes

13.2%

Eosinophils

2.1%

Hemoglobin

12.3 g/dL (123 g/L)

Platelet count

184 × 103/µL (184 × 109/L)

Mean corpuscular volume

76 fL

Reticulocyte

1%

Absolute reticulocyte count

48 × 103/µL

Of the following, the best NEXT step in management is

A.	administration of intravenous antibiotics
B.	observation with repeat complete blood cell count in 4 weeks
C.	prescription for a course of oral antibiotics
D.	referral to pediatric hematology/oncology for administration of growth colony-stimulating factor
A

The boy in the vignette has mild neutropenia (0.212 × 4,800 = ~1,018/μL), as defined by an absolute neutrophil count (ANC) less than 1,500/μL but above 1,000/μL. Neutropenia may be divided into mild (ANC 1,000 to 1,500/μL), moderate (ANC 500 to <1,000/μL), and severe (ANC <500/μL) categories, with some literature including a very severe category (ANC <200/μL).

188
Q

A term neonate is admitted to the neonatal intensive care unit for further evaluation of limb abnormalities. Physical examination of the upper extremities reveals internal rotation of the shoulders, extension of the elbows, flexion and ulnar deviation of the wrists, and finger contractures (Item Q262). Examination of the lower extremities reveals dislocation of the hips, extension of the knees, and bilateral clubfoot. The remainder of the neonate’s physical examination findings are unremarkable.

Of the following, the neonate’s MOST likely diagnosis is

A.	amnion rupture sequence
B.	arthrogryposis
C.	Pierre-Robin sequence
D.	prune-belly syndrome
A

The physical examination findings of the neonate in the vignette are most consistent with arthrogryposis (also known as arthrogryposis multiplex congenita), a disorder characterized by congenital contractures that affect 2 or more areas of the body. An isolated congenital contracture, such as isolated clubfoot, is not considered arthrogryposis. Arthrogryposis is associated with more than 400 conditions; however, less than half of cases have a known underlying genetic cause.

PREP Pearls
Arthrogryposis is a disorder characterized by congenital contractures that affect more than 1 area of the body.
The clinical features of arthrogryposis result from decreased or absent fetal joint movement in utero.
The 2 most common forms of arthrogryposis are amyoplasia (congenital absence or underdevelopment of muscles) and distal arthrogryposis.

189
Q

Of the following, the anthropometric measurement (z-score) that BEST reflects this infant’s nutritional status is

A.	abdominal girth
B.	mid–upper arm circumference
C.	weight for age D.	weight for length
A

b

190
Q

15-year-old adolescent boy is seen for a sports preparticipation evaluation to play competitive baseball. He has no significant medical history and is not taking any medications. His family history is negative for kidney disease. The boy’s weight is at the 75th percentile and height is at the 90th percentile for age. His blood pressure is 110/70 mm Hg. The remainder of his physical examination findings are normal. A urine sample was sent to the laboratory for testing after the results of a dipstick test in the office were positive for protein.

The results of urine testing are shown:

Test

Result

Appearance

Yellow

Specific gravity

1.015

Leukocyte esterase

Negative

Nitrite

Negative

Blood

Negative

Protein

2+

Red blood cells/HPF

<5

White blood cells/HPF

<5

Urine protein

60 mg/dL

Urine creatinine

100 mg/dL (8,840 µmol/L)

Of the following, the BEST next step in management of this adolescent is to

A.	order a 24-hour urine protein excretion test
B.	order a first morning urine protein:creatinine ratio
C.	provide clearance for sports participation
D.	provide referral for a pediatric nephrologist
A

The adolescent boy in the vignette has asymptomatic proteinuria, as suggested by the findings of 2+ protein and a urine protein:creatinine ratio (UPCR) of 0.6 mg/mg on random urinalysis. The best next step in his evaluation is to obtain a first morning UPCR to confirm orthostatic proteinuria. If the first morning UPCR is normal (≤0.2 mg/mg), clearance for sports participation can be given. If the first morning UPCR is greater than 0.2 mg/mg, the boy should be referred to a pediatric nephrologist, and clearance for sports participation should be withheld pending that evaluation.

PREP Pearls
Proteinuria is defined as a urine protein excretion greater than 100 mg/m2 daily or urine protein:creatinine ratio of greater than 0.2 mg/mg on a spot urine sample.
In orthostatic proteinuria, a random urine sample will be urine dipstick test positive for protein and the urine protein:creatinine ratio may be greater than 0.2 mg/mg, but on a first morning urine specimen the protein:creatinine ratio will be less than or equal to 0.2 mg/mg.
Children with orthostatic proteinuria should have a first morning urinalysis checked at each health supervision visit.

191
Q

A 16-year-old adolescent girl is seen in the office for a preparticipation physical evaluation (PPE) prior to her school softball season. She also participates in competitive weight lifting. She has a family history of hypertension. Review of medical records from her prior visits reveals blood pressure measurements of 132/93 mm Hg and 134/91 mm Hg at her most recent visits. Her blood pressure on the day of the PPE is 131/92 mm Hg. The remainder of her history and physical examination findings are unremarkable.

Of the following, the BEST approach to this girl’s sports participation is to

A.	clear her for both softball and weight lifting
B.	clear her for softball but not for weight lifting
C.	clear her for weight lifting but not for softball
D.	not clear her for sports participation until she is evaluated for hypertension
A

The girl in the vignette has stage I hypertension. Athletes with stage I hypertension can be cleared for participation in all types of sports provided they do not have other conditions that would limit participation. The girl in the vignette can be cleared for both softball and weight lifting.

PREP Pearls
Young athletes with stage I hypertension can participate in sports without restrictions.
Children and adolescents with untreated stage II hypertension should be restricted from sports with a high static component (eg, weight lifting, martial arts, and gymnastics).
Criteria for stage I hypertension in children younger than 18 years require blood pressure measurements at or above the 95th percentile but lower than the 95th percentile plus 12 mm Hg.

192
Q

A 2-year-old boy is evaluated for leg pain. He was adopted at 15 months of age and did not walk until 18 months of age. The boy has not had any fractures. He eats a balanced diet that includes table foods and cow milk. The boy’s adoptive parents know little about his biological family other than that the biological mother is very short. The boy’s physical examination findings are significant only for bowing of both legs. A right knee radiograph shows osteopenia and widened and frayed metaphyses.

Laboratory data are shown:

Laboratory Test

Result

Calcium

9.2 mg/dL (2.3 mmol/L) (reference range, 9.1-10.5 mg/dL [2.27-2.63 mmol/L])

Phosphate

1.3 mg/dL (0.42 mmol/L) (2.8-6.2 mg/dL [0.9-2.0 mmol/L])

Intact parathyroid hormone

45 pg/mL (reference range, 15-65 pg/mL)

25-hydroxyvitamin D

27 ng/mL (67.4 nmol/L) (reference range, 30-100 ng/mL [74.9-249.6 nmol/L)

1,25-dihydroxyvitamin D

50 pg/mL (120 pmol/L) (reference range, 20-80 pg/mL [48-192 pmol/L])

Alkaline phosphatase

780 U/L (reference range,100-400 U/L)

Of the following, the MOST likely cause of this boy’s findings is

A.	abnormal calcium-sensing receptor
B.	hypoparathyroidism
C.	vitamin D deficiency
D.	X-linked hypophosphatemia
A

The boy in the vignette has hypophosphatemic rickets due to X-linked hypophosphatemia (XLH), the most common inherited form of rickets. Children with XLH typically have symptoms and signs of rickets (eg, leg bowing in ambulating children, bone pain, pathologic fractures) and short stature. Dental abscesses are common. X-linked hypophosphatemia is inherited in an X-linked dominant fashion. The mother of the boy in the vignette is likely affected by XLH given her short stature.

PREP Pearls
The metabolic abnormalities associated with X-linked hypophosphatemia are due to elevated levels of fibroblast growth factor 23 (FGF23), which causes renal phosphate wasting.
X-linked hypophosphatemia is characterized by a low phosphate and inappropriately normal 1,25-dihydroxyvitamin D levels; a low serum phosphate level normally stimulates 1α-hydroxylase in the kidney to increase 1,25-dihydroxyvitamin D levels.
The traditional treatment of X-linked hypophosphatemia is oral phosphate and calcitriol (1,25-dihydroxyvitamin D) replacement. A monoclonal antibody against FGF23 (burosumab) is available to aid management of the condition.

193
Q

A girl is seen for a health supervision visit. She can skip and hop on one foot several times. She is able to write her first name, count to 10, and name 10 colors. She enjoys rhyming words and tells a story with proper sequence.

Of the following, this girl is also MOST likely able to

A.	copy a flag
B.	remember her telephone number
C.	tandem walk
D.	tie her shoelaces
A

PREP Pearls
The ability to memorize a telephone number is a developmental milestone typical of 5-year-old children. Parents should instruct their children to memorize their telephone number for safety purposes.
Language and communication milestones of a 5-year-old include a vocabulary of >2,000 words, complete sentence formation, and sequential storytelling.
Imaginative play and creativity emerge at ages 5 to 6 years.
tying shoes is a 6 yo thing

194
Q

A 3-year-old boy is evaluated for a 1-month history of intermittent fevers up to 40°C, night sweats, and 2.2-kg weight loss. He has no known sick contacts and has not traveled recently. The boy is afebrile. He is irritable but consolable and appears well nourished and well developed. There are multiple subcentimeter cervical lymph nodes palpable, more on the left side than the right, and a 4 × 3–cm, firm, immobile, nontender, left-sided, supraclavicular lymph node. His spleen is palpable 1 cm below the left costal margin. The remainder of his physical examination findings are unremarkable.

Of the following, the BEST next step in this boy’s management is referral to a(n)

A.	emergency department
B.	gastroenterologist
C.	infectious disease specialist
D.	otolaryngologist
A
195
Q

A 7-year-old boy is seen in the emergency department for left knee swelling. He sustained a fall 2 weeks ago, at which time he developed left knee swelling and tenderness. He was seen by an orthopedic surgeon who aspirated 40 mL of blood from that knee. Magnetic resonance imaging did not reveal any significant musculoskeletal abnormality other than hemarthrosis with evidence of some hemosiderin. Over the past few days, the knee swelling has worsened. There is increased swelling and pain, for which he has been taking ibuprofen, that makes it difficult for him to ambulate. He has no fever or erythema at the site of swelling.

Of the following, the BEST test to confirm this child’s diagnosis is

A.	coagulation profile with mixing studies
B.	complete blood cell count with reticulocyte count
C.	joint fluid analysis
D.	magnetic resonance imaging
A

PREP Pearls
A coagulation profile with mixing studies is helpful in evaluating a child with a suspected bleeding disorder.
An individual with hemophilia with bleeding into a joint after trauma may develop a target joint (a joint more likely to have repeated bleeding episodes).
Aspirin and nonsteroidal antiinflammatory drugs (eg, ibuprofen) should be avoided in individuals with known or suspected hemophilia due to the antiplatelet effects of these drugs.

196
Q

A 14-year-old adolescent girl is seen in the office for evaluation of right thigh pain, which began 1 month ago during soccer practice. The pain is located in her anterior right thigh over the femur midshaft. Initially, the pain occurred only with high-impact activities (eg, running), but now also occurs while walking. She recalls a collision with another soccer player around the time her pain began, but is unsure if this was directly related to the onset of her pain. She denies fever, rash, joint swelling, or change in appetite or activity level. The girl’s physical examination findings are remarkable only for mild pain with hip internal rotation and when hopping on her right leg. There are no areas of tenderness. Radiographs of the right hip and femur are unremarkable.

Of the following, the BEST next step in this girl’s evaluation and management is

A.	magnetic resonance imaging of the femur
B.	physical therapy for soft tissue mobilization
C.	splinting the leg in a position of knee flexion for 1 to 2 days
D.	use of a thigh compression wrap during soccer
A

The adolescent in the vignette has a history of gradually worsening, activity-related pain and participates in a high-impact sport. Her history and physical examination findings suggest a femur stress fracture. The best next step in her evaluation and management is magnetic resonance imaging (MRI) of the femur. Item C77 is a T2-weighted MRI scan of the femur demonstrating a right femoral neck stress fracture with edema of the inferior aspect of the femoral neck and a subtle fracture line.

197
Q

A 15-year-old adolescent girl with anorexia nervosa is admitted to the hospital with weight loss of 6 kg over the past 2 months. She has no history of vomiting, diarrhea, or abdominal pain. She is not taking any medications. The girl’s heart rate is 50 beats/min, respiratory rate 16 breaths/min, and blood pressure 100/60 mm Hg. Her weight is less than the third percentile, height is at the 25th percentile, and body mass index is 15 kg/m2. She has pallor, temporal wasting, and a decrease in subcutaneous fat. The remainder of her physical examination findings are unremarkable.

Laboratory data on admission are shown:

Laboratory Test

Result

Sodium

137 mEq/L (137 mmol/L)

Potassium

4 mEq/L (4 mmol/L)

Chloride

98 mEq/L (98 mmol/L)

Bicarbonate

22 mEq/L (22 mmol/L)

Blood urea nitrogen

7 mg/dL (2.5 mmol/L)

Creatinine

0.5 mg/dL (44.2 µmol/L)

Calcium

8.8 mg/dL (2.2 mmol/L)

Magnesium

1.8 mg/dL (0.7 mmol/L)

Phosphorus

3.9 mg/dL (1.3 mmol/L)

Glucose

90 mg/dL (5 mmol/L)

Hemoglobin

10.5 g/dL (105 g/L)

Enteral feeding is started in the hospital.

Of the following, the electrolyte abnormality MOST likely to develop in this adolescent is

A.	hypermagnesemia
B.	hyperphosphatemia
C.	hypokalemia
D.	hyponatremia
A

c

198
Q

A 15-year-old adolescent girl is evaluated in the emergency department for worsening headache and gait changes. Over the past 2 years her walking has progressively looked and felt more awkward and unbalanced. Over the past 6 months she has developed a constant dull headache with associated photophobia, nausea, and vomiting. Her headache has escalated in severity over the past 3 weeks. The headache wakes her from sleep 4 times per week. The pain is worse in the morning, while lying down, and when she coughs or sneezes. Over the past few weeks she developed blurring of her vision and intermittent paresthesias in both hands.

She is alert and oriented. Her visual acuity is 20/50 bilaterally. She has bilateral papilledema and is unable to fully abduct her eyes. The remainder of her cranial nerve findings are normal. She has full strength in all 4 extremities with diffuse hyperreflexia. Her toes are downgoing with plantar stimulation. Coordination testing shows mild dysmetria on finger to nose bilaterally. Her gait is shuffling, wide based, and hesitant; she prefers to hold on to her mother for support while ambulating. The remainder of her physical examination findings are normal.

Of the following, the BEST next step in this adolescent’s management is

A.	lumbar puncture with opening pressure measurement
B.	magnetic resonance imaging of the brain with and without contrast
C.	neurosurgical consultation for placement of a ventriculoperitoneal shunt
D.	ophthalmology consultation for evaluation of fundus and eye movements
A

The adolescent in the vignette is exhibiting symptoms and signs of increased intracranial pressure (ICP) with worsening headache, blurry vision, paresthesias, papilledema, and abnormal gait. Of the response choices, the best next management step is to obtain magnetic resonance imaging (MRI) of the brain with and without contrast to guide further diagnostic work-up. Head computed tomography (CT) without contrast would also be an appropriate option.

199
Q

7-year-old-girl is brought to the emergency department for evaluation of severe abdominal pain, nausea, vomiting, and muscle cramping. She was feeling well earlier in the day and had helped her family clean the garage. On physical examination, her temperature is 36.7°C, heart rate is 128 beats/min, respiratory rate is 22 breaths/min, and blood pressure is 130/80 mm Hg. She is lying in the fetal position, crying, and appears to be in pain. There is a 2-cm, circular lesion with central pallor and a central punctum on her right hand. The remainder of her physical examination findings are unremarkable.

Of the following, the envenomation MOST likely responsible for this girl’s symptoms is a

A.	black widow spider bite
B.	brown recluse spider bite
C.	emperor scorpion sting
D.	yellow jacket sting
A

PREP Pearls
Black widow spider venom is neurotoxic. Signs and symptoms begin suddenly, and may include nausea, vomiting, pain and pallor at the bite site, paresthesias, tenderness in regional lymph nodes, sweating, abdominal pain, and muscle cramping.
Severe signs and symptoms of a black widow spider bite include muscle rigidity, difficulty speaking, ptosis, agitation, hypertension, and respiratory distress.
Brown recluse spider venom is a cytotoxin that produces localized effects; systemic symptoms are rare. Treatment is aimed at localized wound care.

200
Q

Hematologic abnormality with Noonan?

A

bleeding diathesis, commonly Factor 11 deficiency

201
Q

DTaP is given at 2, 4, 6, 15 mos, and 5 year (kindergarten visit). The 4th dose can be given at 12 months if XX mos has elapsed since the 3rd dose

A

6 months

202
Q

A Td booster is indicated if xx years has elapsed from clean wound and xx years from dirty wound

A

10 years, 5 years

203
Q

For a dirty wound if you’ve received less than xx tetanus vax then you should get tet vax and Ig

A

less than 3

204
Q

If 1st Hib dose was given at 15 mo or older, how many more are needed?
If 1st dose was given at less than 1 year how long should you wait for the next dose?
If 1st dose was between 12-14 months how long should you wait for the next dose?

A

None
4 weeks
8 weeks

205
Q

Hep A is given at 1 year, given 2 doses how many months apart

A

6

206
Q

If less than 12 mos old and traveling to Hep A hot spot can they get Hep a vax?

A

they get Ig. If older than 12 months then they get vax

207
Q

MMR can’t be given within xx of other live vaccines. How long to wait for PPD?

A

4 weeks; give today or 4-6 weeks

208
Q

Post exposure varicella: give vaccine within xx hours and up to xx hours after exposure

A

give within 72 hours and up to 120 hours

209
Q

Catch up varicella can be given within how many months of the first if under age 13?

A

3 mos apart if under 13, 28 days apart if over 13 yo

210
Q

If first dose of PCV13 is given over age 2, when do you give the rest?

A

They don’t need anymore. If 1st dose is given before age 1 you need to wait 4 weeks before second. Between age 1-2 wait 8 weeks.
PPSV23 is 8 weeks after last PCV13 if over 2 you need to wait

211
Q

First dose of rota is given before xx weeks and completed before xx months

A

before 15 weeks and completed by 8 months. Wait 4 weeks in between doses

212
Q

Persons age 7–18 years not fully vaccinated* with DTaP: 1 dose Tdap as part of the catch-up series (preferably the first dose); if additional doses are needed, use Td or Tdap. If the first dose of Tdap/Td was given after the 1st birhtday when do you give the next dose?

A

Tdap administered at age 7–10 years
Children age 7–9 years who receive Tdap should receive the routine Tdap dose at age 11–12 years.
Children age 10 years who receive Tdap do not need the routine Tdap dose at age 11–12 years.
6 months (as final dose)
if first dose of DTaP/DT or Tdap/Td was administered at or after the 1st birthday.

213
Q

minimum dose interval IPV

A

4 weeks

214
Q

minimum dose interval Hep B

A

4 weeks between one and two, 8 weeks between two and three

215
Q

post exposure varicella

A

all unimmunized contacts over 12 mos old with no natural h/o varicella should get the vaccine within 3 but up to 5 days after exposure
if immunocompromised then they get Ig within 10 days exposure